You are on page 1of 243

BỘ GIÁO DỤC VÀ ĐÀO TẠO

VIỆN NGHIÊN CỨU CAO CẤP VỀ TOÁN

BÀI BÁO CHỦ ĐỀ HÌNH HỌC PHẲNG


ĐỀ TÀI
ỨNG DỤNG CỦA PHƯƠNG TÍCH, TRỤC ĐẲNG PHƯƠNG
VÀO BÀI TOÁN HÌNH HỌC PHẲNG
Văn Phú Quốc
GV. Trường THPT chuyên Nguyễn Bỉnh Khiêm,
tỉnh Quảng Nam
Phương tích và trục đẳng phương là một chủ đề quen thuộc và hấp dẫn trong
hình học phẳng. Các kết quả của nó vô cùng đơn giản, tự nhiên nhưng lại ảnh hưởng
sâu sắc đến các nội dung quan trọng như: tính độ dài, góc, chứng minh đẳng thức-bất
đẳng thức hình học, tập hợp điểm, điểm-đường cố định, quan hệ vuông góc, quan hệ
đồng quy-thẳng hàng…Tìm được mối liên hệ giữa phương tích và trục đẳng phương
với các nội dung trên sẽ giúp người làm toán hướng đến những lời giải hay, đẹp, gọn
gàng và ấn tượng.
A. CƠ SỞ LÝ THUYẾT
I. Phương tích của một điểm đối với đường tròn (Power of a point)
1. Định lý 1. Cho đường tròn  O; R  và điểm M cố định, đặt OM  d . Một đường
thẳng thay đổi cắt đường tròn tại hai điểm A và B . Khi đó:
MA.MB  MO 2  R 2  d 2  R 2 .
Chứng minh
Gọi C là điểm đối xứng của A qua tâm O
A Vì ABC  900 (góc nội tiếp chắn nửa đường tròn)
nên B là hình chiếu của C trên AM .
B Bây giờ ta có
       
M O  
MA.MB  MA.MB  MC .MA  MO  OC MO  OA 
     2  2
 
 MO  OA MO  OA  MO  OA
 MO 2  OA2  d 2  R 2
C
2. Định nghĩa. Đại lượng không đổi MA.MB  d 2  R 2 được gọi là phương tích của
điểm M đối với đường tròn  O  , ký hiệu PM / O  . Ta có công thức:

PM / O   MA.MB  d 2  R 2
3. Các tính chất
3.1. Tính chất 1
 Nếu A, B cố định và AB. AM  const thì M cố định.
 Điểm M nằm ngoài  O; R   PM /  O   0 .

Trang 1
 Điểm M nằm trong  O; R   PM /  O   0 .
 Điểm M nằm trên biên  O; R   PM /  O   0 .
3.2. Tính chất 2
Cho đường tròn  O; R  và một điểm M nằm ngoài đường tròn  O  . Qua M kẻ cát
 
tuyến MAB và tiếp tuyến MT với  O  . Khi đó MA.MB  MT 2  OM 2  R 2 .
3.2. Tính chất 3
Cho AB, CD là hai đường thẳng phân biệt sao cho AB  CD  M ( M  A, B, C , D ).
   
Khi đó nếu MA.MB  MC.MD thì bốn điểm A, B, C , D cùng nằm trên một đường tròn.
Chứng minh
  ABC
Giả sử đường tròn ngoại  . Khi đó:
 tiếp cắt CD
  tại
 D
MA.MB  MC.MD  MD  MD  D  D .
Vậy bốn điểm A, B, C , D cùng nằm trên một đường tròn.
3.3. Tính chất 4
Cho AB, MT là hai đường thẳng phân biệt sao cho AB  MT  M (với M  A, B, T ).
 
Khí đó nếu MA.MB  MT 2 thì đường tròn ngoại tiếp  ABT tiếp xúc với MT tại T .
4. Một số ví dụ cơ bản
4.1. Ví dụ 1. Cho  ABC nội tiếp đường tròn  O; R  và G là trọng tâm  ABC .
1
Chứng minh rằng: PG /  O   
9
 AB 2  BC 2  CA2  .
Lời giải    
Vì G là trọng tâm  ABC nên 3OG  OA  OB  OC .
     

Suy ra 9OG 2  OA2  OB 2  OC 2  2 OA.OB  OB.OC  OC.OA 
     

 3R 2  2 OA.OB  OB.OC  OC.OA  (1)
Ta có
   2  2   2
2OA.OB  OA  OB  OA  OB   OA2  OB 2  AB 2  2 R 2  AB 2 .
Tương
 tự
  
2OB.OC  2 R 2  BC 2 ; 2OC.OA  2 R 2  CA2
     
 
Suy ra 2 OA.OB  OB.OC  OC .OA  6 R 2   AB 2  BC 2  CA2  (2)
Thay (2) vào (1) ta được:
1
9OG 2  9 R 2   AB 2  BC 2  CA2   OG 2  R 2    AB 2  BC 2  CA2  .
9
1
 AB 2  BC 2  CA2  .
Do đó PG /  O   
9
[Phương tích này được gọi là phương tích trọng tâm]

Trang 2
4.2. Ví dụ 2. Cho  ABC nội tiếp đường tròn  O; R  và H là trực tâm  ABC .
Chứng minh rằng: PH / O   8 R 2 cos A cos B cos C .
Lời giải
Đối với bài toán này, ta quan tâm đến trường hợp  ABC nhọn, còn những trường hợp
vuông hoặc tù ta chứng minh tương tự.
A Gọi I , A lần lượt là giao điểm của AH với BC ,  O  .
B'
Áp dụng định lý sin trong  HAB ta có
AH AB AH AB
  
sin 
ABH sin  sin  90  A  sin 1800  C 
0
AHB
C' H O AH AB AB
   HA  .cos A  2 R cos A .
cos A sin C sin C
B I C Tương tự ta cũng có HB  2 R cos B ; HC  2 R cos C .
A'   C
Vì BHA   BA
 A nên  BHA cân tại B .
Suy ra I là trung điểm AH .
  4 R cos B cos C .
Khi đó HA  2 IH  2.HB.cos BHA
Vì  ABC nhọn nên PH / O   HA.HA   HA.HA  8 R 2 cos A cos B cos C .
[Phương tích này được gọi là phương tích trực tâm]
4.3. Ví dụ 3. Cho đường tròn  O; R  và ba điểm thẳng hàng A, B , C . Chứng minh
đẳng thức: PA/  O  .BC  PB /  O  .CA  PC / O  . AB  BC .CA. AB  0 .
Lời giải
Ta có: PA/  O  .BC  PB /  O  .CA  PC / O  . AB  BC .CA. AB  0
  OA2  R 2  .BC   OB 2  R 2  .CA   OC 2  R 2  . AB  BC .CA. AB


 OA2 .BC  OB 2 .CA  OC 2 . AB  BC.CA. AB  R 2 BC  CA  AB 
 OA2 .BC  OB 2 .CA  OC 2 . AB  BC.CA. AB
Ta sẽ chứng minh hệ thức Stewart OA2 .BC  OB 2 .CA  OC 2 . AB  BC.CA. AB  0
Trường hợp 1. Điểm O nằm trên đường thẳng chứa ba điểm A, B , C .
2 2 2
OA .BC  OB .CA  OC . AB  BC.CA. AB
2 2 2
    
 OA OC  OB  OB OA  OC  OC OB  OA  OC  OB OA  OC OB  OA     
2 2 2 2 2 2 2 2
 OA .OC  OA .OB  OB .OA  OB .OC  OC .OB  OC .OA  OA .OC  OA .OB
2 2 2 2
OB .OA  OB .OC  OC .OB  OC .OA  OA.OB.OC  OA.OB.OC  0 .

Trang 3
Trường hợp 2. Điểm O không nằm trên đường thẳng chứa ba điểm A, B , C .
O

A C
B H

Gọi H là hình chiếu của điểm O lên đường thẳng chứa ba điểm A, B , C .
2 2 2
OA .BC  OB .CA  OC . AB  BC.CA. AB
 2 2
  2 2
 OH  HA .BC  OH  HB CA  OH  HC . AB  BC.CA. AB   2 2

 OH
2
 BC  CA  AB    AH
2 2 2
.BC  BH .CA  CH . AB  BC.CA. AB 
0.
Vậy PA/  O  .BC  PB /  O  .CA  PC / O  . AB  BC .CA. AB  0 .
5. Phương tích trong hệ tọa độ Descartes
Trong mặt phẳng với hệ tọa độ Oxy , cho điểm M  x0 ; y0  và đường tròn
 C  : x2  y 2  2ax  2by  c  0 .
Đặt f  x, y   x 2  y 2  2ax  2by  c . Khi đó
PM / C   f  x0 ; y0   x02  y02  2ax0  2by0  c .
II. Trục đẳng phương của hai đường tròn (Radical axis) và tâm đẳng phương
(radical center)
1. Định lý 2. Cho hai đường tròn không đồng tâm  O1 ; R1  ,  O2 ; R2  . Tập hợp điểm M
có phương tích đối với hai đường tròn bằng nhau là một đường thẳng. Đường thẳng
này được gọi là trục đẳng phương của hai đường tròn  O1  và  O2  .
Chứng minh

Giả sử điểm M có cùng phương tích đối


với hai đường tròn  O1  và  O2  .
M
Gọi H là hình chiếu của điểm M lên
đường thẳng O1O2 và I là trung điểm đoạn
thẳng O1O2 .
O2
Khi đó
O1 H I
 MH 2  HO12    MH 2  HO22   R12  R22
 HO12  HO22  R12  R22

Trang 4

 HO1  HO2  HO  HO   R
1 2 1
2
 R22
R12  R22
 O2O1.2 HI  R12  R22  IH  .
2O1O2
Vì H là điểm cố định nên tập hợp các điểm M có cùng phương tích với hai đường
tròn là đường thẳng qua H và vuông góc với O1O2 .
2. Các tính chất về trục đẳng phương
Xét hai đường tròn  O1  và  O2  . Từ kết quả ở định lý 2 ta suy ra một số tính chất
sau:
2.1. Tính chất 1. Trục đẳng phương của hai đường tròn vuông góc với đường thẳng
nối tâm.
2.2. Tính chất 2. Nếu hai đường tròn cắt nhau tại A và B thì AB chính là trục đẳng
phương của chúng.
2.3. Tính chất 3. Nếu PM / O1   PM / O2  thì đường thẳng qua M vuông góc với O1O2 là
trục đẳng phương của hai đường tròn.
2.4. Tính chất 4. Nếu PM / O1   PM / O2  và PN / O1   PN / O2  thì đường thẳng MN chính
là trục đẳng phương của hai đường tròn.
2.5. Tính chất 5. Nếu PM / O1   PM / O2  ; PN / O1   PN / O2  và PP / O1   PP / O2  thì ba điểm
M , N , P thẳng hàng.
2.6. Tính chất 6. Nếu  O1  và  O2  tiếp xúc nhau tại A thì đường thẳng qua A và
vuông góc với O1O2 chính là trục đẳng phương của hai đường tròn.
3. Cách dựng trục đẳng phương của hai đường tròn không đồng tâm
Trong mặt phẳng, cho hai đường tròn không đồng tâm  O1  và  O2  . Xét các khả
năng có thể xảy ra:
3.1. Khả năng 1.  O1    O2    A, B . Khi đó đường thẳng AB chính là trục đẳng
phương của hai đường tròn.
3.2. Khả năng 2.  O1  và  O2  tiếp xúc nhau tại A. Khi đó tiếp tuyến chung tại A
chính là trục đẳng phương của hai đường tròn.
3.3. Khả năng 3.  O1    O2    . Ta dựng theo các bước dưới đây:
 Bước 1: Dựng đường tròn  O3  sao cho  O3    O1    A, B và
 O3    O2   C , D .
 Bước 2: AB  CD  M .
 Bước 3: Dựng đường thẳng qua M và vuông góc với đường thẳng O1O2 chính
là trục đẳng phương của  O1  và  O2  .
4. Trục đẳng phương trong hệ tọa độ Descartes
Trong mặt phẳng với hệ tọa độ Oxy , cho hai đường tròn không đồng tâm:
 C1  : x 2  y 2  2a1 x  2b1 y  c1  0 và  C2  : x 2  y 2  2a2 x  2b2 y  c2  0

Trang 5
(với ai2  bi2  ci  0 , i  1,2 ).
Giả sử điểm M  x0 ; y0  .
Ta có PM / C1   PM /  C2   x02  y02  2a2 x0  2b2 y0  c2  x02  y02  2a2 x0  2b2 y0  c2
 2  a1  a2  x0  2  b1  b2  y0  c1  c2  0 .
Vậy trục đẳng phương của hai đường tròn  C1  và  C2  có phương trình:
2  a1  a2  x  2  b1  b2  y  c1  c2  0 .
5. Định lý 3. Cho ba đường tròn  O1  ,  O2  và  O3  . Khi đó ba trục đẳng phương của
các cặp đường tròn hoặc trùng nhau hoặc song song hoặc cùng đi qua một điểm. Điểm
đó là tâm đẳng phương của ba đường tròn.
Chứng minh
Gọi  ij là trục đẳng phương của hai đường tròn  Oi  và  O j  . Ta xem xét các trường
hợp sau đây:
Trường hợp 1. Giả sử có một cặp đường thẳng song song. Không giảm tính tổng quát,
ta có thể giả sử 12 //  23 . Ta có 12  O1O2 ,  23  O2O3 . Điều này dẫn đến O1 , O2 , O3
thẳng hàng. Lại có 13  O1O3 nên 13 //  23 // 12 .
Trường hợp 2. Giả sử có một cặp đường thẳng cắt nhau. Không giảm tính tổng quát, ta
có thể giả sử 12   23  M .
12

O1 O2

23
13

O3

 PM /  O1   PM / O2 
Bây giờ ta có   PM / O1   PM /  O3   M  13 .
P
 M /  O2   PM / O3 

Bắt đầu từ đây, ta suy ra:


- Nếu có hai đường thẳng trùng nhau thì đó cũng là trục đẳng phương của cặp đường
tròn còn lại.
- Nếu hai trục đẳng phương chỉ cắt nhau tại một điểm thì điểm đó cũng thuộc trục
đẳng phương còn lại.

Trang 6
6. Các tính chất về tâm đẳng phương
6.1. Tính chất 1. Nếu ba đường tròn đôi một cắt nhau thì các dây cung cùng đi qua
một điểm.
6.2. Tính chất 2. Nếu ba trục đẳng phương song song hoặc trùng nhau thì tâm của ba
đường tròn thẳng hàng.
6.3. Tính chất 3. Nếu ba đường tròn cùng đi qua một điểm và có các tâm thẳng hàng
thì các trục đẳng phương trùng nhau.
7. Một số ví dụ cơ bản
7.1. Ví dụ 1. Cho  ABC , một đường thẳng song song với BC cắt AB, AC tương ứng
tại D, E . Xác định trục đẳng phương của các đường tròn đường kính BE , CD .
Lời giải
A Gọi  C1  ,  C2  lần lượt là các đường tròn đường
kính BE , CD .
C'
Gọi AA, BB, CC  là các đường cao của  ABC
B' và H là trực tâm.
Dễ thấy các tứ giác HACB , HABC  nội tiếp,
H
cho nên
D E  AB. AC  AH . AA
  AB. AC  AC . AB (1)
C   AC . AB  AH . AA
B A'
AC AB
Vì DE // BC nên  (2)
AE AD
Từ (1) và (2) suy ra AB. AE  AC '. AD
 PA/ C1   PA/  C2 
Suy ra A nằm trên trục đẳng phương của hai đường tròn  C1  và  C2  .
Hơn nữa, ta thấy AA vuông góc với đường nối tâm của hai đường tròn  C1  và  C2 
Vậy AA là trục đẳng phương của  C1  và  C2  .
7.2. Ví dụ 2. Cho đường tròn  O  và hai điểm P, Q cố định ( P nằm ngoài  O  và Q
nằm trong  O  ). Dây cung AB của  O  luôn đi qua Q . PA, PB lần lượt cắt  O  lần
thứ hai tại D, C . Chứng minh rằng CD luôn đi qua một điểm cố định.
Lời giải
A Gọi E là giao điểm thứ hai của PQ với đường tròn
D  PAB  và F là giao điểm của CD với PQ .
P E Ta có OQ 2  R 2  QA.QB  QP.QE .
F Q
O
C Mà P, Q cố định nên QP không đổi. Suy ra QE không
B đổi. Do đó E cố định.

Trang 7
  PBA
Mặt khác, PDC   PEA nên tứ giác DAEF nội tiếp. Dẫn đến
PO 2  R 2  PD.PA  PE.PF .
Vì P, E cố định nên PF không đổi. Do đó F cố định.
Vậy đường thẳng CD luôn đi qua một điểm cố định F .
7.3. Ví dụ 3. Cho đường tròn  O  đường kính AB và CD . Tiếp tuyến tại B của  O 
cắt AC tại E , DE cắt  O  tại điểm thứ hai F . Chứng minh rằng AF , BC , OE đồng
quy.
Lời giải
Ký hiệu  C1  ,  C2  tương ứng là đường tròn ngoại
tiếp của các tam giác AEF , BCE .
E
B Ta có AF , BC tương ứng là trục đẳng phương của
F
 O  và  C1  ,  O  và  C2  .
D Mặt khác OAF  FDB   FEA ; OBC
  CEB
.
C
O
Suy ra OA, OB lần lượt là tiếp tuyến của  C1  ,  C2  .
A Kết hợp với OA2  OB 2 ta suy ra OE là trục đẳng
phương của  C1  và  C2  .
Suy ra AF , BC , OE đồng quy tại tâm đẳng phương
của ba đường tròn. [tư tưởng của định lý 3].

B. MỘT SỐ ỨNG DỤNG


I. Ứng dụng 1: Chứng minh một số định lý về hình học phẳng
1.1. [Định lý Brianchon]. Cho ABCDEF là một lục giác ngoại tiếp đường tròn  O  .
Khi đó AD, BE , CF đồng quy.
Chứng minh
Gọi G , H , I , J , K , L tương ứng là tiếp điểm của
AB, BC , CD, DE , EF , FA với  O  .
S
O1 Trên các tia KF , HB, GB, JD, ID, LF lần lượt
lấy các điểm P, S , Q, R, N , M sao cho
P KP  SH  GQ  JR  IN  LM .
Dựng đường tròn  O1  tiếp xúc với EF , CB tại
A G B Q
L
P, S ; đường tròn  O2  tiếp xúc với AF , CD tại
H O3
F C M , N và đường tròn  O3  tiếp xúc với AB, ED
I R
K tại Q, R .
J D
E
Ta có FP  PK  FK  LM  LF  FM
CS  SH  HC  IN  IC  CN
M O2

Trang 8
Suy ra FC là trục đẳng phương của các đường tròn  O1  và  O2  .
Chứng minh tương tự, ta cũng thu được: AD là trục đẳng phương của các đường tròn
 O2  và  O3  ; BE là trục đẳng phương của  O3  và  O1  .
Áp dụng định lý 3 [về tâm đẳng phương] ta đi đến AD, BE , CF đồng quy.
1.2. [Hệ thức Euler]. Cho  ABC nội tiếp đường tròn  O; R  và ngoại tiếp đường tròn
 I ; R  . Chứng minh rằng OI 2  R 2  2 Rr .
Chứng minh
Gọi E là tiếp điểm của AB và  I ; r  ;
A gọi A '  AI   O; R  .
IE r
Ta có IA   (do AA là phân giác

sin IAE A
sin
E I 2
 ).
BAC
O
  1800     A  B  IBA

B
Mặt khác, BIA
C AIE  BIE
2 2
 
 AIB cân tại A  IA  BA .
Áp dụng sin cho  BAA ta có:
A'
A A
BA  2 R sin  IA  2 R sin .
2 2
Vì I là điểm nằm bên trong  O; R  nên
r A
PI /  O    IA.IA   .2 R sin  2 Rr  OI 2  R 2  2 Rr  OI 2  R 2  2 Rr .
A 2
sin
2
1.3. [Định lý Fuss]. Cho tứ giác ABCD nội tiếp đường tròn  O; R  và ngoại tiếp
1 1 1
đường tròn  I ; r  . Đặt OI  d  0 . Khi đó 2
 2
 .
R  d R  d  r2
Chứng minh
A
Kéo dài BI , DI cắt  O  tại M , N .
E   IBC
 MNC 
N Ta có: 
B   IDC
 NMC 
O
 1 
  IDC ABC  
I  MNC NMC  IBC
2
ADC  900 
M
O là trung điểm đoạn thẳng MN .
D F C
Sử dụng công thức đường trung tuyến trong  IMN có

Trang 9
2 2 IM 2  IN 2 MN 2 IM 2  IN 2
d  OI     R2
2 4 2
2 2
IM  IN
Suy ra R 2  d 2  .
2
1 1 2  R 2  d 2  IM 2  IN 2
Do đó 2
 2
 2
 2
 R  d   R  d   R2  d 2   
PI /  O 
IM 2 IN 2 1 1
 2 2
 2 2
 2 2
IM .IB IN .ID IB ID

2 ABC

2 ADC
 sin 2 IDF
sin 2 IBE  sin  sin
   2 2  1 .
2 2
IE IF 2 r2
II. Ứng dụng 2: Chứng minh quan hệ vuông góc
2.1. [Mathematical Olympiad Summer Program (MOSP) 1995]. Cho  ABC có
đường cao BD và CE cắt nhau tại H . M là trung điểm của BC , N là giao điểm của
DE và BC .
Chứng minh rằng NH  AM .
Lời giải
Gọi O, I tương ứng là trung điểm của
A AH , MH .
  DAH
Ta có DEH   DBC   FEH
 FED  2.FEH
  2.DBC  DMC 
O D  tức giác EDMF nội tiếp
 NE.ND  NF .NM  N nằm trên trục
E H đẳng phương của đường tròn đường kính
MH và đường tròn đường kính AH .
I
N C Mặt khác, H  O   I   NH là trục
B F M
đẳng phương của  O  và  I  .
Suy ra NH  OI . Mà OI // AM (do OI là đường trung bình  AHM ) nên
NH  AM .
2.2. [Iran 1996]. Cho hai điểm D, E tương ứng nằm nằm trên các cạnh AB, AC của
 ABC sao cho DE // BC . Gọi P là điểm bất kì nằm bên trong  ABC ; các đường
thẳng PB, PC lần lượt cắt DE tại F và G . Gọi O1 , O2 lần lượt là tâm đường tròn
ngoại tiếp các tam giác PDG , PFE . Chứng minh AP  O1O2 .

Trang 10
Lời giải
Gọi M , N lần lượt là giao điểm thứ
A hai của AB với  O1  , của AC với
 O2  .
D E   PGD
Ta có PMD   PCB   tứ giác
G F
BPCM nội tiếp.
Tương tự ta cũng chứng minh được
P O2 tứ giác BPCN nội tiếp.
B C
Điều đó dẫn đến tứ giác BMNC nội
O1 N
tiếp. Mà DE // BC cho nên ta thu
được tứ giác MDEN nội tiếp.
Áp dụng định lý 3 [về tâm đẳng
phương] cho các đường tròn
 DGP  ,  PEF  và  DENM  ta có
M
DM  EN  A nằm trên trục đẳng
phương của  O1  và  O2  .
P nằm trên trục đẳng phương  AP là trục đẳng phương của  O1  và  O2  .
Do đó AP  O1O2 (điều phải chứng minh).
2.3. [IMO 1985]. Cho  ABC . Đường tròn tâm O đi qua các điểm A, C và lại cắt các
đoạn AB, BC theo thứ tự tại hai điểm phân biệt K , N . Giả sử đường tròn ngoại tiếp
của các tam giác ABC và KBN cắt nhau tại B và M . Chứng minh OMB   900 .
Lời giải
D Gọi D  AC  KN .
Ta có
  BMA  KMB
  1800   
A
KMA   
ACB  1800  KNB 

 KNB .
ACB  KDA
 tứ giác AKMD nội tiếp
   1800  
AMD  AKD ACB  1800   AMB
M K Do đó B, M , D thẳng hàng.
O Gọi R là bán kính của đường tròn  O  .
2 2
C  BM .BD  BK .BA  BO  R
B N Ta có  2 2
 *
 DM .DB  DC.DA  DO  R
  BM  DM  .BD  BO 2  DO 2  2 R 2
 BD 2  BO 2  DO 2  2 R 2 .

Trang 11
Từ (*) ta suy ra:
 BO 2  R 2   DO 2  R 2   BO  DO  2 R  BO  DO 
2 2 2 2 2 2 2
2 2
BM  DM      
 BD   BD  BD 2
  900 .
 BO 2  DO 2  BMO

2.4. [Belarus 2000]. Gọi P là giao điểm của đường chéo AC và BD của tứ giác lồi
ABCD trong đó AB  AC  BD . Gọi O và I lần lượt là tâm đường tròn ngoại tiếp và
nội tiếp của  ABP . Nếu minh rằng nếu O không trùng với I thì OI  CD .
Lời giải
Đầu tiên ta chứng minh một luận đề hết sức hữu
ích về hai đường vuông góc. Cho hai đường XY
và UV , đặt X , Y  lần lượt là chân đường vuông
A góc của X và Y tương ứng, nối đường thẳng
T UV . Sử dụng trực tiếp khoảng cách, XY  UV
nếu và chỉ nếu UX   X V  UY   Y V .
O B Vì UX   X V  UV  UY   Y V , phép tính trên
I đạt được nếu và chỉ nếu
UX 2  X V  UY 2  Y V 2 hay
UX 2  XV 2  UY 2  YV 2 .
P Do đó nó thỏa mãn đẳng thức
D DO 2  CO 2  DI 2  CI 2 .
Đặt AB  AC  BD  p, PC  a và PD  b .
C Thế thì AP  p  a và BP  p  b . Gọi R là bán
kính đường tròn ngoại tiếp  ABP . Sử dụng định
lý về phương tích của một điểm, ta có pb  DP.DB  DO 2  R 2 .
Tương tự, pa  CO 2  R 2 .
Do đó DO 2  CO 2  p  b  a  .
Vì  ABD là cân với BA  BD và I nằm trên đường phân giác của  ABD  ID  IA .
Ngoài ra IB  IC . Gọi T là điểm tiếp xúc của đường tròn nội tiếp  ABC với cạnh
AB  AP  BP p  b  a p  a b
AB . Thế thì AT   và BT  bởi vì
2 2 2
IT  AB, AI 2  BI 2  AT 2  BT 2 . Đặt các tham số lại với nhau chúng ta thấy rằng
DI 2  CI 2  AI 2  BI 2  AT 2  BT 2   AT  BT  AT  BT   p  b  a   DO 2  CO 2 .
Suy ra OI  CD .
III. Ứng dụng 3: Chứng minh quan hệ đồng quy, thẳng hàng
3.1. [Vietnam TST 2009]. Cho  ABC nhọn nội tiếp đường tròn  O  . Các điểm
A1 , B1 , C1 lần lượt là chân đường vuông góc của A, B , C xuống các cạnh đối diện. Các
điểm A2 , B2 , C2 đối xứng với A1 , B1 , C1 qua trung điểm BC , CA, AB . Đường tròn ngoại

Trang 12
tiếp các tam giác AB2C2 , BC2 A2 , CA2 B2 cắt  O  lần thứ hai tại A3 , B3 , C3 . Chứng minh
các đường thẳng A1 A3 , B1 B3 , C1C3 đồng quy.
Lời giải
A A3 Gọi  I ; R  là đường tròn ngoại tiếp  AB2C2 ,
M là trung điểm AB và G  AM  A1 A3 .
I  AC . AB  AB1. AC
C2 Ta có  1
B1  AC1  BC2 , AB1  CB2
B2  BC2 .BA  CB2 .CA
O
C1 G  BI 2  R 2  CI 2  R 2
 BI  CI
 IO  BC
B A1 M A2 C Mà IO  AA3 cho nên BC // AA3 .
1
Dễ dàng có A1M  AA3 .
2
AG AA3
Theo định lý Thales ta có   2.
MG A1M
Suy ra G là trọng tâm  ABC .
Chứng minh tương tự, ta cũng suy ra được B1 B3 , C1C3 cũng đi qua G .
Vậy A1 A3 , B1 B3 , C1C3 đồng quy tại trọng tâm G của  ABC .
3.2. [IMO Shortlist 2013, Thailand]. Cho  ABC nhọn với H là trực tâm. Gọi W là
điểm trên cạnh BC . Gọi M , N tương ứng là chân đường cao kẻ từ đỉnh B và C . Gọi
1 là đường tròn ngoại tiếp  BWN và X là điểm trên 1 đầu mút của đường kính,
đối diện với W . Tương tự, 2 là đường tròn ngoại tiếp CWM và Y là điểm trên 2
đầu mút của đường kính, đối diện với W . Chứng minh rằng X , Y , H thẳng hàng.
Lời giải
A Gọi L là chân đường cao kẻ từ A và Z là
Y giao điểm thứ hai khác W của hai đường tròn
1 , 2 .
M
Ta sẽ chứng minh X , Y , Z và H cùng nằm
N
H Z trên một đường thẳng.
Từ BNC   BMC  900 ta suy ra B, C , N và
X M cùng nằm trên một đường tròn, gọi đường
C
tròn này là 3 .
B L W
Để ý rằng WZ , BN , CM tương ứng là trục
đẳng phương của các cặp đường tròn
1 và 2 ; 1 và 3 ; 2 và 3 .

Trang 13
Thế thì A  BN  CM là tâm đẳng phương của ba đường tròn và do đó WZ qua A.
Vì WX ,WY tương ứng là đường kính của 1 và 2 nên ta có WZX  WZY   900 , vì
vậy X , Y nằm trên đường thẳng qua Z , vuông góc với WZ .
Vì tứ giác BLHN có hai góc đối diện vuông nên nội tiếp đường tròn. Từ phương tích
của điểm A đối với các đường tròn 1 và  BLHN  ta được:
AL. AH  AB. AN  AW . AZ .
AZ AL
Nếu H nằm trên đường thẳng AW thì H  Z lập tức. Mặt khác, vì  nên
AH AW
các tam giác AHZ , AWL đồng dạng.
  WLA
Vì thế, HZA   900 . Vậy H cũng nằm trên đường thẳng XYZ .
3.3.[Iran NMO 2001]. Cho  ABC nội tiếp đường tròn  O  . Gọi  I  ,  I a  lần lượt là
đường tròn nội tiếp, bàng tiếp góc A. Giả sử I a I cắt BC và  O  lần lượt tại A, M
 và NI , NI cắt  O  lần lượt tại S , T .
 M  A . Gọi N là trung điểm cung MBA a

Chứng minh rằng S , T , A thẳng hàng.


Lời giải

  1 NA  1   
A
NTS
2
  
AS  NM
2

 AS  NIM

 I 
aTS  I a IS  tứ giác I aTIS nội tiếp đường

S tròn và ký hiệu đường tròn này là 1  .


I O
  ICI
Mặt khác, IBI   900 nên IBI C nội
N a a a
A' tiếp đường tròn và ký hiệu đường tròn này là
B C
2  .
T Nhận thấy I a I , BC , TS tương ứng là trục đẳng
M
phương của các cặp đường tròn 1  và 2  ,
O
và 2  ,  O  và 1  . Do đó I a I , BC , TS
Ia đồng quy tại A . Vậy ba điểm S , T , A thẳng
hàng.
3.4. [IMO 1995]. Trên đường thẳng d lấy bốn điểm A, B, C , D (theo thứ tự đó).
Đường tròn đường kính AC và BD cắt nhau tại X , Y . Đường thẳng XY cắt BC tại
Z . Lấy P là một điểm trên XY khác Z . Đường thẳng CP cắt đường tròn đường kính
AC tại điểm thứ hai là M và BP cắt đường tròn đường kính BD tại điểm thứ hai N .
Chứng minh rằng AM , DN , XY đồng quy.

Trang 14
Lời giải
P  Nếu P  X thì M  X , N  X . Khi đó
AM , DN , XY đồng quy.
 Nếu P  Y thì chứng minh tương tự trường
X hợp trên.
N  Nếu P  X , P  Y .
M
J Gọi J  AM  XY , J   DN  XY .
Tứ giác JMCZ nội tiếp nên PJ .PZ  PM .PC .
A B Z C D Tương tự PJ .PZ  PN .PB .
Do P nằm trên trục đẳng phương của đường
tròn đường kính AC và đường tròn đường kính
BD nên PM .PC  PN .PB
 PJ .PZ  PJ .PZ  J  J  .
Y
Vậy AM , DN , XY đồng quy.

3.5. [Luyện thi Quốc gia, Quốc tế của TP.Hồ Chí Minh]. Cho hai điểm A, B phân
biệt nằm trên đường tròn  O  và C nằm ngoài  O  . Gọi CS và CT là các tiếp tuyến
của C với  O  với S , T là các tiếp điểm, M là trung điểm của cung nhỏ AB . Các
đường thẳng MS , MT cắt AB lần lượt tại E , F . Đường thẳng đi qua E , F vuông góc
với AB cắt OS , OT lần lượt tại X , Y . Một đường thẳng bất kì qua C cắt  O  tại
P, Q ( P nằm giữa C và Q ). Gọi R là giao của MP với AB , Z là tâm đường tròn
ngoại tiếp tam giác  PQR  . Chứng minh rằng X , Y , Z thẳng hàng.
Lời giải
Không mất tính tổng quát, ta giả sử E nằm
X ngoài đoạn AB .
E
Ta thấy rằng EX // OM , mà OSM cân tại
S O nên  XES cân tại X . Xét đường tròn
A ( X , XE ) , dễ thấy rằng đường tròn này tiếp
Q M xúc với  O  tại S và SC là tiếp tuyến
P
O
chung trong của hai đường tròn này.
C
F
Ta có: MAB   MPA nên hai MAR và
Y
B MPA đồng dạng. Tương tự, MAE và
MSA cũng đồng dạng.
T MA MR MA ME
Từ đó suy ra  , 
MP MA MS MA
Z
hay MP.MR  MS .ME .
Suy ra M nằm trên trục đẳng phương của
R
đường tròn ( X ) và ( PQR ) .

Trang 15
Ta cũng có CS 2  CP.CQ nên C cũng nằm trên trục đẳng phương của hai đường tròn
( X ) và ( PQR ) .
Từ đó suy ra CM là trục đẳng phương của ( X ) và ( PQR ) hay CM  XZ .
Hoàn toàn tương tự, ta cũng có CM  YZ . Vậy X , Y , Z thẳng hàng.
IV. Ứng dụng 4: Chứng minh một tập hợp điểm cùng nằm trên một đường tròn
4.1. [APMO 2009]. Trong cùng một mặt phẳng, cho ba đường tròn 1 ,  2 , 3 không đè
lên nhau và tiếp nhau ngoài nhau. Với mỗi điểm P trong mặt phẳng đó và nằm ngoài
cả ba vòng tròn, ta dựng sáu điểm A1 , B1 , A2 , B2 , A3 , B3 theo cách sau đây:
Với mọi i  1, 2,3 các điểm Ai , Bi phân biệt và cùng nằm trên i sao cho các đường
thẳng PAi , PBi đều tiếp xúc với i . Ta gọi điểm P là điểm ngoại lệ nếu từ cách dựng
trên, ba đường thẳng A1 B1 , A2 B2 , A3 B3 đồng quy. Chứng minh rằng mọi điểm ngoại lệ
nếu có cùng nằm trên một đường tròn.
Lời giải
Gọi Oi và ri lần lượt là tâm và bán
Q
kính của i với i  1, 2,3 .
Giả sử P là điểm ngoại lệ và ba
đường thẳng A1 B1 , A2 B2 , A3 B3 đồng
A1 O quy tại Q . Dựng đường tròn đường
r kính PQ . Gọi  là đường tròn tâm
O bán kính r . Ta sẽ chứng minh
mọi điểm ngoại lệ đều nằm trên  .
O1 P Giả sử PO1  A1 B1  X 1 .
X1
Vì PO1  A1 B1 nên X 1 nằm trên  .
Do PA1 là một tiếp tuyến của 1 nên
 PA1O1 vuông và đồng dạng với
B1
OX OA
 A1 X 1O1 . Suy ra 1 1  1 1 hay
O1 A1 O1P
O1 X 1.O1 P  O1 A12  r12 .
Mặt khác, O1 X 1 .O1 P cũng là phương tích của điểm O1 đối với  nên
r12  O1 X 1.O1 P   O1O  r  O1O  r   O1O 2  r 2 và do đó
r 2  OO12  r12   OO1  r1  OO1  r1  .
Như vậy r 2 chính là phương tích của điểm O đối với 1 . Lý luận tương tự r 2 cũng là
phương tích của điểm O đối với đường tròn  2 và 3 . Suy ra O phải nằm trên tâm
đẳng phương của ba đường tròn đã cho.
Do r là căn bậc hai phương tích của điểm O đối với ba đường tròn đã cho nên r
không phụ thuộc vào vị trí điểm P . Điều này có nghĩa là mọi điểm ngoại lệ đều nằm
trên  .

Trang 16
Nhận xét. Trong trường hợp tâm đẳng phương của ba đường tròn nằm tại vô cùng
(khi đó ba trục đẳng phương song song với nhau), sẽ không có điểm ngoại lệ nào nằm
trong mặt phẳng, điều này đã giải thích từ “nếu có” ở đề bài.
4.2. [IMO 2008]. Cho  ABC nhọn với trực tâm H . Gọi M 1 , M 2 , M 3 lần lượt là trung
điểm BC , CA, AB . Đường tròn tâm M 1 bán kính M 1 H cắt BC tại A1 , A2 ; đường tròn
tâm M 2 bán kính M 2 H cắt AC tại B1 , B2 ; đường tròn tâm M 3 bán kính M 3 H cắt
AB tại C1 , C2 . Chứng minh rằng sáu điểm A1 , A2 , B1 , B2 , C1 , C2 cùng thuộc một đường
tròn.
Lời giải
A Do M 1M 2 // AB ( M 1M 2 đường trung bình) và
B2 AB  HC ( H trực tâm) nên M 1M 2  HC .
C1  CA1 .CA2  CB1.CB2
 Bốn điểm A1 , A2 , B1 , B2 thuộc đường tròn và
M3 M2 gọi đường tròn này là 1  .
Chứng minh tương tự ta thu được:
H
C2 - Bốn điểm A1 , A2 , B1 , B2 thuộc đường tròn 2 
B1
- Bốn điểm C1 , C2 , B1 , B2 thuộc đường tròn 3 
B A1 M1 A2 C Nếu sáu điểm A , A , B , B , C , C không cùng
1 2 1 2 1 2

thuộc một đường tròn thì các trục đẳng phương


của ba đường tròn 1  , 2  , 3  phải đồng
quy tại một điểm nhưng chúng lại cắt nhau tại
A, B , C nên đây là điều vô lý. Vậy mệnh đề phản chứng là sai, ta đi đến kết luận của
bài toán.
4.3.[IMO Shortlist 2006, Ukraine]. Cho hình thang ABCD với hai cạnh song song
AB  CD . Hai điểm K và L nằm trên các đoạn thẳng AB và CD theo thứ tự thỏa
AK DL
mãn  . Giả sử rằng có các điểm P và Q nằm trên đoạn KL thỏa mãn
KB LC

APB  BCD và CQD  ABC . Chứng minh rằng các điểm P, Q, B, C cùng thuộc một
đường tròn.
Lời giải
AK DL
Từ giả thiết, ta có   AD, BC , KL đồng quy tại E .
BK CL
Dựng đường tròn  O1  qua hai điểm C , D và tiếp xúc với BC ; dựng đường tròn  O2 
qua hai điểm A, B và tiếp xúc với BC . Khi đó DQC  ABC  DCE  nên Q   O  .
1

Chứng minh tương tự ta cũng được P   O2  .


Gọi F là giao điểm thứ hai của EQ với  O1  .

Trang 17
2
E Ta có EF .EQ  EC (1)
 
F
Mặt khác, O 1CD  O2 BA  AO2 B  DO1C

O C DC EC
D L C  1    k  E , O1 , O2 thẳng hàng
O2 B AB EB
O1 P
EO1  
và  k  EO1  k EO2 .
EO2
Q
Suy ra phép vị tự V E ,k  :  O1    O2  .
A K B
Mà E , F , P thẳng hàng, F   O1  , P   O2  dẫn
  EF EC
O2 đến EF  k EP  k (2)
EP EB
Từ (1) và (2) suy ra EP.EQ  EC.EB .
Vậy bốn điểm P, Q, B, C cùng thuộc một đường
tròn.

4.4. [International Zhautykov Olympiad 2008]. Trong mặt phẳng cho hai đường
tròn  O1  và  O2  . Gọi A1 A2 là tiếp tuyến chung của hai đường tròn
 A  O  , A  O  , K
1 1 2 2 là trung điểm A1 A2 . Từ K lần lượt kẻ hai tiếp tuyến
KB1 , KB2 đến  O1  ,  O2  . Gọi L  A1 B1  A2 B2 , P  KL  O1O2 . Chứng minh rằng
B1 , B2 , P, L cùng nằm trên một đường tròn.
Lời giải
A1 Do KA1  KA2  KB1  KB2 nên tứ giác
K
A2 A1 B1B2 A2 nội tiếp đường tròn
 LB1.LA1  LB2 .LA2
P  KL là trục đẳng phương của hai
O1
B1 B2
O2 đường tròn  O1  và  O2 
 KL  O1O2 .
L
Để ý rằng ba điểm A1 , B1 , P nhìn đoạn
O1 K dưới một góc 900 nên nội tiếp.
Tương tự, ta cũng chứng minh được tứ giác A2 B2 PK nội tiếp.
Áp dụng định lý Miquel ta suy ra tứ giác B1 PB2 L nội tiếp.
Vậy B1 , B2 , P, L cùng nằm trên một đường tròn.

Trang 18
V. Ứng dụng 5: Chứng minh bài toán điểm-đường cố định
5.1. [VMO 2014]. Cho  ABC nhọn nội tiếp đường tròn  O  , trong đó B, C cố định
và A thay đổi trên  O  . Trên các tia AB và AC lần lượt lấy các điểm M và N sao
cho MA  MC và NA  NB . Các đường tròn ngoại tiếp các tam giác AMN và ABC
cắt nhau tại P  P  A  . Đường thẳng MN cắt BC tại Q .
a) Chứng minh rằng ba điểm A, P , Q thẳng hàng.
b) Gọi D là trung điểm của BC . Các đường tròn có tâm là M , N và cùng đi qua A
cắt nhau tại K  K  A  . Đường thẳng qua A vuông góc với AK cắt BC tại E .
Đường tròn ngoại tiếp  ADE cắt  O  tại F  F  A . Chứng minh rằng AF đi qua
một điểm cố định.
Lời giải

a) Không mất tính tổng quát, ta giả sử


A
AB  AC như hình vẽ, các trường
hợp còn loại hoàn toàn tương tự.
Khi đó, M nằm ngoài đoạn AB và
N N nằm trong đoạn AC . Vì NA  NB
O
nên NBA   NAB
 ; lại vì MA  MC
Q D
B
C   MAC
nên MCA .
E
Suy ra NBA   MCA hay tứ giác
F
P d
M K BMCN nội tiếp và ta được:
QM .QN  QB.QC .
Từ đây suy ra Q có cùng phương tích
I
đến hai đường tròn  O  và  AMN 
nên nó nằm trên trục đẳng phương
AP của hai đường tròn này. Do đó
A, P , Q thẳng hàng.
b) Ta thấy rằng đường tròn  ODC  tiếp xúc với  O  tại C nên trục đẳng phương của
hai đường tròn này là tiếp tuyến d của  O  tại C .
Ta cần chứng minh O   ADE  . Thật vậy, ta có O, M cùng nằm trên trung trực của
AC nên OM  AC . Tương tự, ON  AB cho nên O là trực tâm  AMN .
Suy ra AO  MN .
Xét hai đường tròn  M , MA  ,  N , NA  ta thấy AK  MN . Suy ra A, O, K thẳng hàng
  900 . Mặt khác, ODE
nên OAE   900 nên tứ giác AODE nội tiếp hay O   ADE  .
Do đó, trục đẳng phương của  ADE  và  ODC  chính là OD . Hơn nữa, trục đẳng
phương của  O  và  ADE  là AF .

Trang 19
Nhận thấy ba đường tròn  O  ,  ADE  ,  ODC  đôi một có các trục đẳng phương là
OD, d , AF nên theo định lý 3 [về tâm đẳng phương] ba đường này đồng quy tại một
điểm. Vậy AF đi qua giao điểm I  OD  d và I là một điểm cố định.
5.2. [Chọn đội tuyển phổ thông năng khiếu (PTNK) 2008].
Cho tam giác ABC có đỉnh A cố định và B, C thay đổi trên đường thẳng d cố định
sao cho nếu gọi A là hình chiếu của A lên d thì AB. AC âm và không đổi. Gọi M
là hình chiếu của A lên AB . Gọi N là hình chiếu của A lên AC . Gọi K là giao
điểm của các tiếp tuyến của đường tròn ngoại tiếp  AMN tại M và N . Chứng minh
rằng K thuộc đường thẳng cố định.
Lời giải
2
A Ta có AM . AB  AA  AN . AC
 tứ giác BMNC nội tiếp
 AMN   ACB .
O
N Lại có ADB  
 ACB nên  AMN   ADB .
Điều này dẫn đến tứ giác MPDB nội tiếp.
P I 2
M Do đó AP. AD  AM . AB  AA .
B A' C Mà A, A, D cố định nên P cố định.
Gọi H là hình chiếu của K trên AA .
D 1
H K Khi đó OP.OH  OI .OK  ON 2  AA2 .
4
Mà A, P, A, O cố định nên H cố định.
Vậy K thuộc đường thẳng qua H và vuông góc với AA .
5.3. [VMO 2003]. Cho đường tròn O1; R1  tiếp xúc ngoài với đường tròn O2 ; R2  tại
M với R2  R1 . Xét điểm A di động trên đường tròn O2 ; R2  sao cho A, O1 , O2
không thẳng hàng. Từ A kẻ tiếp tuyến AB, AC đến O1  . Các đường thẳng MB, MC
cắt O2  tại E , F . Gọi D là giao điểm của EF với tiếp tuyến tại A của đường tròn
O2  . Chứng minh rằng D di động trên một đường thẳng cố định.
Lời giải
D Qua M kẻ tiếp tuyến chung của O1  và O2  .
  CMy
Ta có MCA   FMD   FAM 
A
B
FAM FCA (g.g)
F
 FA2  FM .FC  FO12  R12 (1)
O2 M
O1 Tương tự, EA  EO  R
2
1
2
1
2
(2)

y
E

Trang 20
Ký hiệu  A;0 là đường tròn tâm A, bán kính R  0 . Từ (1) và (2) ta suy ra EF là
trục đẳng phương của  A;0 với O1  .
Mà D nằm trên EF nên DA2  DO12  R12  PD / O1   PD /  O2  .
Vậy D nằm trên trục đẳng phương của hai đường tròn cố định O1  và O2  .
5.4.[Đề đề nghị kỳ thi Olympic duyên hải và đồng bằng Bắc Bộ năm 2014].
Cho  ABC vuông ở A, đường cao AH . Gọi E , F theo thứ tự là hình chiếu của H
trên AB, AC . Chứng minh rằng: Khi A, H không thay đổi còn B, C thay đổi thì
đường tròn ngoại tiếp tứ giác BCFE luôn đi qua 2 điểm cố định.
Lời giải
Theo giả thiết, trong các tam giác vuông AHB và
A
2 2
F AHC ta có: AE. AB  AH và AF . AC  AH .
P
Do đó: AE. AB  AF . AC . Suy ra tứ giác BECF nội
E tiếp.
B C
Gọi P, Q là giao điểm của AH với đường tròn
H
 BEFC  .
Do tứ giác BPCQ nội tiếp nên: HP.HQ  HB.HC .
Trong tam giác vuông ABC có: HB.HC  AH 2 .
Do đó HP.HQ  HB.HC  AH 2 .
Q
Mặt khác: AP. AQ  AE. AB  AH 2 .
Suy ra

( AH  HP )( AH  HQ )  AH 2

 AH 2  AH ( HQ  HP )  HP.HQ  AH 2
 AH  HQ  HP.
 HP  HQ  AH
Do đó:  2
.
 HP.HQ  AH
 5 1
 HP  AH
 2
Giải hệ này ta được:   P, Q cố định.
 HQ  5  1 AH
 2
Vậy đường tròn  BEFC  luôn đi qua 2 điểm cố định P, Q .

Trang 21
VI. Ứng dụng 6: Tính toán các đại lượng hình học
6.1. [USA 1998]. Cho hai đường tròn đồng tâm O ký hiệu là C1  và C2  ( C2  nằm
trong C1  ). Từ một điểm A nằm trên C1  kẻ tiếp tuyến AB đến C2  . AB cắt C2 
lần thứ hai tại C . D là trung điểm của AB . Một đường thẳng qua A cắt O2  tại
E , F sao cho đường trung trực của DF và EC giao nhau tại điểm M nằm trên AC .
AM
Tính tỉ số .
MC
Lời giải
Rõ ràng B là trung điểm AC .
1
Ta có PA/ C2   AE. AF  AB 2  AB.2 AB  AD. AC .
A D B M C 2
Suy ra tứ giác DCFE nội tiếp đường tròn. Do đó M là
E tâm đường tròn ngoại tiếp tứ giác DCFE .
F
1
O Mà M nằm trên AC nên MD  MC  DC .
2
 5
 AM  AB
 4 AM 3
Điều đó giúp ta tính được    .
 3 MC 5
MC  AB
 4
6.2. [Russian MO 2008]. Cho  ABC nội tiếp đường tròn O; R  , ngoại tiếp đường
tròn  I ; r  . Đường tròn  I  tiếp xác với AB, AC lần lượt tại X , Y . Gọi K là điểm
chính giữa cung AB không chứa C . Giả sử XY chia đôi đoạn AK . Tính BAC ?
Lời giải
K S A
X
Y Gọi S  XY  AK .
I   KCB
Ta có KAB ;
B C   1800  
SXA   AXS CIB
AXY  BIC
AS AX AX .CB
O    AS 
CB CI CI
1 AX .BC
 AK  hay KI .CI  2 AX .BC .
2 CI
Đặt
a bc
BC  a, CA  b, AB  c, S  S ABC , p 
2
abc S
Ta có 2 Rr  2  p  a  a  .   p  a  a  bc  4 p  p  a   b 2  c 2  bc  a 2
4S p

Trang 22
Mặt khác, theo định lý hàm côsin ta có a 2  b 2  c 2  2bc cos A .
1
Khi đó b 2  c 2  bc  b 2  c 2  2bc cos A  cos A    A  1200 .
2

Vậy BAC  120 . 0

6.3. [Russian MO 2007]. Cho hai đường tròn O1  và O2  cắt nhau tại A, B . Gọi
PQ, RS là hai tiếp tuyến chung của hai đường tròn  P, R  O1 ; Q, S  O2  . Giả sử
RB
RB // PQ , RB cắt O2  lần nữa tại W . Tính tỉ số ?
BW
Lời giải
Q Gọi J  AB  PQ . Suy ra J là trung
J
P điểm của đoạn thẳng PQ .
Rõ ràng AB // PR ; PQ // RB nên tứ giác
A
PJBR là hình bình hành
O2 W  PJ  RB
O1  RB.RW  RS 2  PQ 2  4 PJ 2  4 RB 2
B  RW  4 RB .
RB 1
R Vậy  .
RW 3
S
VII. Ứng dụng 7: Chứng minh một số mệnh đề về góc của tam giác và tìm quỹ tích
điểm.
7.1. [Belarus 2000, problem 1]. Cho tứ giác ABCD có hai đường chéo cắt nhau ở
M . Đường phân giác của  ACD cắt tia BA ở K . Chứng minh rằng:
nếu MA.MC  MA.CD  MB.MD thì BKC   CDB.
Lời giải

A B
K Gọi N  CK  BD . Áp dụng định lý về đường phân giác cho
M MCD ta có:
CD MC MC.DN
N   CD  .
D DN MN MN
MC.DN
Khi đó: MB.MD  MA.MC  MA.
MN
C
 DN  MD
  MA.MC   1     MA.MC  .
 MN  MN
hay MA.MC  MB.MN .

Trang 23
Vì M nằm trong tứ giác ABCN , theo định lý về phương tích thì A, B, C , N cùng
nằm trên đường tròn.

Từ đó KBD ABN     KCD
ACN  NCD  . Suy ra K , B, C , D cùng nằm trên một
  CDB
đường tròn. Do đó ta có BKC .

7.2.[Belarus 2000, problem 2]. Cho  ABC và C   . Gọi M là trung điểm của
2
cạnh huyền AB , H là chân đường cao CH và P là điểm nằm trong tam giác sao cho
 khi và chi  
AP  AC . Chứng minh rằng PM là phân giác .BPH A .
3
Lời giải
Điểm P nằm trên đường tròn
C tâm A bán kính AC .
Đường tròn  cắt các đường
P thẳng CH , MH và PH tương
ứng tại D, N và Q .
H M N Vì MA  MC ,  A  600 khi và
B
A chỉ khi  ACM đều. Nghĩa là khi
và chỉ khi M  N .
Điều đó khẳng định PM là phân
 khi và chỉ khi
giác BPH
D M N.
Thật vậy, AH là đường cao
Q
thuộc đáy của tam giác cân
ACD nên H là trung điểm của CD và CD là một dây cung của đường tròn  . Theo
định lý về phương tích của một điểm ta có PH .HQ  CH .HD  CH 2 . Vì CH là
đường cao thuộc cạnh huyền của  ABC nên CH 2  AH .HB . Như vậy
PH .HQ  AH .HB .
Do H  AB  PQ nên tứ giác APBQ nội tiếp. Để ý trên đường tròn  rằng:
  QAN
QAB   2QPN  2 HPN
.
  QPB
Thế thì HPB   QAB   2 HPN.
 ). Do đó PM là phân giác HPB
Vì N  HB  PN ( PN là phân giác HPB  khi và chỉ
khi M  N .
7.3. [APMO 1995]. Cho tứ giác lồi cố định ABCD nội tiếp trong đường tròn mà AB
không song song với CD . Tìm quỹ tích các điểm P thỏa mãn điều kiện: ta có thể tìm
được hai đường tròn lần lượt nhận AB và CD làm dây cung sao cho chúng tiếp xúc
nhau tại P .

Trang 24
Lời giải

A Giả sử AB  CD  X .
Từ X kẻ một tiếp tuyến đến đường tròn
ngoại tiếp tứ giác ABCD và gọi R là độ
dài đoạn tiếp tuyến này. Như vậy quỹ tích
D
cần tìm là đường tròn tâm X , bán kính R .
B Thật vậy, ta gọi  C0  là đường tròn tâm X
C
bán kính R .
Lấy một điểm P bất kì trên  C0  . Xét
đường tròn ngoại tiếp tứ giác ABCD ta có:
R 2  XA. XB  XC. XD .
X
Suy ra XP 2  XA. XB  XC. XD .
Gọi  C1  là đường tròn đi qua ba điểm
C , D, P , ta có XC. XD  XP 2 nên XP là tiếp tuyến của đường tròn  C1  .
Tương tự gọi  C2  là đường tròn đi qua ba điểm A, B , P thì XP là tiếp tuyến của
đường tròn này.
Như vậy  C1  và  C2  tiếp xúc nhau tại P .
Ta chú ý rằng nếu P là một trong bốn giao điểm của AB hoặc CD với  C0  thì
không thể dựng các đường tròn như trên được, trừ khi ta chấp nhận rằng đó là đường
tròn suy biến thành đường thẳng.
Như vậy ta đã chứng minh được rằng tất cả các điểm nằm trên  C0  đều là các điểm
của quỹ tích (hoặc trừ đi bốn điểm nói trên nếu không chấp nhận trường hợp suy biến).
Bây giờ đảo lại, ta sẽ chứng minh rằng không có điểm nào của quỹ tích mà không nằm
trên  C0  .
Thật vậy, với bất kì đường tròn nào đi qua hai điểm C , D , chỉ có hai đường tròn qua
A, B và tiếp xúc với nó (điều này rõ ràng nếu ta xét cách mà đường tròn qua A, B thay
đổi khi tâm của nó di chuyển trên đường trung trực của AB ), do vậy có nhiều nhất là
hai điểm thuộc quỹ tích nằm trên đường tròn đi qua C , D cho trước.
Mà đây là hai giao điểm của đường tròn đó với  C0  , vậy không có điểm nào của quỹ
tích mà không nằm trên  C0  .

Trang 25
C. BÀI TẬP ĐỀ NGHỊ
 
1. Cho đường tròn O tiếp xúc đường thẳng d tại H . Hai điểm M , N di động trên d
sao cho HM .HN   k 2 ( k  0 cho trước ). Từ M , N kẻ tiếp tuyến MA và NB của
O 
(với A, B  H ). Chứng minh rằng:
a) Đường tròn  OMN  luôn đi qua 2 điểm cố định.
b) Đường thẳng AB luôn đi qua 1 điểm cố định.
 
2. Cho AB và AC là các tiếp tuyến của đường tròn O với B,C thuộc O . Lấy  
điểm M bất kì trên AC ( M , A khác phía so với C ). Giả sử (O) cắt đường tròn
(ABM) tại điểm thứ hai P, Q là chân đường vuông góc hạ từ C xuống MB . Chứng
  2
minh rằng: MPQ AMB .
3. Cho tam giác ABC có đường tròn tâm I nội tiếp, tiếp xúc các cạnh BC , CA, AB tại
D, E , F . AI cắt đường tròn  I  tại M , N ( M nằm giữa A và N ). DM cắt cạnh EF
tại K , NK cắt đường tròn  I  tại điểm P khác N . Chứng minh rằng các điểm
A, P, D thẳng hàng.
4. Cho tứ giác ABCD nội tiếp đường tròn tâm O bán kính R . Các đường thẳng AB
và CD cắt nhau tại P , đường thẳng AD và đường thẳng BC cắt nhau tại Q . Chứng
2

minh rằng: cos POQ   2R  .


 
 OP  OQ 
5. Cho tứ giác ABCD nội tiếp đường tròn  O; R  . Gọi P, Q, M lần lượt là giao điểm
của các cặp đường thẳng AB và DC , AD và BC , AC và BD . Chứng minh rằng bán
kính đường tròn ngoại tiếp các tam giác OPQ , OMP và OMQ bằng nhau.
6. Cho đường tròn tâm O đường kính AB . Một điểm H thuộc đoạn AB . Đường
thẳng qua H cắt đường tròn tại C . Đường tròn đường kính CH cắt AC , BC và
 O  lần lượt tại D, E và F .
a) Chứng minh rằng AB, DE và CF đồng quy.
b) Đường tròn  C ; CH  cắt  O  tại P và Q . Chứng minh rằng P, D, E , Q thẳng
hàng.
7. Cho tam giác ABC , D thuộc đoạn BC , E thuộc đoạn AD . Đường tròn ngoại tiếp
 BDE cắt AB tại K . Đường tròn ngoại tiếp CDE cắt AC tại L . Gọi M là giao
điểm của DK với BE , N là giao điểm của DL với CE , O là tâm đường tròn ngoại
tiếp tam giác EBC . Chứng minh rằng AO  MN .

===HẾT===

Trang 26
ĐƯỜNG THẲNG ĐI QUA ĐIỂM CỐ ĐỊNH TRONG
MỘT SỐ MÔ HÌNH QUEN THUỘC
ThS. Nguyễn Văn Hoàng - Trường THPT Chuyên Quang Trung - Bình Phước

Bài viết này sẽ phân tích việc dự đoán điểm cố định và chứng minh đường thẳng đi qua điểm
cố định thông qua một số kết quả hình học trong mô hình quen thuộc.

Mục lục
1 Tóm tắt nội dung 1

2 Mô hình đường đối trung 2

3 Mô hình hai đường đẳng giác 8

4 Mô hình trục đẳng phương, tâm đẳng phương 12

5 Mô hình đường tròn nội tiếp tam giác 17

6 Bài tập đề nghị 20

7 Tài liệu tham khảo 28

1 Tóm tắt nội dung


Chứng minh đường thẳng luôn đi qua một điểm cố định là một dạng toán thường gặp trong
các kì thi Olympic toán học: IMO, VMO, Olympic 30/4 ... Đây là một dạng toán chứng minh
đòi hỏi chúng ta phải có kĩ năng dự đoán điểm cố định, điều này yêu cầu phải có kinh nghiệm,
kiến thức tổng hợp và nhiều kĩ năng khác trong việc giải quyết một bài toán hình học phẳng.
Vì lẽ đó mà đây là dạng toán hay và làm nhiều học sinh bối rối khi đối diện, cụ thể là học sinh
không thể dự đoán được điểm cố định. Việc dự đoán điểm cố định có thể:

• thông qua kết quả hình học trong mô hình quen thuộc.

• thông qua vẽ hình chính xác.

• thông qua những suy luận và tính toán ban đầu (chẳng hạn cho điểm di động di chuyển
về các đầu mút, hoặc di chuyển về các điểm đặc biệt ...)

Việc dự đoán điểm cố định là nhân tố quan trọng để ta định hình công cụ và phương pháp giải
bài toán đó. Khi tạo ra bài toán đường đi qua điểm cố định người ta có thể dựa vào một số
kết quả hình học quen thuộc, sau đó giấu đi một số chi tiết, yêu cầu người làm toán phải phát
hiện ra kết quả hình học đó mới dự đoán được điểm cố định. Đặc biệt, nhờ sự "nhạy bén" hình
học mà ta có thể dự đoán được điểm cố định.

1
Vậy sự nhạy bén đó từ đâu? Đó là người làm toán có trong mình nhiều kết quả và mô hình
hình học. Bài viết này sẽ phân tích việc dự đoán điểm cố định và chứng minh đường thẳng đi
qua điểm cố định thông qua một số kết quả hình học trong mô hình quen thuộc. Tất
nhiên là không thể liệt kê hết các mô hình thường gặp nhưng tạo một thói quen trong việc dự
đoán điểm cố định và tạo "niềm tin" trong việc giải toán, từ đó phát triển kĩ năng làm toán
của học sinh.

2 Mô hình đường đối trung


Kết quả 1. Cho tam giác ABC nội tiếp đường tròn (O). Gọi D là điểm thuộc cạnh BC. Khi
đó các mệnh đề sau là tương đương:

1. AD là đường đối trung của tam giác ABC.


d(D; AB) AB
2. = .
d(D; AC) AC
DB AB 2
3. =( ) .
DC AC
4. AD đi qua giao điểm S của các tiếp tuyến kẻ từ B và C với (O).

5. (AEBC) = −1 với E là giao điểm của AD và (O).

6. ABEC là tứ giác điều hòa.


sin ∠DAB AB
7. = .
sin ∠DAC AC
Chứng minh. Các kết quả quen thuộc, bạn đọc tự chứng minh.

A
b

b b
D b
M b
B C

b
E

Ví dụ 1. Cho đường tròn (J) cố định, hai điểm B, C cố định và A di động trên (J) sao cho A
khác B và C. Gọi I là tâm đường tròn nội tiếp tam giác ABC và O là tâm đường tròn ngoại
tiếp tam giác BIC. Từ A kẻ hai tiếp tuyến AP và AQ đến (O). Gọi M, N là hình chiếu của
B, C trên AP, AQ tương ứng. Đường thẳng qua M song song AB và đường thẳng qua N song
song AC cắt nhau tại F . Chứng minh rằng AF luôn đi qua một điểm cố định.

2
Dự đoán điểm cố định: Ta thấy d(F, AB) = d(M, AB), d(F, AC) = d(N, AC), ta nghĩ đến
kết quả của đường đối trung, do đó ta sẽ chứng minh AF là đường đối trung của tam giác
ABC.

F
b

b
A

b
J
b
I
M b
b
Q
b
B

P
b

b N
b

O b

C
b

I Hướng dẫn giải.

• Ta có A, I, O thẳng hàng; O thuộc (J) và 4AP O = 4AQO, do đó AP, AQ là hai đường


đẳng giác của tam giác ABC ứng với góc A. Suy ra 4ABM ∼ 4ACN.

• Ta có:
d(F, AB) d(M, AB) AB
= =
d(F, AC) d(N, AC) AC
Suy ra AF là đường đối trung của tam giác ABC. Do đó AF đi qua điểm cố định là giao
điểm của hai tiếp tuyến tại B và C với (J).

Ví dụ 2 (VN TST 2015). Cho đường tròn (O), dây cung BC cố định và điểm A di động trên
(O). Gọi I, H lần lượt là trung điểm BC và trực tâm tam giác ABC. Tia IH cắt (O) tại K,
AH cắt BC tại D, KD cắt (O) tại M, từ M kẻ đường vuông góc BC cắt AI tại N. Đường
tròn tiếp xúc AK tại A và đi qua N cắt AB, AC tại P, Q, gọi J là trung điểm P Q. Chứng
minh rằng AJ luôn đi qua một điểm cố định.

Dự đoán điểm cố định: Đây là mô hình trực tâm quen thuộc, dễ dàng thấy AJ qua tâm O
qua vẽ hình chính xác, do đó ta liên tưởng chứng minh AJ và AH là hai đường đẳng giác, mà
AJ là đường trung tuyến nên ta nghĩ đến chứng minh AH là đường đối trung của tam giác
AP Q.

3
A
b

b
P
b
E
K J
b
b
R b

b
O
N b
Q
F b H
b

b b
D b b
I b
b

B C
T

I Hướng dẫn giải.


• Gọi E, F lần lượt là chân đường cao kẻ từ B, C. Ta có AK, EF, BC đồng quy tại T (kết
quả của tâm đẳng phương).

• Từ một kết quả quen thuộc trong mô hình trực tâm, ta có

A(T DBC) = −1 ⇒ A(AHP Q) = −1 ⇒ (ARP Q) = −1

với R là giao điểm thứ hai của AH và (AP Q). Suy ra AR là đường đối trung của tam
giác AP Q.

• Mặt khác AR và AO là hai đường đẳng giác góc A của tam giác ABC, suy ra AO là
đường trung tuyến của tam giác AP Q, do đó AJ đi qua O cố định.

Kết quả 2. Cho tam giác AEF . Một đường tròn qua E, F cắt các cạnh AE, AF lần lượt tại
B và C. Gọi AK, AT lần lượt là đường trung tuyến của tam giác AEF và ABC. Khi đó AK
là đường đối trung của tam giác ABC và AT là đường đối trung của tam giác AEF .
A b

b
C
b

b
T
B

b b b

E K F

Chứng minh. Ta có 4ABC ∼ 4AF E ⇒ ∠EAK = ∠CAT , nghĩa là AT và AK là hai đường


đẳng giác góc A. Mặt khác AT là đường trung tuyến của tam giác ABC nên AK là đường đối
trung của tam giác ABC.

4
Ở kết quả 2, đường thẳng BC được gọi là đường đối song của EF .

Ví dụ 3. Cho hai đường tròn (O1 ), (O2 ) cắt nhau tại hai điểm B và C, A là điểm thay đổi
trên (O1 ) sao cho A khác B và C. AB cắt (O2 ) tại điểm thứ hai E, AC cắt (O2 ) tại điểm thứ
hai F . Gọi MN là đường trung bình của tam giác ABC ứng với cạnh BC. Các đường thẳng
BN và CM cắt nhau tại P . Đường tròn ngoại tiếp các tam giác BMP và CNP cắt nhau tại
điểm thứ hai Q. Gọi K là trung điểm EF. Chứng minh ba điểm A, Q, K cùng thuộc một đường
thẳng d và d luôn đi qua một điểm cố định khi A di động trên (O1).

Dự đoán điểm cố định: Ta thấy BC là đường đối song của EF mà K là trung điểm EF
(như yêu cầu chứng minh) nên dễ dàng dự đoán AQ là đường đối trung của tam giác ABC, từ
đó suy ra điểm cố định.

E
b

M K
b
b
b b
b

O1 b
b
O2
P Q

A b
b
b
b
N C
F

Hướng dẫn giải.

• Phép vị tự quay tâm Q biến B thành M, biến N thành C nên 4BQM ∼ 4NQC.

• Gọi D là giao điểm của AQ và BC. Ta có

DB SABD AB.d(D, AB) AB.d(Q, AB) AB.d(Q, MB) AB.MB AB 2


= = = = = =
DC SADC AC.d(D, AC) AC.d(Q, AC) AC.d(Q, MC) AC.CN) AC 2

• Suy ra, AQ là đường đối trung của tam giác ABC, hay AQ đi qua điểm cố định là giao
điểm của hai tiếp tuyến tại B và C với (O1 ).

• Theo kết quả 2 ta có A, Q, K thẳng hàng.

5
Kết quả 3. Cho tam giác ABC nội tiếp đường tròn (O). Tiếp tuyến tại A của (O) cắt BC
tại E. Đường tròn (AOE) cắt (O) tại điểm thứ hai F khi đó AF là đường đối trung của tam
giác ABC.

A
b

b
O

b b
b
E B C

Chứng minh. Ta có EA và EF đều là tiếp tuyến của (O). Do đó theo kết quả cơ bản AF đi
qua giao điểm của hai tiếp tuyến tại B và C của (O) nên AF là đường đối trung của tam giác
ABC.

Ví dụ 4 (VMO 2014). Cho tam giác ABC nhọn nội tiếp (O), trong đó B, C cố định và A
thay đổi trên (O). Trên các tia AB và AC lần lượt lấy các điểm M, N sao cho MA = MC và
NA = NB. Các đường tròn ngoại tiếp các tam giác AMN và ABC cắt nhau tại P khác A.
Đường thẳng MN cắt đường thẳng BC tại Q.

a. Chứng minh rằng ba điểm A, P, Q thẳng hàng.

b. Gọi D là trung điểm của BC. Các đường tròn có tâm là M, N và cùng đi qua A cắt nhau
tại K (K khác A). Đường thẳng qua A vuông góc với AK cắt BC tại E. Đường tròn
ngoại tiếp tam giác ADE cắt (O) tại F (F khác A). Chứng minh rằng đường thẳng AF
đi qua một điểm cố định.

Dự đoán điểm cố định. Ví dụ này cũng như các ví dụ ở trên, một dấu hiệu giúp ta liên
tưởng đến mô hình đường đối trung là điểm A di động trên đường tròn, dấu hiệu nữa là giả
thiết MA = MC và NA = NB ta nghĩ đến các đường tròn giao nhau. Mặt khác, với giải thiết
đường thẳng qua A vuông góc với AK ta có thể nghĩ về đường tròn đường kính hoặc tiếp tuyến
với đường tròn. Kết nối các giả thiết cùng với kết quả 3, ta có lời giải sau đây.

6
A
b

b
b
N

E b b b b b
C
B D
Q

b
F K

I Hướng dẫn giải.


(a) Không mất tính tổng quát, ta giả sử AB < AC như hình vẽ, các trường hợp còn lại hoàn
toàn tương tự. Khi đó, M nằm ngoài đoạn AB và N nằm trong đoạn AC. Do NA = NB nên
∠NBA = ∠NAB và do MA = MC nên ∠MCA = ∠MAC. Từ đây suy ra ∠NBA = ∠MCA
hay tứ giác BMCN nội tiếp và ta được

QM.QN = QB.QC.

Từ đây suy ra Q có cùng phương tích với hai đường tròn (O) và (AMN) nên Q nằm trên trục
đẳng phương của hai đường tròn này. Trục đẳng phương đó chính là dây chung AP nên suy ra
A, P, Q thẳng hàng.
(b) Ta sẽ chứng minh rằng O thuộc (ADE). Thật vậy, ta có O, M cùng nằm trên trung trực
của AC nên OM ⊥ AC. Tương tự thì ON ⊥ AB nên O là trực tâm tam giác AMN. Suy ra
AO ⊥ MN.
Xét hai đường tròn (M, MA), (N, NA) thì do dây chung vuông góc với đường nối tâm, ta có
AK ⊥ MN. Từ đây suy ra A, O, K thẳng hàng nên ∠OAE = 900 . Do đó ∠OAE = ∠EDO =
900 . Suy ra O ∈ (ADE). Theo kết quả 3 AF đi qua giao điểm hai tiếp tuyến tại B, C của
(O), do đó AF đi qua điểm cố định. ♣

7
3 Mô hình hai đường đẳng giác
Kết quả 4. Cho tam giác ABC. Các cặp đường thẳng da , d0a là hai đường đẳng giác góc A.
Định nghĩa tương tự với db , d0b và dc , d0c . Khi đó da , db, dc đồng quy khi và chỉ khi d0a , d0b, d0c đồng
quy.

Chứng minh. Sử dụng định lý Ceva sin dạng lượng giác.

Chú ý.

• Khi đó hai điểm đồng quy được gọi là hai điểm đẳng giác liên hợp.

• Đặc biệt nếu da , db , dc là các đường trung tuyến thì d0a , d0b , d0c là các đường đối trung tương
ứng và chúng đồng quy tại một điểm gọi là điểm Lemoine.

Ví dụ 5. Cho tam giác ABC nhọn nội tiếp (O), trong đó B, C cố định và A thay đổi trên (O).
Kẻ đường kính AD, đường thẳng CD cắt AB tại E, BD cắt AC tại F . Đường thẳng đối xứng
BC qua đường phân giác góc ∠ACD cắt EO tại M, đường thẳng đối xứng BC qua đường
phân giác góc ∠ABD cắt F O tại N. Chứng minh MN luôn đi qua một điểm cố định.

Dự đoán điểm cố định. Bài toán cho các đường thẳng đối xứng qua đường phân giác là dấu
hiệu trực tiếp nghĩ đến hai đường đẳng giác và hai điểm đẳng giác liên hiệp.

b
A

b M
O b

b N

b
T
B b b

E b

b F

I Hướng dẫn giải.

• Gọi T là trung điểm BC. Gọi I là giao điểm của hai tiếp tuyến tại B, C của (O).

• Ta có 4EBC ∼ 4EDA. Mặt khác O và T là trung điểm AD, BC tương ứng. Suy ra
ET, EO đẳng giác góc ∠CEA =⇒ M, T là hai điểm đẳng giác liên hiệp đối với 4ACE.
Do đó AT, AM đẳng giác góc ∠BAC =⇒ AM là đường đối trung của 4ABC.

8
• Tương tự, ta cũng chứng minh được AN là đường đối trung của 4ABC, suy ra MN là
đường đối trung của tam giác 4ABC =⇒ MN đi qua điểm I cố định là giao điểm của
hai tiếp tuyến tại B, C của (O).

Kết quả 5. Cho tam giác ABC có (O) là tâm đường tròn ngoại tiếp và Oa là tâm đường tròn
(OBC). Gọi N là tâm đường tròn Euler của tam giác ABC. Khi đó AN và AOa đẳng giác góc
A.

A
b

b
H b
N O
b
b

M
B b b b b C

Oa

Chứng minh. • Gọi H, M lần lượt là trực tâm tam giác ABC và trung điểm BC. Gọi L là
giao điểm của OOa và AN.
Khi đó tứ giác AHLO là hình bình hành, do đó OL = AH = 2OM. Suy ra tam giác
OCL cân tại C, ta được ∠CLO = ∠COL = ∠OCOa .

• Từ đây suy ra:


4OCL ∼ 4OOaC ⇒ OC 2 = OL.OOa ⇒ OA2 = OL.OOa
⇒ 4OAL ∼ 4OOaA ⇒ ∠OAN = ∠OAL = ∠OOaA = HAOa .
Điều này chứng tỏ AN, AOa là hai đường đẳng giác góc ∠HAO. Mà AH và AO là hai
đường đẳng giác góc A nên AN, AOa là hai đường đẳng giác góc A.

Ví dụ 6. Cho tam giác ABC nhọn nội tiếp (O), trong đó B, C cố định và A thay đổi trên
(O). Gọi E, F đối xứng B, C qua AC và AB. Chứng minh rằng đường thẳng d qua A vuông
góc EF luôn đi qua một điểm cố định khi A di động.
Dự đoán điểm cố định. Có hai yếu tố giúp tìm ra điểm cố định. Một là vẽ hình chính xác.
Hai là nhận thấy d là đường cao của tam giác AEF . Khi đó ta liên tưởng đến mô hình hai
đường đẳng giác.
Phân tích. Dựng tâm K của đường tròn (EAF ) ⇒ d và AK đẳng giác góc ∠EAF .
Kéo dài AK thấy AK đi qua tâm đường tròn Euler N của tam giác ABC. Do đó d và AN
đẳng giác góc ∠EAF . Từ tính chất đối xứng ⇒ d và AN đẳng giác góc ∠BAC.

9
b
E

b
A

b
Z
b
F
Y O
b
b
G b

b
b

J
b
N
H

B b b b b C

Oa

I Hướng dẫn giải.

• Gọi N là tâm đường tròn Euler của tam giác ABC và Y, Z lần lượt là hình chiếu vuông
góc của N trên AB và AC. Khi đó AN đi qua tâm đường tròn ngoại tiếp tam giác AY Z.
Ta sẽ chứng minh d đẳng giác với AN, tức là chứng minh d ⊥ Y Z hay cần chứng minh
Y Z k EF .

• Thật vậy, gọi G là trọng tâm tam giác ABC, gọi M là trung điểm AB và gọi J là giao
điểm của CF và AB. Trước tiên ta có
GN 1
=
GH 4
Mặt khác
2HJ + CH HF NY 1
2NY = HJ + OM = = ⇒= = .
2 2 HF 4
GN 1
Mà NY k HF và = nên G, Y, F thẳng hàng và
GH 4
GY 1
= .
GF 4

GZ 1
• Tương tự ta có = . Do đó
GE 4
GY GZ 1
= = .
GF GE 4

Suy ra, Y Z k EF hay d đẳng giác với AN .

• Theo kết quả 4, suy ra d đi qua tâm Oa đường tròn (OBC), do đó d luôn đi qua một
điểm cố định.

10
Ví dụ 7. Cho tam giác ABC nhọn nội tiếp (O), trong đó B, C cố định và A thay đổi trên (O).
Gọi E, F đối xứng B, C qua AC và AB. Gọi K là tâm đường tròn (AEF ). Chứng minh rằng
đường thẳng AK luôn đi qua một điểm cố định khi A di động.

Dự đoán điểm cố định. Nhận thấy AK và đường thẳng d qua A vuông góc EF là hai đường
đẳng giác góc A của tam giác EAF . Theo ví dụ trên AN và d đẳng giác nên dự đoán K, A, N
thẳng hàng, do đó AK đi qua điểm đối xứng của O qua BC.

b
K

b
E
L
b

A
b

F
b

N b
b
O

B b b C

b
O0

Oa
b

I Hướng dẫn giải.

• Gọi L là hình chiếu của A trên EF . Khi đó theo ví dụ trên, AL và AN là hai đường đẳng
giác góc A.
Ta sẽ chứng minh K, A, N thẳng hàng tức là chứng minh AL và AK đẳng giác góc A.

• Thật vậy, ta có AK và AL là hai đường đẳng giác góc ∠EAF . Do đó

∠KAF = ∠LAE.

Gọi Ax, Ay là hai tia đối của tia AL và AK. Khi đó ∠F Ay = ∠EAx (bù với hai góc
bằng nhau).

• Mặt khác ∠EAB = ∠BAC = ∠CAF (tính chất đối xứng). Suy ra

∠CAy = ∠BAx

mà ∠BAx = ∠CAN ⇒ ∠CAy = ∠CAN ⇒ K, A, N.

• Vì AN qua O 0 đối xứng với O qua BC nên AK luôn đi qua điểm cố định O 0.

11
4 Mô hình trục đẳng phương, tâm đẳng phương
Kết quả 6. Tập hợp các điểm có cùng phương tích với hai đường tròn không đồng tâm là một
đường thẳng, gọi là trục đẳng phương của hai đường tròn.

Kết quả 7. Cho ba đường tròn không đồng tâm (O1 ), (O2 ), (O3). Khi đó 3 trục đẳng phương
của các cặp đường tròn hoặc trùng nhau hoặc song song hoặc đồng quy tại một điểm M và gọi
M là tâm đẳng phương của ba đường tròn trên.

Ví dụ 8 (THTT). Cho tam giác ABC cố định. Các điểm E, F lần lượt di chuyển trên các
đoạn CA, AB sao cho BF = CE, gọi D = BE ∩ CF . Gọi H, K theo thứ tự là trực tâm của
tam giác DEF và DBC. Chứng minh rằng HK luôn đi qua một điểm cố định.

Dự đoán điểm cố định: Ta nhớ lại kết quả quen thuộc sau: Cho tam giác ABC nội tiếp
đường tròn (O). Lấy M, N thuộc cạnh AB, AC tương ứng sao cho BM = CN. Khi đó đường
trung trực MN đi qua trung điểm cung BC chứa A. Bổ đề này quen thuộc nên không chứng
minh ở đây.
b
K

X b

A
b

b
F

E
b

M b b

b I N
D b

G b

b b

B C

I Hướng dẫn giải. Gọi (M), (N) là đường tròn tâm M đường kính BF và đường tròn tâm
N đường kính CE. Gọi G = HF ∩ BE, I = HE ∩ CF .

• Ta có PH/(M ) = HG.HF = HI.HE = PH/(N ) .

• Tương tự, ta cũng có PK/(M ) = PK/(N ) .

• Gọi X là trung điểm cung BC chứa A. Vì BM = CN nên theo kết quả quen thuộc ở
trên, đường trung trực MN qua X hay XM = XN và BM = CN do đó

PX/(M ) = PX/(N ) .

Do đó H, K, X thẳng hàng, hay HK luôn đi qua điểm cố định X.

12
Ví dụ 9 (HSG Trường THPT Chuyên KHTN Hà Nội năm 2011-2012). Tứ giác lồi ABCD
không có hai đường chéo vuông góc nội tiếp (O). Gọi P là điểm di chuyển trên cung AB không
chứa C, D. Gọi M = P D ∩ AC, N = P C ∩ BD. Gọi Q là giao điểm thứ hai của đường tròn
(AP M) và đường tròn (BP N). Chứng minh rằng P Q luôn đi qua một điểm cố định.

Dự đoán điểm cố định: Bằng hình vẽ chính xác ta cũng khó dự đoán điểm cố định, nhưng
để ý P Q là trục đẳng phương (dây cung chung) của hai đường tròn (AP M) và (BP N), do đó
điểm cố định sẽ có cùng phương tích của hai đường tròn này. Từ đó có thể nghĩ đến điểm cố
định là giao điểm của hai tiếp tuyến tại A và B của hai đường tròn.

b
B
P
b

b
b
Q
N

A
b M
b

b
L

I Hướng dẫn giải.

• Gọi Ax, By lần lượt là tiếp tuyến của (AP M) và (BP N) tại A và B. Khi đó

∠xAC = ∠AP D = ∠ACD = const ⇒ Ax cố định.

Tương tự, By cố định. Gọi L = Ax ∩ By. Suy ra L cố định.

• Tam giác ABL cân tại L . Thật vậy,

∠BAL = ∠BAC + ∠CAL.

∠ABL = ∠DBL + ∠ABD.


Mặt khác ∠BAC = ∠BP C = ∠DBL, ∠CAL = ∠AP D = ∠ABD. Do đó

∠BAL = ∠ABL.

• Tam giác ABL cân tại L nên LA2 = LB 2 . Do đó L thuộc trục đẳng phương của hai
đường tròn (AP M) và (BP N). Suy ra P Q đi qua L cố định.

13
Ví dụ 10 (Thầy Trần Quang Hùng tổng quát đề thi Mathley của thầy Nguyễn Minh Hà).
Cho tam giác ABC, đường tròn (K) thay đổi luôn tiếp xúc CA, AB tại E, F . Đường thẳng
BE và CF cắt nhau tại H, AH cắt BC tại D. Hai đường thẳng DE, DF lần lượt cắt AB, AC
tại M, N. Gọi L là trực tâm tam giác DMN. Chứng minh đường thẳng KL qua điểm cố định
khi đường tròn (K) di chuyển.

Dự đoán điểm cố định. Vẽ hình đúng nhận thấy LK qua tâm O đường tròn ngoại tiếp tam
giác ABC, tư tưởng chứng minh thẳng hàng bằng phương tích vì L là trực tâm DMN.

b
N
b

B b
b
b

L
J
b

F b
b

O1
A
b

H O2
b b b
K

D b

b b
O b
E b

I Hướng dẫn giải.

• Gọi (O1 ), (O2 ) lần lượt là tâm đường tròn đường kính EN, F M. Vì L là trực tâm tam
giác DMN nên:
PL/(O1 ) = PL/(O2 ) (1)

• Mặt khác:
PK/(O1 ) = KE 2 ; PK/(O2 ) = KF 2 ⇒ PK/(O1 ) = PK/(O2 ) (2).

• Gọi O là tâm đường tròn ngoại tiếp tam giác ABC. Kẻ OI ⊥ EN, OJ ⊥ F M. Ta có
−−→ −−→ −→ −→ −→ −→
PO/(O1 ) = OE.ON = (OI + IE)(OI + IN) = OI 2 + IE.IN

Mà (ACEN) = −1 ⇒ IE.IN = IA2 . Do đó PO/(O1 ) = OA2 .


Tương tự, PO/(O2 ) = OA2 ⇒ PO/(O1 ) = PO/(O2 ) (3).

• Từ (1), (2) và (3), suy ra K, L, O thẳng hàng. Do đó KL đi qua điểm (O) cố định.

14
Ví dụ 11 (VMO 2007). Cho hình thang ABCD có đáy lớn BC và nội tiếp (O). Gọi P là một
điểm thay đổi trên BC và nằm ngoài đoạn BC sao cho P A không là tiếp tuyến của (O). Đường
tròn đường kính P D cắt (O) tại E (E khác D). Gọi M là giao điểm của BC với DE, N là giao
điểm khác A của P A với (O). Chứng minh rằng đường thẳng MN đi qua một điểm cố định.

Dự đoán điểm cố định. Việc tìm ra điểm cố định A0 thật sự không khó. Có thể dự đoán thông
qua việc vẽ hình chính xác. Từ việc đã có sẵn hai đường tròn với dây cung chung là DE có đi
qua M, ta kết nối với dữ kiện điểm A cố định để xây dựng một đường tròn thứ ba rồi từ đó áp
dụng tính đồng quy của các trục đẳng phương để chứng minh thẳng hàng.

A D
b b

N
b

b
O

M C
b b b b b
P
B F

A0

IHướng dẫn giải.

• Gọi A0 là điểm đối xứng của A qua tâm O. Ta chứng minh N, M, A0 thẳng hàng, từ đó
suy ra MN đi qua A0 cố định.

• Thật vậy, ta có DE là trục đẳng phương của đường tròn (O) và đường tròn (T1 ) đường
kính P D. Vì ∠P NA0 = 900 nên NA là trục đẳng phương của đường tròn (O) và đường
tròn (T2 ) đường kính P A0.

• Giả sử DA cắt BC tại F , do ∠ADA0 = 900 ⇒ ∠P F A0 = 900. Do đó F là giao điểm thứ


hai của (T1 ) và (T2 ), suy ra F P là trục đẳng phương của (T1 ) và (T2 ).
Vì các trục đẳng phương đồng quy tại tâm đẳng phương, suy ra DE, F P và NA0 đồng
quy tại điểm M. Vậy M, N, A0 thẳng hàng.

15
5 Mô hình đường tròn nội tiếp tam giác
Kết quả 8. Đường tròn (I) nội tiếp tam giác ABC tiếp xúc với các cạnh AB, AC tại M, N
tương ứng. Đường thẳng BI cắt MN tại E. Khi đó ∠BEC = 900 .

A
b

N b
E
b

b
F
M b I
b

b b

B C

Chứng minh. Ta xét trường hợp E nằm ngoài đoạn MN, trường hợp còn lại chứng minh tương
tự. Trước tiên ta có
∠A
∠ENC = ∠ANM = 900 − .
2
Mặt khác
∠B ∠C ∠A
∠EIC = ∠IBC + ∠ICB = + = 900 − ,
2 2 2
suy ra ∠ENC = ∠EIC hay tứ giác INEC nội tiếp. Do đó ∠BEC = ∠INC = 900 .

Chú ý. Nếu CI cắt MN tại F thì tương tự ta cũng chứng minh được ∠BF C = 900 , tức là
B, F, E, C đồng viên.

Ví dụ 12. Cho góc ∠xBy = ϕ không đổi. Điểm A cố định trên tia Bx. Điểm C di động trên
tia By. Đường tròn (I) nội tiếp tam giác ABC tiếp xúc với các cạnh AC, BC tại E, F tương
ứng. Chứng minh rằng đường thẳng EF luôn đi qua một điểm cố định.

A
b

b
F

b
M
I
b

b b b

B C
E

I Hướng dẫn giải. Gọi M là giao điểm của EF và BI.


Khi đó, theo kết quả 8 ∠AMB = 900 .
ϕ
Mặt khác ∠ABM = không đổi và AB cố định nên BI cố định, do đó M cố định.
2
Điều này suy ra EF đi qua điểm cố định M. ♣

16
Ví dụ 13. Cho tam giác ABC, gọi D thay đổi trên BC sao cho C nằm giữa B và D. Đường
tròn nội tiếp các tam giác ABD và ACD cắt nhau tại P và Q. Chứng minh rằng P Q luôn đi
qua một điểm cố định.

Dự đoán điểm cố định. Mới nhìn thì ta chưa đoán được ngay điểm cố định, tuy nhiên, liên
tưởng đến kết quả trên, ta vẽ thêm các đường phụ MN, XY và các giao điểm EF thì thấy ngay
điểm cố định là trung điểm EF .
A
b

b
N

E K F
b b b

b
b

P
b Y
O
O0 b

Q b

b b b b b b
D

B M L C X

I Hướng dẫn giải.

• Gọi (O) là đường tròn nội tiếp tam giác ABD và (O) tiếp xúc BD và AD lần lượt
tại M, N. Gọi (O 0) là đường tròn nội tiếp tam giác ACD và (O 0) tiếp xúc các cạnh
AC, AD lần lượt tại X, Y . BO cắt MN tại E, CO 0 cắt XY tại F . Ta sẽ chứng minh
P Q qua trung điểm EF .

• Thật vậy, ta có các góc ∠ABC, ∠ACD không đổi nên theo ví dụ trên E, F cố định.

• Vì MN, P Q, XY đều vuông góc OO 0 nên MN k P Q k XY .

• Gọi L, K lần lượt là giao điểm của P Q với BD, EF . Khi đó L là trung điểm MX. Do đó
P Q là đường trung bình của hình thang EF XM. Do đó P Q qua trung điểm của EF .

Ví dụ 14 (VMO 2013 - Bài 3). Cho tam giác không cân ABC. Kí hiệu (I) là đường tròn tâm I
nội tiếp tam giác ABC và D, E, F lần lượt là các tiếp điểm của (I) với các cạnh BC, CA, AB.
Đường thẳng đi qua E và vuông góc BI cắt (I) tại K, (K 6= E). Đường thẳng đi qua F và
vuông góc CI cắt (I) tại KL, (L 6= F ). Gọi J là trung điểm KL.

a) Chứng minh rằng D, I, J thẳng hàng.


AB
b) Giả sử các đỉnh B và C cố định, đỉnh A thay đổi sao cho tỉ số = k và k không đổi.
AC
Gọi M, N lần lượt là các giao điểm của IE, IF với (I) (M 6= E, N 6= F ). MN cắt IB, IC
lần lượt tại P, Q. Chứng minh rằng đường trung trực của P Q luôn đi qua một điểm cố
định.

17
Y
b

b
A

F b

E
b
b X
b

I
b
b
P
M
N
b C
b b b b b b

B W A0 D R
Q
b

Phân tích.
Câu a) Tương đối đơn giản và đa số thí sinh có thể làm được ý này.
Câu b) Nhìn vào mô hình bài toán ta liên tưởng đến bài hình VMO 2009. Vì đây là mô hình
quen thuộc nên ta dễ dàng chứng minh một số kết quả đã có. Do đó nếu chịu khó phân tích
mô hình, chúng ta có thể khai thác được nhiều tính chất hình học từ đó đưa đến kết quả cần
chứng minh. Tuy nhiên, cái mới và cái khó của bài toán ở chỗ chứng minh đường thẳng đi qua
một điểm cố định bằng cách nào, khi mà giả thiết cho B, C cố định và A thay đổi sao cho tỉ số
AB
= k và k không đổi. Khai thác giả thiết này nhiều HS nghĩ đến đường tròn Apollonious,
AC
nghĩ đến đây là vì chưa dự đoán được điểm cố định mà P Q đi qua. Giả thiết này ta cũng có
thể liên tưởng đến chân đường phân giác trong góc A cố định và phải chăng điểm cố định có
liên quan đến điểm này. Đến đây ta phải giải quyết khâu đầu tiên là tìm điểm cố định.
AB
Để ý rằng điểm A thỏa mãn = k thì điểm A0 đối xứng với A qua BC cũng có tính chất
AC
này. Khi đó ta sẽ có mô hình ngược lại với đường thẳng P 0 Q0 đối xứng P Q qua BC và hai
đường trung trực của P Q và P 0 Q0 cắt nhau tại một điểm trên BC và đó là điểm cố định cần
tìm.
Cùng với nhận định chân đường phân giác trong góc A cố định, ta rút ra điểm cố định R đối
xứng với trung điểm W của BC qua chân đường phân giác trong góc A.
Sau khi dự đoán xong, điều khó nhất là dùng công cụ gì để chứng minh, điều này đòi hỏi HS
nắm vững các công cụ để chứng minh một đường thẳng đi qua một điểm cố định và sử dụng
phép biến hình trong trường hợp này. Một cách khác có thể dùng tính toán đại số và một số
biến đổi góc nhưng khá vất vả. Một điều khó khăn nữa khi không sử dụng phép biến hình là khó
vẽ hình chính xác vì các vị trí điểm khá gần nhau. Do đó, lựa chọn phép biến hình là phương
án tối ưu và khôn ngoan nhất.
I Hướng dẫn giải. (a) Vì (I) tiếp xúc các cạnh BC, CA lần lượt tại D, E nên DE ⊥ CI,
mà F L ⊥ CI (gt), do đó DE k F L. Suy ra DEF L là hình thang cân. Tương tự ta cũng có
DF EK là hình thang cân, suy ra

DK = EF = DL.

Do đó D nằm trên đường trung trực của KL. Vì I và J cũng nằm trên đường trung trực của
KL nên D, I, J thẳng hàng.
(b) Gọi W là trung điểm BC, gọi A0 là chân đường phân giác trong góc A, gọi R là điểm đối

18
xứng W qua A0 . Ta sẽ chứng minh đường trung trực của P Q luôn đi qua R.

• Trước tiên, gọi X, Y lần lượt là giao điểm của EF với BI, CI, Khi đó theo kết quả 8
B, C, X, Y đồng viên.

• Vì B, C, X, Y cùng nằm trên đường tròn đường kính BC nên W là tâm, do đó W nằm
trên đường trung trực của XY (1).

• Dễ thấy rằng I là trung điểm ME và cũng là trung điểm NF do đó EF k MN, hay


XY k P Q ⇒ P Q ⊥ AI.
Từ đó ∆IEX = ∆IMP nên X và P đối xứng nhau qua I, tương tự ta cũng có Y và Q
đối xứng nhau qua I. Suy ra hai đường trung trực của XY và P Q đối xứng nhau qua
AI. Kết hợp (1) ta có đường trung trực của P Q qua R (2).

• Vì B, C cố định nên W cố định. Theo giả thiết ta có


A0 B AB
= = k = const.
AC
0 AC
nên A0 cố định. Do đó R cố định. Kết hợp (2) ta có điều phải chứng minh.

6 Bài tập đề nghị


BÀI 15. Cho tam giác ABC nhọn nội tiếp (O), trong đó B, C cố định và A thay đổi trên (O).
Dựng hai hình vuông ABDE và ACGF bên ngoài tam giác ABC. Đường thẳng AF cắt BD
tại M, AE cắt CG tại N. Các đường tròn (DMF ) và (GNE) cắt nhau tại P và Q. Chứng
minh rằng P Q luôn đi qua một điểm cố định khi A di động trên (O).

Q
b

F
b

E b b
A G
b b

N
P b
b

b
b
D b b
M B C

b
I

19
Phân tích. Điều thú vị của bài toán này là có thể sử dụng một trong các kết quả của đường đối
trung. Hơn nữa, ta thấy AQ là đường kính của (AEF ) nên liên tưởng đến mô hình hai đường
đẳng giác và phép quay khi mà mô hình quá quen thuộc (kẻ thêm hình vuông bên ngoài tam
giác). Do đó ta có hai cách trình bày lời giải sau đây.
I Hướng dẫn giải 1. Cần chứng minh AQ là đường đối trung của tam giác ABC.

• Gọi Q∗ = DE∩GF , khi đó ∠Q∗ F A = ∠QDM nên Q∗ ∈ (DMF ). Tương tự Q∗ ∈ (GNE).


Suy ra Q∗ ≡ Q.

AB AM
• Ta có 4ABM ∼ 4ACN ⇒ = ⇒ AE.AN = AM.AC (AB = AE) ⇒ A thuộc
AC AN
trục đẳng phương của (DMF ) và GNE, do đó Q, A, P thẳng hàng.

d(Q, AB) AE AB
• Hơn nữa, = = . Suy ra AQ là đường đối trung của tam giác ABC hay
d(Q, AC) AF AC
P Q là đường đối trung của tam giác ABC.

Q
b

F
b

b
F0

H b

E b
A b
G
b

H0
b
b

N
b
b

b
b P
D b b
K b
M B C

b
I

I Hướng dẫn giải 2.

• Gọi Q∗ = DE∩GF , khi đó ∠Q∗ F A = ∠QDM nên Q∗ ∈ (DMF ). Tương tự Q∗ ∈ (GNE).


Suy ra Q∗ ≡ Q.

AB AM
• Ta có 4ABM ∼ 4ACN ⇒ = ⇒ AE.AN = AM.AC (AB = AE) ⇒ A thuộc
AC AN
trục đẳng phương của (DMF ) và GNE, do đó Q, A, P thẳng hàng.

• Gọi K là trung điểm BC. Gọi H = AK ∩ EF . Xét phép quay tâm A góc quay 900 biến:
E 7→ B, F 7→ F 0 , H 7→ H 0 ⇒ AK ⊥ AH 0 mà AK k BF 0 nên AH 0 ⊥ BF 0 ⇒ AH ⊥ EF .

20
• Vì AQ là đường kính của AEF nên AQ và AK đẳng giác góc ∠EAF hay AQ và AK
đẳng giác góc ∠BAC. Do đó AQ là đường đối trung của tam giác ABC.

BÀI 16. Cho hai đường tròn (O) và (O 0) cắt nhau tại hai điểm A và B. Trên tia AB lấy một
điểm M cố định. Qua M vẽ hai đường thẳng bất kì cắt (O) tại E và C; cắt (O 0 ) tại F và D.
Chứng minh rằng đường thẳng nối hai tâm ngoại tiếp của hai tam giác ACD và BEF luôn đi
qua một điểm cố định.

M
b

F b
E
b

A
b

O0
b
b
P

O
b
D
b
Q
B b
b
C

I Hướng dẫn giải. Ta có AB là trục đẳng phương hai đường tròn (O) và (O 0 ), M nằm trên
trục đẳng phương của hai đường tròn trên nên

ME.MC = MA.MB = MF.MD = k.

Khi đó, phép nghịch đảo f tâm M tỉ số k biến tam giác EBF thành tam giác CAD. Do đó
phép nghịch đảo này biến đường tròn (EBF ) thành đường tròn (CAD). Suy ra đường thẳng
nối hai tâm ngoại tiếp của hai tam giác ACD và BEF đi qua M cố định. ♣

BÀI 17. Cho tam giác ABC là tam giác nhọn không cân nội tiếp trong đường tròn tâm O.
Một đường thẳng d thay đổi sao cho d luôn vuông góc với OA và luôn cắt các tia AB và AC
lần lượt tại M và N. Giả sử các đường thẳng BN và CM cắt nhau tại K.

1. Giả sử AK cắt BC tại P . Chứng minh rằng đường tròn ngoại tiếp tam giác MNP luôn
đi qua một điểm cố định.

2. Gọi H là trực tâm tam giác AMN. Chứng minh rằng HK luôn đi qua một điểm cố định.

21
A
b

b N
H b

M
b

b K b O

b
I

S b b b b

B P C

I Phân tích: ở bài toán này có hai hình ảnh quen thuộc là MN vuông góc với OA ta sẽ có
BCNM nội tiếp và CM, BN, AP đồng quy sẽ có hàng điểm điều hòa. Sử dụng phương tích và
hệ thức Maclaurin sẽ giải quyết ý 1. Còn ý 2 chỉ là tính chất cơ bản của tứ giác toàn phần.
Hướng dẫn giải.
1) Kéo dài MN cắt BC tại S. Gọi Q là trung điểm của BC. Kẻ tiếp tuyến Ax của (O) tại A.
Ta có: Ax song song với MN. Suy ra tứ giác MNCB nội tiếp. Do đó

SB.SC = SM.SN.

Ta có CM, BN, AP đồng quy, suy ra(S, P, B, C) = −1, mà Q là trung điểm BC, theo hệ thức
Maclaurin ta có
SB.SC = SP.SQ.
Do đó
SP.SQ = SM.SN.
Vậy tứ giác MNQP nội tiếp, suy ra đường tròn ngoại tiếp tam giác NMP luôn đi qua trung
điểm cố định của BC.

2) Gọi I là trực tâm của tam giác ABC. Ta có HI chính là đường thẳng Steiner của tứ giác
toàn phần MNCBAS. Theo tính chất của đường thẳng Steiner trong tứ giác toàn phần ta có
HI sẽ đi qua điểm K là giao điểm của CM và BN. ♣

BÀI 18. Cho tam giác ABC nhọn nội tiếp (O), trong đó A cố định và B, C thay đổi trên (O)
sao cho BC luôn song song với một đường thẳng cố định. Các tiếp tuyến của (O) tại B và C
cắt nhau tại K. Gọi M là trung điểm BC, N là giao điểm của AN và đường tròn (O). Chứng
minh rằng đường thẳng KN luôn đi qua một điểm cố định.

22
A
b b
I

b
O

b b
M b
B C

I Hướng dẫn giải. Gọi giao điểm thứ hai của (O) và KN là I.
Tứ giác IBNC điều hòa nên (AI, AN, AC, AB) = −1. Mà M là trung điểm BC nên AI k BC.
Suy ra I cố định. Vậy KN luôn qua I cố định. ♣

BÀI 19. Cho hai đường tròn (O) và (O 0) cắt nhau tại A, B. Đường thẳng d quay quanh B cắt
(O) và (O 0) tại C, D tương ứng. Gọi M là trung điểm CD. AM lại cắt (O 0) tại P . Đường thẳng
qua M vuông góc OM cắt AC tại Q. Chứng minh rằng P Q luôn đi qua một điểm cố định.

Q
b

b S

A
b

O2
O1 b
K b
b

C b
b

B M b

N b
D

I Hướng dẫn giải.

• Gọi K, N lần lượt là giao điểm của (O1 ) và AP , (ACP ) và d. Gọi S là giao điểm thứ hai
của P Q và (O2 ). Ta sẽ chứng minh S cố định.

23
• Ta chứng minh được phép đối xứng tâm ĐM biến A, B, C theo thứ tự thành P, N, D. Do
đó phép đối xứng tâm ĐM biến (O1 ) thành (P ND). Kết hợp với MQ ⊥ O1 M suy ra MQ
là trục đẳng phương của (O1) và (P ND).

• Mặt khác, AQ và NP là trục đẳng phương của (O1 ), (ACP ) và (ACP ) và (P ND). Do
đó Q thuộc NP .

• Do đó
(BS, BA) ≡ (P S, P A) ≡ (P N, P A) ≡ (BK, AK)
Suy ra BS tiếp xúc (O1 ) tại B. Do đó S cố định.

BÀI 20. Cho (O) là một đường tròn cố định và A, B là hai điểm cố định trên (O) sao cho
A, B, O không thẳng hàng. Điểm C di động trên (O) (C khác A, B). Gọi (O1 ), (O2) qua A, B
và tiếp xúc BC, AC tại C tương ứng. (O1 ) cắt (O2 ) tại D (D 6= C). Đường thẳng AD và BD
cắt (O2 ), (O1 ) tại E và F (E, F 6= D). Chứng minh rằng đường thẳng qua C và vuông góc EF
luôn đi qua một điểm cố định.

b
E

Fb
C
b

O2
b

O1
b

b
O
D b

b b
B
A b X

Phân tích. Bài toán này lấy ý tưởng từ 2006 USA TST Problem 6.
I Hướng dẫn giải. Ta chứng minh đường thẳng qua C và vuông góc EF luôn đi qua một
điểm cố định là tâm đường tròn (AOB).

• Ta có

∠CBE = ∠CDE = ∠CF A = ∠ACB = ∠CEB = ∠CDF = ∠CAF

Do đó 4CAF ∼ 4CBE và ∠CAF = ∠CF A = ∠CBE = ∠CEB = ∠ACB. (∗)

• Vì ∠ADB = 360◦ − ∠CDA − ∠CDB = 2∠ACB =⇒ A, B, D, O đồng viên, gọi tâm


đường tròn này là X.

24
• Từ (∗) (Hai tam giác CAF và CBE cân tại C và đồng dạng) nên theo 2006 USA TST
P6 ta có CX ⊥ EF =⇒ đường thẳng qua C và vuông góc EF luôn đi qua một điểm cố
định X.

Bổ đề 1 (2006 USA TST). Cho tam giác ABC. Các tam giác P AB và QAC được dựng bên
ngoài tam giác ABC sao cho AP = AB, AQ = AC và ∠BAP = ∠CAQ. Các đoạn thẳng BQ
và CP cắt nhau tại R. Gọi O là tâm của đường tròn (BCR). Chứng minh rằng AO ⊥ P Q.
b

F
b

E b
N
b

b
A
M b

O2 b Q
O1 H b
b b

R C
b
b

b
O

Chứng minh. • Ta có 4AP C = 4ABQ nên ∠AP R = ∠ABR. Suy ra A, P, B, R cùng nằm
trên đường tròn, gọi tâm là O1 . Tương tự A, R, C, Q cùng thuộc đường tròn tâm O2 .

• Giả sử (O1), (O2 ) cắt AQ, AP tại E, F tương ứng. Gọi I là tâm của đường tròn (EAF )
Giả sử O1 I, O2 I cắt AE, AF lần lượt tại M, N.

• Ta có ∠AEP = ∠ARC = ∠AF Q nên 4AEP ∼ 4AF Q. Mà M, N là trung điểm AE, AF


nên 4P AM ∼ 4QAN. Suy ra ∠MP N = ∠MQN. Do đó P, M, N, Q đồng viên.

• Mặt khác, I, M, A, N đồng viên nên ∠AQP = ∠MNA = ∠MIA. Suy ra I, M, H, Q đồng
viên (IA cắt P Q tại H). Do đó IA⊥P Q.
Hơn nữa, P Q là trục đẳng phương của (I) và (O), suy ra IO⊥P Q. (vì P A.P F = P R.P C
và QA.QE = QB.QR) Điều này dẫn đến I, A, O thẳng hàng hay AO⊥P Q.

Thêm một kết quả cho bài toán này: Chứng minh CD luôn đi qua một điểm cố định khi
C di động trên (O).

25
b

F C b
b
O2

b
O1
b O
D

b b

A B
b
T

b
X

I Hướng dẫn giải.


Bổ đề 2. Cho tứ giác ABCD điều hòa. Một đường tròn qua A, B tiếp xúc BC cắt BD tại M.
Chứng minh MB = MD.
A
b

b
B b

M b D

Chứng minh. Vì BC là tiếp tuyến của (ABM) nên

∠MAB = ∠MBC = ∠DBC = ∠DAC

Suy ra, AC và AM đẳng giác góc A. Mặt khác ABCD là tứ giác điều hòa nên AC là đường
đối trung của tam giác ABD. Do đó AM là đường trung tuyến hay MB = MD.
Trở lại bài toán.
• Gọi X là giao điểm của tiếp tuyến tại A, B của (O), gọi T là giao điểm của CX và (O).
Khi đó CAT B là tứ giác điều hòa.

• Theo bổ đề trên D là trung điểm CT nên C, D, T thẳng hàng. Do đó CD luôn đi qua


một điểm cố định X.

26
7 Tài liệu tham khảo
1. Nguyễn Minh Hà, Đường thẳng và đường tròn luôn đi qua điểm cố định.

2. Trần Quang Hùng, Họ đường thẳng đường tròn đi qua điểm cố định.

3. Các diễn đàn toán học như Mathscope, Artofproblemsolving ...

27
CỰC, ĐỐI CỰC VÀ ỨNG DỤNG

Nguyễn Thái Vấn – Trường THPT Chuyên Lương văn Chánh

Trong các chuyên đề hình học bồi dưỡng học sinh giỏi, Cực và đối cực là một chuyên đề
thú vị, được dùng để chứng minh một số bài toán trong hình học phẳng như: các bài toán về
quan hệ vuông góc và quan hệ song song giữa hai đường thẳng; chứng minh ba đường thẳng
đồng qui, ba điểm thẳng hàng; Các bài toán về chứng minh đường thẳng đi qua điểm cố định,
điểm nằm trên đường thẳng cố định….. Việc dùng cực và đối cực có thể giúp định hướng tìm
lời giải nhanh hơn và ít phức tạp hơn. Trong khuôn khổ bài viết này chỉ đề cập đến cực và đối
cực đối với một đường tròn và ứng dụng của nó.
I. Đặt vấn đề
1. Định lý Pascal : Cho 6 điểm A, B, C, D, E, F cùng thuộc một đường tròn. Khi đó các giao
điểm của các cặp cạnh AB và DE, BC và EF, CD và FA thẳng hàng.
+ Xét 1 trường hợp đặc biệt khi AD, BE, CF đồng qui tại I. n ≥ 2
P

B
C
A
I

D
L
O

E
F

( Hình 1 )

1
( Hình 2 )

Bây giờ lấy G bất kỳ trên (O), gọi giao điểm của của tia GI và vòng tròn (O) là H. Gọi
AB I ED = {J } , BC I EF = {L} , CD I AF = {P} , AC I FD = {Q} , FH I CG = {M } , HA I DG = {N }

+ Theo định lý Pascal 6 điểm A, B, C, D, E, F và định lý Desargues cho hai tam giác ABC và
DEF, ta suy ra J, L, P, Q thẳng hàng.
+ Tương tự với 6 điểm A, H, F, D, G, C và hai tam giác AHF và DGC, ta cũng suy ra M, N, P,
Q thẳng hàng.
+ Từ trên , ta có thể mở rộng thêm ra rằng, giao điểm của các cặp đường thẳng tạo bởi 4 điểm
bất kỳ thuộc đường tròn sao cho tồn tại đường thẳng đi qua 2 điểm cũng đi qua I và đường
thẳng đi qua hai điểm còn lại cũng đi qua I thì cũng nằm trên đường thẳng tạo bởi các điểm
thẳng hàng .
Như vậy sự đồng qui của các đường thẳng tại I với sự thẳng hàng của các giao điểm có mối
quan hệ gì không ? Ta có kết quả sau:
2. Định lí : Cho 2n điểm thứ tự nằm trên vòng tròn (O) là A1, A2, ….., A2n, n ≥ 2 sao cho các
đường thẳng đi qua Ai và Ai+ n đồng qui tại một điểm I. Khi đó, các giao điểm của các cặp

đường thẳng ( Ai Ai +1 ; Ai + n Ai + n +1 ) thẳng hàng.


2
Chứng minh:
+ Định lí đúng với n=2
+ Định lý đúng với n= 3. Do định lý Pascan trong trường hợp đặc biệt .
Từ n ≥ 4 , mỗi lần tăng n lên 1 đơn vị, tức là thêm 2 điểm nữa, ta coi như là thêm hai
điểm khi có sẵn 6 điểm, tức là chỉ chứng minh cho trường hợp n = 4, các trường hợp khác là
hiển nhiên
+ n = 4. Ta có định lí đúng cho 6 điểm A, B, C, D, E, F trên đường tròn, lấy điểm G bất kỳ trên
(O), gọi giao điểm của tia GI và đường tròn (O) là H. Gọi
AB I ED = {J }, BC I EF = {L}, CD I AF = {P}, AC I FD = {Q}, FH I CG = {M }, HA I DG = {N },

+ Theo định lý Pascal 6 điểm và định lý Desargues cho hai tam giác ABC và DEF, ta suy ra J,
L, P,Q thẳng hàng.
+ Tương tự với 6 điểm A, H, F, D, G, C và hai tam giác AHF và DGC, ta cũng suy ra M, N, P,
Q thẳng hàng. Suy ra J, L, M, N thẳng hàng.
Vậy định lí đúng với n = 4, hay định lý đúng với n ≥ 2 .
Nhận xét: Từ kết quả trên ta thấy mối quan hệ giữa giao điểm I và đường thẳng chứa các
điểm thẳng hàng liên quan đến khái niệm được trình bày sau đây đó là cực và đối cực .
II. Khái niệm cực, đối cực
1. Định nghĩa 1: Ta nói hai đường tròn (O,R) và (O’, R’) là hai đường tròn trực giao nếu:
OO' 2 = R 2 + R' 2 .

( Hình 3 )

3
2. Định nghĩa 2: Ta nói hai điểm M, N liên hiệp với nhau qua đường tròn(O, R) nếu
đường tròn (O, R) và đường tròn đường kính MN là hai đường tròn trực giao.

( Hình 4 )

Mệnh đề 1: Cho đường tròn (O;R). Giả sử M, N là hai điểm không nằm trên đường tròn (O; R)
và khác O. Đường thẳng MN cắt đường tròn (O; R) tại 2 điểm A, B. Khi đó, hai điểm M, N liên
hợp với nhau đối với đường tròn (O; R) khi và chỉ khi (ABMN) = -1.
Chứng minh : Gọi O’ là trung điểm MN và R’ là bán kính của đường tròn đường kính MN.
Khi đó hai điểm M, N liên hợp với nhau đối với đường (O) khi và chỉ khi OO’2 = R2 + R’2

€ OO ' 2 - R 2 = R' 2 € PO ' /(O ) = R' 2 € O' A. O ' B = O ' M 2 = O' N 2 € ( ABMN ) = -1

Mệnh đề 2: Cho đường tròn (O; R) và một điểm M khác O. Tập hợp các điểm N sao cho M và
N liên hợp với nhau đối với đường tròn (O; R) là một đường thẳng vuông góc với OM (Hình 5)

Chứng minh:
+ Nếu N là điểm liên hợp của M đối với đường tròn (O) thì đường tròn đường kính MN trực
giao với đường tròn (O). Khi đó đường kính AB đi qua M của đường tròn (O) bị đường tròn
đường kính MN chia điều hòa với đường thẳng AB. Ta có (ABMH) = -1.
+ Trong hàng điểm điều hòa A, B, M, H thì H được xác định và MH ^ NH hay N nằm trên
đường thẳng d vuông góc với OM tại H. Ngược lại nếu N’ bất kì trên d thì đường tròn đường
đường kính MN’ đi qua H và do (ABMH) = -1 nên đường tròn đường kính MN’ trực giao với
đường tròn (O). Vậy N’ liên hợp với M đối với (O) .
4
( Hình 5 )

Định nghĩa 3 Cho đường tròn (O; R) và một điểm M khác O. Tập hợp các điểm N sao cho M
và N liên hợp với nhau đối với đường tròn (O; R) là một đường thẳng dM. Ta gọi dM là đường
đối cực của điểm M đối với đường tròn (O, R) và điểm M là cực của đường thẳng dM đối với
đường tròn (O; R). ( Hình 5)
Ta còn có thể định nghĩa cực đối cực bằng phép nghịch đảo như sau:
Định nghĩa 4: Trên mặt phẳng cho đường tròn (O,R) và một điểm M khác O. Phép nghịch đảo
cực O phương tích R2 biến M thành H. Gọi d là một đường thẳng qua H và vuông góc với OM.
Khi ấy ta gọi: d là đường đối cực của M đối với đường tròn (O) và M là cực của d đối với
đường tròn (O).

(Hình 6)
5
+ Từ định nghĩa 3, ta thu được 2 kết quả :
1) Với hai điểm S, P trên mặt phẳng mà P nằm trên đường đối cực của S đối với (O) và SP cắt
(O) ở M, N thì bốn điểm S, P, M, N lập thành 1 hàng điểm điều hòa.
2) Với hai điểm S, P trên mặt phẳng mà SP cắt (O) ở M, N thỏa mãn bốn điểm S, P, M, N lập
thành 1 hàng điểm điều hòa thì P nằm trên đường đối cực của S và S nằm trên đường đối cực
của P.
+ Các tính chất của cực, đối cực
Mệnh đề 3: OS vuông góc với đường đối cực của S.
Mệnh đề 4: Đối với một đường tròn cho trước, nếu đường đối cực của điểm A đi qua điểm B
thì đường đối cực của điểm B đi qua điểm A.(Định lí La Hire)
Chứng minh: Nếu điểm B nằm trên đường a của điểm A thì A và B là hai điểm liên hợp đối với
đường tròn cho trước. Mặt khác ta biết rằng tập hợp các điểm liên hiệp của điểm B là đường đối
cực b của điểm B đi qua điểm A.
Mệnh đề 5: Đối với một đường tròn cho trước, các đường đối cực của các điểm thẳng hàng thì
đồng quy và các cực của các đường thẳng đồng quy thì thẳng hàng .
Chứng minh:
Theo định lí 3, giả sử các điểm A1, A2, …, An nằm trên đường thẳng b nghĩa là các điểm Ai
thuộc b với i=1, 2,… n thì điểm B thuộc các đường thẳng ai (i=1, 2,…n)
trong đó điểm B là cực của đường thẳng b và ai là các đường đối cực của các điểm Ai.
Vậy các đường đối cực của các điểm Ai đều đồng qui tại điểm B. Ý sau chứng minh tương tự.
Mệnh đề 6: (Một số cách xác định đường đối cực thông dụng)
Trường hợp 1: Khi cực S ở ngoài đường tròn (O)

S O
S'

(Hình 7)
6
Trường hợp 2 : Khi cực S nằm trong đường tròn (O)

P O C
S
A
S

B F O
D B

( Hình 8 ) ( Hình 9 )

Trường hợp 3 : S nằm trên (O)

S
O

( Hình 10 )

7
III. Ứng dụng cực, đối cực
Bài toán 1: ( HSG quốc gia Việt Nam năm 2012)
Trong mặt phẳng, cho tứ giác lồi ABCD nội tiếp đường tròn tâm O và có các cặp cạnh
đối không song song. Gọi M,N tương ứng là giao điểm của các đường thẳng AB và CD, AD và
BC. Gọi P,Q,S,T tương ứng là giao điểm các đường phân giác trong của các cặp ∠MAN và
∠MBN,∠MBN và ∠MCN,∠MCN và ∠MDN,∠MDN và ∠MAN. Giả sử bốn điểm P,Q,S,T đôi
một phân biệt.
1) Chứng minh tứ giác PQTS nội tiếp. Gọi I là tâm của đường tròn đó.
2) Gọi E là giao điểm của các đường chéo AC và BD. Chứng minh rằng ba điểm E, O, I
thẳng hàng.

P
S
N

T
I

A Q
B

E M
O C

D
( Hình 11 )

Bình luận lời giải:


1
+ Ý1 : Ta chứng minh được –TPQ = –QST = 90 0 - (–C + –D ) nên tứ giác PQTS nội tiếp.
2
+ Ý 2: Ta cần chứng minh OE và OI cùng vuông góc với MN
- Ta chứng minh MN là trục đẳng phương của hai đường tròn (I) và (O)
- Còn đối với OE thì chứng minh E là trực tâm tam giác OMN hay dùng định lý Brocard.
Tuy nhiên nếu dùng định lý Brocard thì ta cần phải chứng minh lại đầy đủ nó. Điều này
làm cho việc chứng minh bài toán phức tạp hơn.

8
Còn nếu ta dùng cực, đối cực thì việc chứng minh OE vuông góc với MN khá đơn giản và nó
còn định hướng ta chỉ cần chứng minh thêm OI ^ MN thì bài toán hoàn tất. Mặt khác trong việc
chứng minh định lý Brocard, nếu dùng cực đối cực cũng rất đơn giản so với dùng trục đẳng
phương. Sau đây là lời giải chi tiết cho câu 2
2) Chứng minh rằng ba điểm E, O, I thẳng hàng
Ta có Q, T lần lượt là tâm đường tròn bàng tiếp của các tam giác BCM và ADM nên
chúng phải cùng nằm trên phân giác ngoài của góc –AMC hay M,Q,T thẳng hàng. Hơn nữa
–BCM –BAD
cũng do các tâm đường tròn bàng tiếp nên –MQB = 90 0 - = 90 0 - = –BAT
2 2
Hay tứ giác ABQT nội tiếp. Suy ra MA.MB = MQ.MT hay M có cùng phương tích đến hai đường
tròn (O), (I). Hoàn toàn tương tự đối với điểm N, từ đó suy ra MN là trục đẳng phương của hai
đường tròn (O), (I) nên OI ^ MN (1)
Mặt khác MN là đường đối cực của E đối với đường tròn (O) . Nên OE ^ MN (2)
Từ (1) và (2) suy ra ba điểm E, O, I thẳng hàng.
Bài toán phụ: ( Định lý Brokard )
Cho tứ giác lồi ABCD nội tiếp đường tròn tâm O, AD giao BC tại M, AB giao CD tại N,
AC giao BD tại I. Chứng minh rằng O là trực tâm của tam giác MIN.

( Hình 12 )

Chứng minh:
Cách 1:
Gọi H là giao thứ 2 của hai đường tròn ngoại tiếp các tam giác AID,BIC.

Xét tứ giác DOHC,ta có:


Từ đó suy ra tứ giác DOHC nội tiếp.Tương tự ta cũng suy ra tứ giác AOHB nội tiếp.
9
Dễ thấy suy ra N nằm trên trục đẳng phương của hai đường tròn
--> thẳng hàng.

Ta có: .
Từ đó suy ra , Tương tự ta có:
Suy ra O là trực tâm tam giác MIN (đpcm).
Cách 2:
+ Xét cực đối cực đối với đường tròn (O) + MI là đường đối cực của N, suy ra ON ^ MI
+ NI là đường đối cực của M, suy ra OM ^ NI
Vậy I là trực tâm tam giác IMN.
Bài toán 2: (HSG quốc gia Việt Nam bảng A năm học 2004-2005)
Trong mặt phẳng cho đường tròn (O) cố định bán kính R. Cho A, B là hai điểm cố định
nằm trên (O) sao cho ba điểm A, B, O không thẳng hàng. Xét một điểm C nằm trên đường tròn
(O), C không trùng với A và B. Dựng đường tròn (O1) đi qua A và tiếp xúc với đường thẳng
BC ở C; dựng đường tròn (O2) đi qua B và tiếp xúc với đường thẳng AC ở C.Hai đường tròn
này cắt nhau tại điểm thứ hai D khác C. Chứng minh rằng:
1) CD £ R

2) Đường thẳng CD luôn đi qua một điểm cố định , khi điểm C di động trên đường tròn (O) sao
cho C không trùng với A và B.((O) kí hiệu đường tròn tâm O)

( Hình 13 )

10
Ta thấy O1C ^ CB, OO 2 ^ CB  O1C // OO 2
Tương tự O2 C // OO1 Suy ra OO1CO2 là hình bình hành. Nên O1O2 đi qua trung điểm của OC.
Mà O1O2 đi qua trung điểm của CD nên O1O2 // OD . Lại vì O1O2 ^ CD nên
Từ đó sẽ có CD £ OC = R .
2) Chú ý rằng

(DA, DB) ∫ (DA, DC ) + (DC, DB) ∫ (O1 A, O1C ) + (O2C, O2 B ) = 2(CA, CB) ∫ (OA, OB) (modp )
2
+ Suy ra A, D, O, B đồng viên. Ta thấy OD, AB, tiếp tuyến tại C của (O) lần lượt là các trục
đẳng phương của từng cặp đường tròn (ADOB) và (COD), (O) và (ADOB) , (O) và (COD)
+ Do đó 3 đường nói trên đồng quy ở một điểm S. Xét cực và đối cực đối với (O).
+ Chú ý đường đối cực của S phải đi qua C và vuông góc với OS nên CD chính là đường đối
cực của S. Vì S thuộc AB cố định nên CD sẽ đi qua cực của AB là một điểm cố định.

Bài toán 3: (HSG quốc gia Việt Nam bảng A năm học 2002-2003)
Trong mặt phẳng cho hai đường tròn cố định (O1 ) , (O2 ) tiếp xúc nhau tại điểm M và bán
kính đường tròn (O2 ) lớn hơn bán kính đường tròn (O1 ) . Xét điểm A nằm trên đường tròn
(O2 ) sao cho ba điểm O1 , O2 , A không thẳng hàng .Từ A kẻ các tiếp tuyến AB và AC đến đường
tròn (O1 ) (B,C là tiếp điểm ). Các đường thẳng MB và MC cắt lại đường tròn (O2 ) tương ứng tại
E và F. Gọi D là giao điểm của đường thẳng EF và tiếp tuyến tại A của đường tròn (O2 ) . Chứng
minh rằng điểm D di động trên một đường thẳng cố định khi A di động trên đường tròn (O2 ) sao
cho ba điểm O1 , O2 , A không thẳng hàng .

A
B
F

M O1
O2

C
G

E H

11
(Hình 14 )

+ Có hai trường hợp là tiếp xúc trong hoặc ngoài với nhau. Ở đây sẽ giải khi chúng tiếp xúc
ngoài, khi tiếp xúc trong thì hoàn toàn tương tự

AM cắt lại (O1 ) ở G.Tiếp tuyến của (O1 ) tại G, M cắt nhau ở H.
+ Xét cực và đối cực đối với (O1 ) . Ta thấy đường đối cực của H là MG đi qua A nên đường đối
cực của A sẽ đi qua H, nói cách khác B, C, H thẳng hàng.
Trong phép vị tự tâm M biến O1 Æ O2 thì:

Suy ra: qua phép vị tự ấy.


Do đó : D, M, H thẳng hàng.
+ Lại chú ý HM là tiếp tuyến chung của nên D luôn thuộc một đường cố định là
tiếp tuyến chung của (O1 ) , (O2 ) .

Bài toán 4: ( Chuyển thể từ bài MEMO - 2010) Cho tam giác ABC có đường tròn nội tiếp là
(I). Tiếp điểm của (I) trên BC, CA, AB lần lượt là D, E, F. AD cắt lại (I) ở M. Đường thẳng
qua M vuông góc với AD cắt EF ở N. Chứng minh rằng AN//BC.
Giải

A S
N

P
M E
J
G
F I

B D
C
( Hình 15 )

12
Xét cực và đối cực đối với (I)
Gọi P là giao điểm thứ hai của MN với (I), dễ thấy D, P, I thẳng hàng EF cắt IP, IA lần lượt ở J,
G. Ta thấy suy ra M, G, I, D đồng viên. Do đó

Suy ra MGJP nội tiếp Từ đó có :


Chú ý rằng G là trung điểm của FE nên suy ra (NJEF)= -1 (Theo Maclaurine)
Hay N thuộc đường đối cực của J (theo hệ quả 2) (1)
+ Mặt khác đường đối cực của A là EF đi qua J nên đường đối cực của J đi qua A .
Từ (1) và (2) suy ra đường đối cực của J là AN ,
Nên IJ vuông góc với AN Mà IJ vuông góc với BC nên suy ra điều phải chứng minh.
Bài toán 5 ( MOP- 1995) Cho tứ giác ABCD ngoại tiếp (O). Tiếp điểm thuộc các cạnh AB,
BC, CD, DA lần lượt là M, N, P, Q. AN, AP cắt (O) tại E, F. Chứng minh rằng ME, QF, AC
đồng qui.

B
N
M C

P
E
J O
F

Q
D
( Hình 16 )

13
Giải :

Gọi K là cực của AC. Xét tứ giác nội tiếp MNPQ thì theo tính chất cực và đối cực của tứ
giác nội tiếp ta có MQ và NP cắt nhau tại K.

Lại xét đến tứ giác nội tiếp EFPN thì cũng có EF và NP cắt nhau tại K, suy ra MQ và EF
cắt nhau tại K.

Ta thấy ME và QF cắt nhau tại 1 điểm thuộc đường đối cực của K tức thuộc AC hay ME,
QF, AC đồng qui.

Bài toán 6 (MOP- 1997) Cho ABC là một tam giác và O là tâm đường tròn ngoại tiếp của nó.
Các đường thẳng AB và AC cắt lại đường tròn ngoại tiếp tam giác BOC ở B1 ,C1 tương ứng. Gọi
D là giao điểm của BC và B1C1 . Chứng minh rằng đường tròn tiếp xúc với AD tại A và có tâm
nằm trên B1C1 trực giao với đường tròn đường kính OD .

Giải :

I
O C1

B1
D B C

( Hình 17 )

Gọi ( I ) là đường tròn tiếp xúc với AD tại A và có tâm nằm trên B1C1
Xét cực và đối cực đối với ( I ) Ta thấy: ( AB1 , AC1 ) ∫
(OB, OC ) ∫ (C1 B, C1C ) (modp )
2 2
(1) Mà OA= OB (2)
Từ (1) và (2) suy ra : C1O ^ AB (3)
14
Tương tự : B1O ^ AC (4)
Từ (3) và (4) suy ra AO ^ B1C1
Từ đây sẽ dễ có O thuộc đường đối cực của D suy ra điều phải chứng minh.

Bài toán 7: Cho tam giác ABC với các đường cao BB', CC'. Gọi E, F lần lượt là trung điểm
của AC và AB. EF cắt B'C' ở K. Chứng minh rằng AK vuông góc với đường thẳng Euler của
tam giác ABC.

F E
K
I B'
C'
G

B C

( Hình 18 )

Ta xét cực và đối cực đối với đường tròn Euler của tam giác ABC ( kí hiệu là (S) với S là tâm )
+ Gọi I là giao điểm của FB' và EC', G là giao điểm của CF và BE, H là giao điểm của BB' và
CC'
+ Sử dụng định lí Pappus cho hai bộ 3 điểm (F,C',B) và (E,B',C) ta suy ra H, G, I thẳng hàng,
do đó SI chính là đường thằng Euler của tam giác ABC. (1)
+ Mặt khác, chú ý E, F, B', C' cùng nằm trên (S) thì suy ra AK chính là đường đối cực của I, suy
ra SI vuông góc với AK. (2)
Từ (1) và (2) suy ra điều cần chứng minh.

15
Bài toán 8: Cho tam giác ABC nội tiếp đường tròn (O, R). Các phân giác trong BE, CF cắt lại
(O) lần lượt ở M, N. Đường thẳng qua M vuông góc với BM cắt đường thẳng qua N vuông góc
với CN tại S. Chứng minh rằng SO vuông góc với EF.

A
G L
P
Q
H
K M

N
O
E
F
D
V
I

B C

( Hình 19 )

Nhận xét: Xét cực và đối cực với (O), ta sẽ xác định đường đối cực của S , rồi chứng minh nó
song song với EF
+ SN, SM cắt lại (O) lần lượt ở L, G, ta có C, O, G thẳng hàng; B, O, L thẳng hàng.
+ Tiếp tuyến của (O) tại G, N cắt nhau ở Q, Tiếp tuyến của (O) ở L, M cắt nhau ở P
OP cắt LM ở H , OQ cắt NG ở K.
+ Ta thấy đường đối cực của Q là GN đi qua S nên đường đối cực của S đi qua Q.
+ Tương tự có đường đối cực của S cũng đi qua P, Do đó đường đối cực của S là PQ.
Bây giờ ta cần chứng minh PQ // EF
Chú ý rằng IE // OP, IF // OQ thế nên để có PQ // EF ta chỉ cần chứng minh :
(FI, FE) ∫ (QO, QP) (mod2p )
Mặt khác nhận thấy : OK . OQ = OG 2 = OL2 = OH . OP
(
Từ đó suy ra Q, K, H, P đồng viên nên QO, QP ∫ HK, HO ) ( ) (mod 2p )
16
( ) (
Suy ra ta cần có (*) FI, FE ∫ HK, HO (mod2p ))
Kẻ ID, IV lần lượt vuông góc với AC, AB chú ý rằng :

ID Ê Cˆ
sin Á A + ˜
IE sin IED sin IFV Ë 2 ¯ sin NAC CM OK
= = = = = =
IF IV sin IED Ê B ˆ sin MAB BM OH
sinÁ A + ˜
sin IFV Ë 2¯

(Vì OK là đường trung bình của tam giác GNC, OH là đường trung bình của tam giác LBM)
( ) (
Lại có IE // OH, IF // OK nên IE, IF ∫ OH , OK (mod 2p ) )
Từ (1) và (2) suy ra tam giác IEF đồng dạng với tam giác OKH , Do đó (*) đúng .

Bài toán 9 : Cho tam giác ABC và D là một điểm trên cạnh BC thỏa –CAD = –ABC . Đường
tròn (O) đi qua B và D cắt AB, AD tại E, F; DE cắt BF tại G; M là trung điểm AG. Chứng
minh CM ^ AO.
Bổ đề: Cho tứ giác toàn phần ABCDEF. Qua F kẻ các đường thẳng song song với AB, CD,
AC, BD cắt CD, AB, BD, AC tại M, N, P, Q theo thứ tự. Khi đó M, N, P, Q thẳng hàng và
đường thẳng đi qua chúng song song với EG.

B
Q
C

A F
D
P

( Hình 20 )
17
Ta có nhận xét đơn giản sau: Cho hai tam giác ABC, DEF có các đường thẳng AD, BE,
CF đồng qui. Khi đó nếu AB//DE và BC//EF thì AC//DF.

Áp dụng nhận xét trên cho các cặp tam giác ( FQN, DGE), (FMQ, BEG), (FMP, AEG),
ta suy ra các đường thẳng NQ, MQ, MP cùng song song với EG, do đó M,N,P,Q thẳng hàng và
đường thẳng đi qua chúng song song với EG.

Trở lại bài toán : Qua A kẻ AH//BD, AK//BF, AI//DE( H thuộc EF, K thuộc ED, I thuộc
BF). Gọi J là giao điểm của EF, BC. Xét tứ giác toàn phần BEFDAJ, theo bổ đề trên, ta có
C,H,I,K thẳng hàng và đường thẳng đi qua chúng song song với JG. Vì CH là đường chéo của
hình bình hành AHJC nên CH đi qua trung điểm N của AJ. MN là đường trung bình của tam
giác AGJ nên MN//GJ. Suy ra C,N,M thẳng hàng và CM//GJ.

Mặt khác , JG là đường đối cực của A đối với (O), do đó OA vuông góc với JG. Suy ra
CM ^ AO.

A
H

E M

N
I

O
G
F

C
D J
B

( Hình 21 )

18
Bài toán 10: Cho tứ giác toàn phần ABCDEF, trong đó tứ giác ABCD có đường tròn nội tiếp
tâm I. Gọi A1, B1, C1, D1 là tiếp điểm của (I) với các cạnh AB, BC, CD, DA. Gọi M là hình
chiếu vuông góc của I lên EF. Hình chiếu của M lên các đường thẳng A1B1, B1C1, C1D1, D1A1
là M1, M2, M3, M4. Chứng minh rằng M1, M2, M3, M4.thẳng hàng.
Bổ đề 1: Cho tứ giác toàn phần ABCDEF, trong đó tứ giác ABCD nội tiếp trong (O). Gọi G là
giao điểm của AC và BD. M là hình chiếu của G lên EF. Khi đó O, M, G thẳng hàng và M là
điểm Miquel của tứ giác toàn phần.
Bổ đề 2: Cho tứ giác toàn phần ABCDEF và M là điểm Miquel của nó . Gọi X, Y, Z, T là hình
chiếu vuông góc của M lên AB, BC, CD, DA. Khi đó X, Y, Z, T thẳng hàng.
Gọi E1 là giao điểm của A1D1 và B1C1; F1 là giao điểm của A1B1 và C1D1. Áp dụng định
lý Brianchon cho lục giác DD1ABB1C, ta có AC , BD, B1D1 tương ứng là đường đối cực của E1,
F1, F, E. Do đó E,F, E1, F1 thẳng hàng. Xét tứ giác toàn phần A1B1C1D1E1F1 . Ta có
MI ^ EF  IM ^ E1 F1 và tứ giác A1B1C1D1 nội tiếp (I). Theo bổ đề 1, ta có M là điểm Miquel
của A1B1C1D1E1F1. Theo bổ đề 2, ta có M1, M2, M3, M4.thẳng hàng.

F1

E1
C
C1
D
B1

D1 X

F A A1 B

( Hình 22 )

∑ Cuối cùng với kiến thức cực đối cực đã trình bày trên thì định lý ban đầu trong đặt vấn đề
được chứng minh dễ dàng vì điểm I là cực còn các giao điểm P, Q, M, N, L, J , … nằm
trên đường đối cực của I nên chúng thẳng hàng.
19
BÀI TẬP TỰ LUYỆN

d ( A, b)
Bài 1: Cho hai điểm A,B cố định và (O;R) thay đổi sao cho = 2 , trong đó a, b theo thứ
d ( B, a )
tự là đường đối cực của A, B đối với (O). Xác định vị trí của O để SOAB lớn nhất.

Bài 2: Cho tam giác ABC cân tại A. Hai đường thẳng d1,d2 bất kì qua A. Các đường thẳng qua
B,C tương ứng vuông góc với d1,d2 cắt nhau tại D. Đường thẳng qua B vuông góc với AB cắt
d1 tại E.Đường thẳng qua C vuông góc với AC cắt d2 tại F. Chứng minh rằng AD vuông góc
với EF .
Bài 3: Cho tam giác ABC với (I) là đường tròn nội tiếp .Tiếp điểm của (I) trên BC, CA, AB lần
lượt là D, E, F.Gọi M, N, P lần lượt là điểm chung của các cặp đường thẳng (EF,BC) ,(DF,CA)
,(DE,AB). Chứng minh rằng M, N, P thẳng hàng .
Bài 4: Cho tam giác ABC, đường tròn nội tiếp tiếp xúc với BC, CA, AB lần lượt tại D, E, F.
Đường tròn nội tiếp tam giác DEF tiếp xúc với EF, FD, DE lần lượt tại M, P , N. Chứng minh
rằng AM, BP, CN đồng quy.
Bài 5: Gọi M,N,P là các giao điểm của đường tròn nội tiếp tam giác ABC với các cạnh
AB,BC,CA tương ứng .Chứng minh rằng trực tâm tam giác MNP,tâm đường tròn nội tiếp tam
giác ABC , tâm đường tròn ngoại tiếp tam giác ABC thẳng hàng.
Bài 6: Cho tam giác ABC nhận (I) là tâm đường tròn nội tiếp. Tiếp điểm của (I)
trên BC,CA,AB lần lượt là D,E,F.Phân giác trong tại I của tam giác BIC cắt BC ở M. AM cắt
FE ở N. Chứng minh rằng DN là phân giác của góc EDF.
Bài 7: Cho hình vuông ABCD ngoại tiếp (O). Tiếp điểm của (O) trên AB, BC, CD, DA lần
lượt là M, N, P, Q. Một điểm S nằm trên cung nhỏ PN của (O). Tiếp tuyến của (O) tại S cắt
BC, CD lần lượt tại H, K. Chứng minh MH//AK.

Bài 8: ( Liên Xô - 1985) Một đường tròn tâm O đi qua các đỉnh A và C của tam giác ABC
và cắt lại các đoạn AB, BC lần lượt ở K và N. Đường tròn ngoại tiếp của tam giác ABC và
KBN cắt nhau tại hai điểm phân biệt B và M. Chứng minh rằng góc OMB vuông .
Bài 9: ( Ukraina-1998) Cho tam giác ABC. Đường tròn (I) nội tiếp tam giác tiếp xúc với các

cạnh BC, CA, AB lần lượt tại K, L, M. Đường thẳng qua B và song song với MK cắt LM, LK
lần lượt ở R, S. Chứng minh rằng góc RIS nhọn.

20
TÀI LIỆU THAM KHẢO

[1] Hoàng Quốc Khánh – Khám phá ứng dụng của cực đối cực.

[2] O. Bottema Topics in Elementary Geometry, Second edition 2008.

[3] Nguyễn Mộng Huy- Các phép biến hình trong mặt phẳng

[4] Đào Huy Cường- Hàng điểm điều hòa, cực đối cực

[5] Toán học tuổi trẻ.

[6] Lê Hải Châu , Vô địch toán quốc tế IMO, 2007.

21
MỘT SỐ BÀI TOÁN HÌNH HỌC CÓ NHIỀU ỨNG DỤNG
Huỳnh Kim Linh

Giáo viên trường THPT Chuyên Lê Quý Đôn Khánh Hòa

I. MỘT BÀI TOÁN HAY CÓ NHIỀU ỨNG DỤNG


Một bài toán nhỏ rất đẹp với lời giải thuần túy hình học được áp dụng vào trong nhiều
tình huống khác nhau tạo ra các bài toán thú vị xuất hiện trong nhiều cuộc thi học sinh
giỏi.

Bài toán 1. Cho thuộc nửa đường tròn đường kính . Tiếp tuyến tại cắt
tại . cắt tại Chứng minh BE song song với

Lời giải. Gọi là trung điểm thì nên ta có là tứ giác nội tiếp.

Do đó

Mặt khác (cùng bù với )

1
Vậy tam giác suy ra . Từ đó ,
vậy .

Nhận xét . Bài toán tuy đơn giản, lời giải đẹp mộc mạc thuần túy hình học nhưng chứa
đựng những ý nghĩa sâu sắc. Chúng ta hãy xét một số ứng dụng của nó qua các bài toán
sau.

Bài toán 2. Cho tam giác ABC nội tiếp các đường cao . là đường
kính . cắt tại . thuộc sao cho vuông góc với
. Đường thẳng qua A vuông góc với lần lượt cắt tại . Tiếp tuyến
tại cắt nhau tại . Chứng minh rằng vuông góc .

Lời giải. Theo bài toán 1 thấy PM,QN đi qua trung điểm I của BC. Lấy L thuộc sao
cho . Dựng khi đó dễ thấy IO đi qua trung điểm PQ nên
chùm . Ta lại thấy lần lượt vuông góc với

2
do đó . Vậy tứ giác LXA’Y điều hòa, vậy tiếp tuyến tại
của cắt nhau trên . Ta có điều phải chứng minh.

Bài toán 3. Cho tam giác ABC nhon, không cân, các đường cao đồng quy
tại , các điểm lần lượt thuộc các cạnh . Gọi là trung điểm
gọi cắt đường thẳng qua vuông góc tại . Gọi giao tại

a) Chứng minh rằng .


b) Gọi DF cắt đường thẳng qua A vuông góc AC tại L. chứng minh rằng LM, NK,
AD đồng quy .
c) Gọi KL cắt EF tại P. chứng minh .

Lời giải.

a) Gọi Z trung điểm DE. Dễ thấy , vậy tứ giác AKNZ nội tiếp. Suy ra
. Mặt khác tứ giác AEDB nội tiếp nên . Từ đó ta
có .Mà Z, N trung điểm DE, AB do đó . Từ đó
nên . Ta có đpcm.

3
b) Gọi ML cắt BC tại T. Tương tự câu a) có do đó A, L, S và A, K, T
thẳng hàng. Áp dụng định lý Menelaus cho tam giác ASC với L, M, T thẳng hàng
Ta có suy ra . Tương tự . Từ đó chú ý
ta có

Do đó SK, LT, AD đồng quy tại Q. đpcm

c) Gọi DF giao BE tại I. Ta chú ý trong tam giác AFD có B, I, H thẳng hàng nên áp
dụng định lí Menelaus . Từ đó chú ý ta có

Do đó ta chứng minh được . Gọi đường thẳng qua A song song BC cắt LT tại
Y. Ta thấy các tam giác ALY và BIC có các cạnh tương ứng song song nên IL, CY
và BA đồng quy tại F. Tương tự SK giao AY tại X thì AK, BX, CA đồng quy tại E.
Gọi EF giao BC tại R. XY là đường thẳng qua A song song BC. Gọi XY cắt EF tại P,
ta sẽ chứng minh P, L, K thẳng hàng, thật vậy ta có biến đổi tỉ số.

Vậy P, L, K thẳng hàng ( đpcm ).

4
Bài toán 4. Cho tam giác ABC nhọn nội tiếp đường tròn (O) với AB<AC. Tiếp
tuyến tại A của (O) cắt BC tại T. D là điểm đối xứng của A qua O. OT cắt DB tại E.

a) Chứng minh rằng AE song song CD.


b) Gọi BE cắt AT tại F. Đường tròn ngoại tiếp tam giác AEF cắt EO tại G khác E.
chứng minh rằng tâm đường tròn nội tiếp tam giác AGB nằm trên (O).

Lời giải.

a) Gọi M trung điểm BC. Ta có . Chú ý . Vậy tứ giác AOMT nội


tiếp, suy ra hay suy ra hai tam giác đồng dạng
. Chú ý M trung điểm BC, O trung điểm AD ta suy ra hai tam
giác đồng dạng tương ứng . Vậy . Do tam giác
ABC nhọn, dễ có suy ra
. Ta có nên AE song
song CD.
b) Từ a) dễ có .

5
Do đó ta có suy ra tứ giác FGBE nội tiếp.
Từ đó dễ có Từ đó ta dễ có GO là phân giác góc AGB mà
OA = OB nên tứ giác AGOB nội tiếp. Gọi đoạn GO cắt (O) tại I. Ta dễ có OI =
OA = OB mà I thuộc phân giác GO nên I là tâm nội tiếp tam giác AGB ( đpcm ).

II. MỞ RỘNG BÀI TOÁN HÌNH HỌC


THI OLYMPIC NHẬT BẢN

Trong kì thi vô địch Nhật Bản năm 2012 có hai bài toán hình học hay như sau:

Bài toán 1: Cho tam giác ABC, tiếp tuyến tại A của đường tròn ngoại tiếp (O) của tam
giác cắt đường thẳng BC tại P. Lấy Q, R lần lượt đối xứng với P qua đường thẳng AB,
AC. Chứng minh rằng QR vuông góc BC.

6
Lời giải.

Gọi

Vì nên tứ giác PMNA nội tiếp.

Suy ra (với Ax là tia đối của tia AP)

Từ đó , suy ra

Mặt khác từ giả thiết ta có MN là đường trung bình của tam giác PQR, nên .

Từ hai điều trên ta có , đây là điều cần chứng minh.

Bài toán 2. Cho tam giác PAB và tam giác PCD sao cho PA = PB, PC=PD; P, A,C và
B, P, D lần lượt thẳng hàng. Đường tròn qua A, C cắt qua B, D tại hai điểm
phân biệt X, Y. Chứng minh rằng tâm đường tròn ngoại tiếp tam giác PXY là trung điểm
đoạn .

Lời giải. Gọi O là trung điểm của , ta chứng minh O là tâm đường tròn ngoại tiếp
tam giác PXY.

Cách 1.

Gọi lần lượt là bán kính của . Áp dụng công thức đường trung tuyến cho
tam giác và tam giác , ta có:
7
Mặt khác từ giả thiết ta có :

Áp dụng công thức đường trung tuyến cho tam giác ta có :

Từ (1), (2), (3) ta có , suy ra O là tâm đường tròn ngoại tiếp tam giác
PXY, đây là điều cần chứng minh.

Cách 2.

Gọi H, T, K là hình chiếu của lên PX.

Gọi

Ta có

8
Mặt khác từ giả thiết ta có

Kết hợp với hai điều trên suy ra

Vì O là trung điểm của nên .

Từ hai điều trên suy ra , do đó OT là đường trung trực của PX. Tương tự
trung trực PY đi qua O.

Suy ra O là tâm đường tròn ngoại tiếp tam giác PXY. Vậy ta có đpcm.

9
MỘT SỐ MỞ RỘNG:

Bài toán 3(mở rộng bài toán 1). Cho tam giác ABC nội tiếp đường tròn (O). P thuộc BC
và ở ngoài (O). T thuộc AP sao cho BT, CT cắt (O) lần thứ 2 lần lượt tại M, N và
Q đối xứng P qua MB, R đối xứng P qua NC. Chứng minh rằng QR vuông
góc
BC.

Lời giải. Gọi

Vì nên tứ giác PFTE nội tiếp.

Từ và tính chất góc nội tiếp ta có

Do đó .

Mặt khác từ giả thiết ta có EF là đường trung bình tam giác PQR nên
10
Từ hai điều trên suy ra , đpcm.

Bài toán 4. ( mở rộng bài toán 1 ). Cho tam giác ABC, đường tròn đi qua B, C cắt AC,
AB tại E, F. Cho BE giao CF tại H. Tiếp tuyến tại A của đường tròn ngoại tiếp tam giác
ABC cắt BC tại T.Trên AB lấy điểm Q, trên AC lấy điểm P sao cho TP song song BE,
TQ song song CF. Chứng minh rằng PQ song song với AH.

Lời giải. Gọi

Theo tính chất góc nội tiếp và góc tạo bởi tia tiếp tuyến và dây cung ta có

Suy ra .
2 2

Trước hết ta có DTAB ~ DTCA  ÊÁ ˆ˜ = ÊÁ


TA AC ˆ
˜
Ë TB ¯ Ë AB ¯

11
Mà TBTC
. = TA2 nên

Và (BCDK) = -1 nên

Theo định lí Thales

Suy ra

Suy ra

Xét tam giác PAQ và tam giác BGC có : và nên tam giác PAQ
đồng dạng tam giác BGC ( c – g – c ).

Từ đó

Suy ra PQ song song AH, đpcm.

Bài toán 5. (mở rộng bài toán 2). Cho tam giác ABC. M là điểm thuộc cạnh AB, N thuộc
AC sao cho MN song song với đương đối trung xuất phát từ đỉnh A của tam giác ABC.
Đường tròn đi qua M, B cắt đường tròn đi qua N, C tại hai điểm phân biệt X,
Y. Chứng minh rằng tâm đường tròn ngoại tiếp tam giác AXY là trung điểm đoạn .

Lời giải. Gọi O là trung điểm , ta chứng minh O là tâm đường tròn ngoại tiếp tam
giác AXY.

12
Cách 1. Đường đối trung xuất phát từ đỉnh A tam giác ABC cắt BC tại S.

Vì AS là đường đối trung tam giác ABC nên

Vì , nên và

Ta có:

Suy ra

Đến dây làm tương tự bài toán 2 ta thu được điều cần chứng minh.

Cách 2. Đường đối trung xuất phát từ đỉnh A của tam giác ABC cắt đường tròn ngoại
tiếp tam giác ABC tại điểm thứ hai L.

13
Gọi

Gọi H, T, K là hình chiếu lên AX

Ta có: tam giác AMN đồng dạng tam giác LCB ( g – g ), kết hợp với tứ giác ABLC điều
hòa ta được

Suy ra . Đến đây làm tương tự với bài toán 2 ta thu được đpcm.

Bài toán 6. Cho tam giác ABC nội tiếp (O). Tiếp tuyến tại A của (O) cắt BC tại . Lấy
đối xứng với qua AB, AC. Gọi là đường thẳng qua . Tương tự có
. Các đường thẳng cắt nhau tạo thành tam giác .

a) Chứng minh rằng .


b) Chứng minh rằng đồng quy tại một điểm S.
c) Chứng minh rằng S thuộc (ABC) và .
d) Chứng minh rằng (ABC) và trực giao.
e) Chứng minh rằng đường thẳng Simson ứng với S của (ABC) song song với đường
thẳng nối .

Lời giải.

a) Là kết quả của bài toán 1.


b) Gọi

Trước hết ta có

Tương tự ta có :

14
Nên theo định lí Menelaus đảo ta có thẳng hàng.

Tứ giác .

Tứ giác nội tiếp nên

Suy ra , do đó

Ta có : và nên

Tương tự . Mà thẳng hàng nên thẳng hàng.

Xét tam giác ABC và tam giác có giao điểm của các cặp cạnh tương ứng thẳng
hàng nên theo định lí Desargues ta có đồng quy tại điểm S, đây là điều
phải chứng minh.
15
c) 4 điểm cùng thuộc một đường tròn nên , suy ra
nên S thuộc (ABC).

Tương tự , ta có điều cần chứng minh.

d) Gọi M là điểm Miquel ứng với tứ giác toàn phần . Gọi


lần lượt là tâm đường tròn ngoại tiếp tam giác .

Theo tính chất đường nối tâm và dây chung ta có và suy ra


. Mặt khác nên .

Từ hai điều trên, kết hợp với A, B, C, M đồng viên, ta có đồng viên.

Tương tự đồng viên. Ta lại có suy ra


đồng viên. Như vậy ta có đồng viên, suy ra (ABC) và trực giao,
đây là điều phải chứng minh.

16
e) Đường thẳng Simson ứng với điểm S của tam giác ABC cắt AB, AC tại . Ta
có suy ra . Mặt khác theo câu b) ta
có , suy ra đường thẳng Simson ứng với điểm S của tam giác ABC
song song với đường thẳng nối , ta có điều cần chứng minh.

17
III. TỪ MỘT BÀI TOÁN QUEN THUỘC
TỚI CÁC BÀI TOÁN THI OLYMPIC

Chúng ta hầu như đều biết bài toán quen thuộc sau đây.
Bài 1. Cho tam giác ABC trên cạnh CA, AB lần lượt lấy các điểm E, F sao cho CE =
BF. Chứng minh rằng đường tròn ngoại tiếp tam giác AEF và đường tròn ngoại tiếp
tam giác ABC cắt nhau trên trung trực của BC và trung trực của EF.

Lời giải. Gọi trung trực BC và EF cắt nhau tại K. Dễ chứng minh các tam giác bằng
nhau 4KEC = 4KFB (c.c.c). Từ đây suy ra ∠KCE = ∠KBF vậy tứ giác AKCB nội tiếp.
Cũng từ hai tam giác bằng nhau suy ra ∠KEC = ∠KFB suy ra ∠KEA = ∠KFA vậy tứ
giác AKEF cùng nội tiếp. Vậy K cũng là giao cùa đường tròn ngoại tiếp tam giác AEF
và ABC. Ta hoàn tất chứng minh.

18
Nhận xét. Bài toán mà các bạn lớp 7 quen thuộc chính là chứng minh trung trực EF luôn
đi qua điểm cố định. Khi đó trong bài toán và lời giải không cần đến các yếu tố đường
tròn. Bạn nào đã quen thuộc phép biến hình thì có thể thấy K chính là tâm quay biến CE
thành BF và nội dung của bài toán cũng chính là cách dựng K, ta lấy giao điểm khác A
của đường tròn ngoại tiếp tam giác AEF và ABC. Bài toán này mang đậm chất biến hình
xong lời giải của bài toán cũng như trong toàn bộ bài viết này được trình bày một cách
đơn giản nhất chỉ mang nội dung kiến thức của cấp THCS chứ không thông qua các
phép biến hình.

Bài toán có nhiều ứng dụng hay mà nhiều đề thi các nước thậm chí là bài hình học thi
toán quốc tế năm 2013 cũng đã khai thác nó. Sau đây là một số ví dụ

Bài 2 (Olympic Toán toàn Nga 2006, lớp 10).


Lấy K, L là hai điểm trên các cung AB và BC của đường tròn ngoại tiếp tam giác ABC
sao cho KL // AC. Chứng minh rằng tâm nội tiếp các tam giác BAK và BCL cách đều
trung điểm cung ABC của tam giác ABC.

Bổ đề : Cho tam giác ABC nội tiếp đường tròn (O), tâm đường tròn nội tiếp I. Tia AI cắt
đường tròn (O) tại D khác A thì D là tâm đường tròn ngoại tiếp tam giác IBC.

Chứng minh
D là giao điểm thứ hai của AI và (O)
Suy ra D là trung điểm cung BC
⇒ DB=DC (1)

BID = B1 + A1 = B1 + A2
DBI = B2 + B3

Mà B1 = B2 , B3 = A2 nên DBI = DIB  DB = DI (2)

Từ (1) và (2) ta có đpcm

19
Giải bài toán.
Gọi I, J là tâm nội tiếp tam giác BAK và BCL. Gọi BI, BJ cắt đường tròn (O) ngoại tiếp
tam giác ABC tại M, N khác B, ta dễ thấy M, N là trung điểm các cung KA, LC.
Do KL // AC nên KA = LC và MN // AC
Do đó kết hợp bổ đề trên dễ chỉ ra MI = MK = NL = NJ.

Áp dụng bài toán 1 cho tam giác BMN nội tiếp (O) với MI = NJ ta suy ra PI = PJ với P
là trung điểm cung MBN . Ta chú ý MN // AC nên P cũng là trung điểm cung MBN vậy
ta có điều phải chứng minh.

Bài 3 (IMO 2013 bài 3). Cho tam giác giác ABC nội tiếp đường tròn (O). Đường tròn
bàng tiếp góc A,B,C lần lượt tiếp xúc BC,CA,AB tại D,E,F. Chứng minh rằng tam giác
ABC vuông khi và chỉ khi tâm ngoại tiếp tam giác DEF nằm trên (O).

20
Lời giải. K là trung điểm cung BAC . Từ tính chất của các tiếp điểm bàng tiếp ta dễ
chứng minh BF = CE nên theo bài toán 1 trung trực EF đi qua K. Nếu tâm ngoại tiếp
tam giác DEF thuộc (O) sẽ nằm ngoài tam giác DEF nên khi đó tam giác DEF tù. Không
mất tổng quát giả sử ∠EDF tù. Do tâm ngoại tiếp tam giác DEF cũng thuộc trung trực
EF vậy tâm ngoại tiếp tam giác DEF phải là giao của trung trung trực EF và (O). Giao
điểm này phải nằm trong góc ∠EDF nên giao điểm này chính là K. Vậy K cũng là tâm
ngoại tiếp tam giác DEF.
Dễ thấy các đường thẳng qua tâm bàng tiếp ứng với các góc A, B, C lần lượt vuông góc
với BC,CA,AB đồng quy tại điểm V. Từ đó các tứ giác DFBV,DECV nội tiếp. Chú ý K
là tâm ngoại tiếp tam giác
EDF nên ta suy ra ∠BVC = ∠BVD + ∠CVD = ∠AFD + ∠AED = 360◦ − ∠BAC −
∠EDF = 360◦ − ∠EKF – (360 − ∠EKF)/2 = (360 − ∠EKF)/2 .
Mặt khác KB = KC. Từ đó dễ suy ra K là tâm ngoại tiếp tam giác BVC hay KB = KV =
KC. Vậy ta chú ý rằng tứ giác DFBV nội tiếp và các cạnh FD,BV có chung đường trung
trực, từ đó theo tính chất đối xứng dễ suy ra VF = BD = AE và tương tự VE = CD = AF.
Vậy tứ giác AEVF là hình bình hành mà ∠AEV = ∠AFV = 90◦ vậy đó là hình chữ nhật
suy ra ∠BAC = 90◦. Ta có điều phải chứng minh.

21
Bài 4. Cho tam giác ABC nhọn có, trực tâm H, tâm ngoại tiếp O, bán kính đường tròn
BA.BH CA.CH 4R2
ngoại tiếp là R. Trên các tia BO,CO lấy các điểm K,L sao cho = =
BK CL BC
Chứng minh rằng trung trực KL đi qua trung điểm BC.

Lời giải. Gọi AD,BE,CF là đường cao của tam giác ABC. Gọi OB giao FD tại K0. Dễ
thấy BK là đường cao của tam giác BFD. Ta lại có tam giác BFD và tam giác BCA đồng
dạng nên
4 R 2 .BK
BE
BK ' FD HB.sinB HB BE.BH BA.BC = BK BE.2 R = BK .
= = = . Suy ra BK ' = =
BE AC AC 2R 2R 2R BA.BC
Do đó K ≡ K. Tương tự L là hình chiếu của C lên DE. Vậy ta chú ý rằng B,C là tâm
bàng tiếp của tam giác DEF do đó K,L là các tiếp điểm bàng tiếp với các cạnh DF, DE
nên ta dễ chứng minh FK = EL. Ta chú ý nếu M là trung điểm BC thì M, D, E, F cùng
thuộc đường tròn Euler của tam giác ABC hơn nữa dễ có ME = MF nên M chính là
trung điểm cung EDF của đường tròn Euler. Áp dụng bài tập 1 dễ chỉ ra trung trực KL
đi qua M. Ta có điều phải chứng minh.

22
Một kết quả đẹp khác từ bài toán 1 như sau
Bài 5. Cho tam giác ABC có tâm nội tiếp I. Gọi M, N, Q, P, S, R lần lượt là đối xứng
của B, C, C, A, A, B qua IC, IB, IA, IC, IB, IA. Gọi X, Y, Z là tâm ngoại tiếp các tam
giác AMN, BPQ, CRS.
a) Chứng minh rằng I là tâm ngoại tiếp tam giác XYZ.
b) Chứng minh rằng trực tâm tam giác XYZ là tâm ngoại tiếp của tam giác ABC.

Ta có bổ đề sau
Bổ đề : Cho tam giác ABC có tâm nội tiếp I, tâm ngoại tiếp O. Gọi M,N là đối xứng của
B,C lần lượt qua IC,IB thì MN vuông góc OI và bán kính đường tròn ngoại tiếp tam giác
AMN bằng OI.

Đây là một bổ đề rất quen thuộc và xuất hiện nhiều trong các tài liệu khác nhau,
các bạn có thể tham khảo nhiều lời giải trong [1,2,3] xin không trình bày lại chứng
minh. Quay lại bài toán

23
Lời giải. a) Theo bổ đề trên bán kính các đường tròn ngoại tiếp tam giác BPQ và
CRS bằng nhau mà C,Q và B,R đối xứng nhau qua IA, từ đó dễ thấy hai đường
tròn ngoại tiếp tam giác BPQ và CRS đối xứng nhau qua IA. Nên Y và Z là hai tâm
tương ứng đối xứng nhau qua IA vậy IY = IZ. Tương tự suy ra I là tâm ngoại tiếp
tam giác XYZ. Ta có điều phải chứng minh.
b) Ta chú ý rằng do tính đối xứng nên BN = CM cùng bằng BC do đó theo
bài toán 1 thì đường tròn (X) ngoại tiếp tam giác AMN đi qua D là trung điểm
cung BAC của đường tròn (O) ngoại tiếp tam giác ABC. Từ đó dễ suy ra OX
vuông góc AD. Ta chú ý AD chính là phân giác ngoài tại A của tam giác ABC nên
AD vuông góc AI do đó ta dễ suy ra OX // AI. Theo chứng minh trên Y Z đối xứng
nhau qua AI nên YZ vuông góc AI do đó YZ vuông góc OX. Tương tự dễ chỉ ra O
là trực tâm tam giác XYZ. Ta có điều phải chứng minh.
Nhận xét. Bài toán này có có một hệ quả đẹp là đường thẳng Euler của tam giác
XYZ cũng là đường thẳng OI của tam giác ABC. Ngoài ra ta còn chú ý rằng từ
chứng minh phần b) dễ suy ra IX = OA do đó đường tròn ngoại tiếp tam giác XYZ
và ABC có bán kính bằng nhau. Ta lại tiếp tục một bài toán khác liên quan tới bài
toán 1

24
Bài 6. Cho tam giác ABC. E,F di chuyển trên cạnh CA,AB sao cho CE = BF.
Chứng minh rằng đường thẳng Euler của tam giác AEF luôn đi qua một điểm cố
định khi E,F di chuyển.

Lời giải. Gọi G, L lần lượt là trọng tâm tam giác ABC và AEF. Gọi (O) và (K) lần
lượt là đường tròn ngoại tiếp tam giác ABC và AEF. LK, GO là đường thẳng Euler
của tam giác AEF và ABC. Gọi LK giao GO tại T ta sẽ chứng minh T cố định, thật
vậy, theo bài toán 1 thì (O) và (K) cắt nhau tại P trên trung trực EF và BC. Gọi M,
N là trung điểm BC, EF. Ta dễ thấy các tam giác cân PEF và PCB đồng dạng có
tâm ngoại tiếp lần lượt là K và O, trung điểm đáy lần lượt là N, M. Do đó theo tính
PK KO PO
chất đồng dạng dễ chỉ ra = = là tỷ số cố định, mặt khác từ đây cũng
PN MN PM
GL 2
suy ra MN // KO. Ta lại chú ý = và GL // MN. Do đó GL // KO
MN 3
GL GL MN 2 PM TG GL
và = . = . là tỷ số cố định. Từ đó ta có = không đổi
KO MN KO 3 PO TO KO
do đó T cố định. Ta có điều phải chứng minh.

25
Bài 7. Cho tam giác ABC. E, F nằm trên cạnh CA, AB sao cho CE = BF. BE giao
CF tại P. Gọi M, N là trung điểm của BC, EF. Q là một điểm trên đường thẳng
MN. Gọi R là đối xứng của P qua Q. Chứng minh rằng AR là phân giác của tam
giác ABC.

Lời giải. Gọi L là trung điểm AP ta đã quen thuộc với kết quả của đường thẳng
Gauss-Newton thì M,N,L thẳng hàng do đó theo tính chất đường trung bình thì AR
song song QL ≡ MN. Theo nhận xét bài trên thì AR là phân giác tam giác ABC. Ta
có điều phải chứng minh.

Tài liệu

[1] Nguyễn Minh Hà, Nguyễn Xuân Bình, Bài tập nâng cao và một số chuyên đề
hình học 10, NXBGD 2010
[2] Trần Quang Hùng, Tuyển tập các bài toán hình học chọn đội tuyển KHTN, năm
2013.

[3[3] Trần Quang Hùng, Tuyển tập các bài toán hình học chọn đội tuyển KHTN,
năm] Trần Quang Hùng, Từ một bài toán quen thuộc tới các bài toán thi Olympiad.

26
Định lý bốn điểm và ứng dụng giải một số bài toán hình học phẳng
Nguyễn Hữu Tâm – THPT chuyên Lê Quý Đôn Bình Định

Trong Hình học phẳng, bài toán về quan hệ vuông góc giữa hai đường thẳng có ảnh hưởng sâu
sắc và liên quan đến nhiều kết quả rất quan trọng và thú vị. Nhiều bài toán sau khi phân tích có thể quy
về việc chứng minh sự vuông góc giữa hai đường thẳng.

Có rất nhiều cách để chứng minh hai đường thẳng vuông góc nhau. Tuy nhiên, đứng trước một
bài toán hay một kết quả cần chứng minh của Hình học phẳng, khó khăn rất lớn đối với học sinh là lựa
chọn công cụ hay hướng tiếp cận bài toán đó.

Trong quá trình tham gia dạy lớp chuyên và bồi dưỡng đội tuyển học sinh giỏi các cấp, đặc biệt là
đội tuyển tỉnh thi quốc gia (phần Hình học phẳng) tôi nhận thấy có một cách để tiếp cận những bài toán
chứng minh hai đường thẳng vuông góc tương đối hiệu quả. Đó là sử dụng một mở rộng của định lý
Pytago (Pythagoras) mà ta gọi là “Định lý bốn điểm”.

1. Định lý bốn điểm

1.1. Định lý. Cho bốn điểm A, B, C, D phân biệt trong mặt phẳng. Khi đó AB vuông góc với CD
khi và chỉ khi AC 2 − AD 2 = BC 2 − BD 2 .

Chứng minh
Cách 1. (Dùng tích vô hướng)
Ta có
AC 2 − AD 2 = BC 2 − BD 2
 2  2  2  2
⇔ AC − AD = BC − BD
     
( )
⇔ DC AC + AD = DC BC + BD ( )
      
( )
⇔ DC AC + CB + AD + DB = 0 ⇔ AB.DC = 0 ⇔ AB ⊥ CD
Cách 2. (Dùng định lý Pytago). Gọi H, K lần lượt là hình chiếu của A, B lên đường thẳng CD.
Nếu AB vuông góc với CD thì H = K nên theo định lý Pytago ta có
AC 2 − AD 2 = HC 2 − HD 2 = BC 2 − BD 2 .
Ngược lại, nếu AC 2 − AD 2 = BC 2 − BD 2 = a thì ta có
a + CD 2
a = AC − AD = HC − HD = HC − ( CD ± HC )
2
2 2 2 2 2
⇒ HC = ∓
2CD
a + CD 2
Đổi vai trò H cho K ta cũng có KC = ∓ . Do đó HC = KC.
2CD
Lại đổi vai trò C bởi D ta cũng chứng minh được HD = KD. Như vậy H trùng K, suy ra AB vuông
góc với CD.

1
1.2. Nhận xét.
1/ Khi B trùng C thì ta có AB vuông góc với BD khi và chỉ khi AB 2 + BD 2 = AD 2 . Ta gặp lại
định lý Pytago.
2/ Định lý được phát biểu dạng điều kiện cần và đủ nên ta có thể vận dụng hai chiều. Ngoài
ra, điều kiện vuông góc được biểu thị dạng hiệu hai bình phương nên có thể lợi dụng tính chất
bình phương vô hướng của vectơ và phương tích của một điểm đối với một đường tròn.
3/ Theo cách chứng minh thứ nhất thì ta đã dùng công cụ tích vô hướng, còn theo cách
chứng minh thứ hai thì ta chỉ cần dùng đến định lý Pytago. Vì vậy với đối tượng là học sinh
THCS, các em cũng có thể vận dụng kết quả của định lý trên để chứng minh hai đường thẳng
vuông góc.

2. Ứng dụng định lý bốn điểm giải một số bài toán Hình học phẳng

2.1. Một số bài toán về chứng minh hai đường thẳng vuông góc

Bài toán 1. (IMO 1985). Cho tam giác ABC, một đường tròn tâm O đi qua A, C và cắt lại các cạnh
BA, BC tại K, N. Giả sử các đường tròn (BKN) và (ABC) cắt nhau tại hai điểm B và M, chứng
minh rằng BM vuông góc với MO.
B

M'
M C' K

N S

m
C
A
P
K' B' O

Lời giải 1. (Dùng phép đối xứng trục)


Gọi m là đường thẳng qua O và vuông góc với BM, ta cần chứng minh M thuộc m. Xét phép đối
xứng trục m, biến C, K thành C', K'. Khi đó CC', KK' cùng vuông góc với m. Do đó CC', KK', BM
song song nhau. Ta có
(C ' K , C ' C ) = ( AK , AC ) = ( NK , NB ) = ( MK , MB )(mod π ).

2
suy ra C', K, M thẳng hàng. Lại có
(CC ', CK ') = (C ' C , C ' K ) = ( AC , AK ) = ( AC , AB) = ( MB, MC ) = (CC ', MC )(mod π ).
suy ra C, K', M thẳng hàng.
Do đó các đường thẳng C'K, CK' giao nhau tại M.
Vì C'K và CK' đối xứng nhau qua m nên M thuộc m.

Lời giải 2. (Dùng phép nghịch đảo)


Gọi S, P, M' lần lượt là giao điểm của các cặp đường thẳng (KC, AN), (KN, AC), (SO, BP). Khi đó
theo kết quả của bài toán Brocard, ta có O là trực tâm tam giác BSP. Ta cần chứng minh M' = M.

Xét phép nghịch đảo f(O, R2), khi đó P biến thành B', A biến thành A và C biến thành C. Do đó
theo tính chất của phép nghịch đảo, đường thẳng AC không đi qua cực O nên biến thành đường
tròn đi qua O. Như vậy các điểm O, A, B', C đồng viên. Hơn nữa do BB’ ⊥ PO nên suy ra
PA.PC = PB'.PO = PM'.PB
suy ra M' thuộc đường tròn (ABC). Do đó
PK.PN = PA.PC = PM'.PB
suy ra M' thuộc đường tròn (BKN). Từ đó M' = M.

Lời giải 3. (Dùng biến đổi góc)


B

R K
M

N S
T
L
C
A
P
O

Giả sử tam giác ABC nhọn (trường hợp tam giác ABC tù giải tương tự). Gọi T và R lần lượt là tâm
 = BAC
các đường tròn (ABC) và (KBN) tương ứng. Vì BNK  nên hai tam giác BNK và BAC đồng
dạng. Do đó ta có
3
=
CBR ABT = 900 − 
ACB
Suy ra BR vuông góc với AC do đó BR // OT.
Tương tự, BT vuông góc với KN suy ra BT // OR. Do đó BROT là hình bình hành.
Gọi L là điểm đối xứng với B qua R thì RLOT cũng là hình bình hành và vì TR vuông góc với BM
nên ta có OL vuông góc với BM. Mặt khác LM cũng vuông góc với BM nên suy ra O, L, M thẳng
hàng, do vậy OM vuông góc với BM.
Nhận xét: Có thể dựa vào tính chất đường đối song để có BR vuông góc với AC và BT vuông góc
với KN, suy ra BROT là hình bình hành. Hơn nữa trung điểm I của RT cũng là trung điểm BO nên
IB=IO, lại có RT là trung trực BM nên IB=IM. Từ đó suy ra tam giác OBM vuông tại M.
Lời giải 4. (Dùng định lý bốn điểm)
Vì các đường thẳng BM, NK và CA là trục đẳng phương của các cặp của ba đường tròn nên chúng
đồng quy tại một điểm P. Do các tứ giác MNKB và KNCA nội tiếp nên ta có
 = BKN
PMN  = NCA
Suy ra tứ giác MNCP nội tiếp, do đó ta có
BM .BP = BN .BC = OB 2 − r 2
PM .PB = PN .PK = OP 2 − r 2
Suy ra
OB 2 − OP 2 = BP ( BM − MP) = ( BM + MP )( BM − MP ) = MB 2 − MP 2
Vậy, OM vuông góc với PB, suy ra đpcm.

Nhận xét. 1/ Bài toán này có nhiều cách tiếp cận khác nhau, mỗi cách có một thế mạnh riêng. Ở
lời giải 1, ta dùng tính chất của phép đối xứng trục, kiến thức lớp 11. Trong lời giải 2 ta đã dùng
đến phép nghịch dảo, một công cụ tương đối mạnh và chỉ được học ở các lớp chuyên. Còn ở các
lời giải 3 và lời giải 4 chúng ta dùng ít kiến thức hơn. Lời giải 3 tuy thuần túy về hình học nhưng
phải xét trường hợp (nếu không thì phải dùng góc định hướng giữa hai đường thẳng) và phải vẽ
thêm đường phụ. Với lời giải 4, ta không cần vẽ đường phụ mà chỉ dùng kiến thức của phương
tích, trục đẳng phương để tính toán rồi dùng định lý bốn điểm để suy ra đpcm, hướng tiếp cận này
tương đối nhẹ nhàng và tường minh.

2/ Từ các lời giải ở trên ta rút ra được một tính chất khá thú vị đó là các điểm O, L, S, M thẳng
hàng.

Bài toán 2. Cho tam giác ABC cân tại A, điểm P nằm trên tia BC; các điểm X, Y lần lượt nằm trên
tia BA và AC sao cho PX // AC và PY // AB. Gọi AT là đường kính của đường tròn ngoại tiếp tam
giác ABC. Chứng minh rằng PT vuông góc với XY.

Lời giải 1. (Dùng tích vô hướng)


Xét trường hợp P thuộc cạnh BC, ta có

4
   PC  PB    
TP. XY = 
 BC
.TB +
BC
.TC  XA + AY

( )
   
= PC.TB. AY + PB.TC. XA
   
( )
= PC.TB. AY cos TB, AY + PB.TC. XA cos TC , XA ( )
X

C P
B

T
Y
   
( ) ( )
Mà cos TB, AY = − cos TC , XA , TB = TC nên ta cần chứng minh PC.AY = PB.XA.
Thật vậy, ta có tam giác CYP đồng dạng với CAB đồng dạng với PXB nên suy ra
CY PX
= ⇒ CY .PB = PX .CP (*)
CP PB
Mặt khác PX // AC, suy ra BX = PX và PY // AB,
suy ra CY = PY = XA. Ta cũng có PX = AY nên từ (*) suy ra PC.AY = PB.XA.
 
Từ đó ta có TP. XY = 0 . Vậy TP vuông góc với XY.
Trường hợp còn lại (P không thuộc cạnh BC), giải tương tự.
Lời giải 2. (Dùng định lý bốn điểm)
Theo tính chất góc nội tiếp nửa đường tròn, ta có:
AB ⊥ BT ⇔ TX 2 = BX 2 + BT 2 (1)
AC ⊥ CT ⇔ TY = TC + CY = BT 2 + CY 2
2 2 2
( 2)
Từ (1) và (2) suy ra TX 2 − TY 2 = BX 2 − CY 2 (3)
Mặt khác, PX // AC, suy ra BX = PX (4)
Và PY // AB, suy ra: CY = PY (5)
Thế (4), (5) vào (3), ta được: TX − TY = PX − PY 2 ⇔ TP ⊥ XY (đpcm).
2 2 2

5
Nhận xét. Rõ ràng việc biến đổi góc là cách tiếp cận rất khó khăn ở bài toán này, do đó chúng ta
nghĩ đến công cụ tích vô hướng hoặc dùng định lý bốn điểm để tấn công. So sánh hai lời giải trên,
ta thấy rất rõ ràng cách tiếp cận bằng định lý bốn điểm trong trường hợp này là giản dị và nhẹ
nhàng hơn rất nhiều.

Bài toán 3 (2001- China) Cho tam giác ABC nhọn, O là tâm đường tròn ngoại tiếp tam giác và ba
đường cao AD, BE, CF gặp nhau tại H. Gọi M, N lần lượt là giao điểm của ED với AB và FD với
AC. Chứng minh rằng OB ⊥ DF , OC ⊥ DE và OH ⊥ MN .

Lời giải 1. (Dùng tính chất đường đẳng giác, trục đẳng phương và đường thẳng Euler)
Theo giả thiết, ta có DF đối song với AC. Mặt khác BE vuông góc với AC, còn BO và BE đẳng
giác trong góc 
ABC nên BO vuông góc với DF. Tương tự, CO vuông góc với DE. Ta còn phải
chứng minh OH ⊥ MN
Ta có MA.MB = ME.MD nên M thuộc trục đẳng phương của hai đường tròn (ABC) và (DEF).
Tương tự ta cũng có N thuộc trục đẳng phương của hai đường tròn này. Suy ra MN vuông góc với
đoạn nối tâm OO’ (với O’ là tâm đường tròn (DEF). Mà O, O’ và H cùng thuộc đường thẳng
Euler của tam giác ABC nên suy ra OH vuông góc với MN.
N

F
H O' O
B
D

M
Lời giải 2. (Dùng định lý bốn điểm)
Theo giả thiết ta có tứ giác ACDF nội tiếp, do đó
 = BDF
BAC  ⇒ OBC
 = 900 − BAC
 = 900 − BDF


6
 + BDF
Suy ra OBC  = 900 .
Vậy, OB ⊥ DF
Chứng minh hoàn toàn tương tự, ta cũng có OC ⊥ DE
Ta còn chứng minh OH ⊥ MN
Ta có:
MA ⊥ CH ⇔ MC 2 − MH 2 = AC 2 − AH 2 (1)
NA ⊥ BH ⇔ NB 2 − NH 2 = AB 2 − AH 2 ( 2)
DA ⊥ BC ⇔ DB − DC = AB − AC
2 2 2 2
(3)
BO ⊥ DF ≡ ND ⇔ BN 2 − BD 2 = ON 2 − OD 2 ( 4)
CO ⊥ DE ≡ MD ⇔ CM − CD = OM − OD
2 2 2 2
(5)
Lấy (1) - (2) + (3) +(4) - (5) vế theo vế, ta được:
HN 2 − HM 2 = ON 2 − OM 2 ⇔ OH ⊥ MN (đpcm).

Nhận xét
1/ Lời giải 1 có vẻ ngắn gọn, tuy nhiên ta đã dùng một số kết quả đã biết về đường thẳng Euler,
đường đẳng giác và trục đẳng phương. Còn lời giải 2 ta chỉ thuần túy sử dụng hai chiều của định
lý bốn điểm để giải, một cách tiếp cận khá bình dị.
2/ Nếu gọi giao điểm của BC và EF là P thì theo định lý Desargues ta có P, M, N thẳng hàng. Như
vậy vai trò của M, N, P trong bài toán là như nhau.

Bài toán 4. (1995-USSR) Cho tứ giác lồi ABCD thỏa điều kiện AB = AD và   = 900 .
ABC = CDA
Điểm F và E nằm trên đường thẳng BC và CD tương ứng sao cho DF vuông góc với AE. Chứng
minh rằng AF vuông góc với BE.

E
P
B

C
Lời giải 1 (Dùng biến đổi góc và tam giác đồng dạng)
Gọi X là giao điểm của AE và DF và P là giao điểm của AF với BE, ta có
AX . AE = AD 2 = AB 2

7
Do đó tam giác AEB đồng dạng tam giác ABX suy ra 
ABX = 
AEB mà 
ABX = 
AFX nên suy ra

AFX =  =
AEB ⇒ PFX XEP
Vậy, tứ giác XPEF nội tiếp suy ra đpcm.

Lời giải 2. (Dùng định lý bốn điểm)


Áp dụng định lý bốn điểm và từ giả thiết, ta có:
AE ⊥ DF ⇔ AD 2 − AF 2 = ED 2 − EF 2 (1)
AB ⊥ BF ⇔ AB − AF = − BF
2 2 2
( 2)
AD ⊥ DE ⇔ AD − AE = − DE
2 2 2
(3)
Lấy (1) - (2) + (3) ta được theo vế (với chú ý AD = AB) ta được
AB 2 − AE 2 = FB 2 − FE 2 ⇔ AF ⊥ BE (đpcm)

Nhận xét. Lời giải 1 thuần túy biến đổi góc nhưng phải viện đến điểm phụ là P và X. Lời giải 2
chỉ đơn thuần tính toán và áp dụng định lý bốn điểm.

Bài toán 5. Cho tứ giác ABCD nội tiếp đường tròn tâm (O) đồng thời lại ngoại tiếp một đường
tròn khác (O’), có các tiếp điểm N, P, Q, M lần lượt với các cạnh AB, BC, CD, DA của tứ giác đã
cho. Chứng minh rằng MP ⊥ NQ .

N
R

O
D
B O'
Q

P
C
F

Lời giải 1. (Dùng phép nghịch đảo)


8
Xét phép nghịch đảo đối với đường tròn (O’), các điểm A, B, C, D lần lượt biến thành các trung
điểm H, K, I, J của các đoạn MN, NP, PQ, QM. Tứ giác HKIJ là hình bình hành vì HK và IJ lần
lượt là đường trung bình của các tam giác NMP và QMP. Mặt khác vì các đỉnh A, B, C, D cùng
thuộc đường tròn (O) (không qua cực nghịch đảo) nên ảnh của chúng qua phép nghịch đảo cũng
thuộc một đường tròn. Do đó HKIJ là hình bình hành nội tiếp nên nó là một hình chữ nhật. Từ đó
suy ra HK vuông góc với HJ, do vậy suy ra MP vuông góc với NQ.
Lời giải 2. (Dùng góc định hướng giữa hai đường thẳng)
Gọi E, F lần lượt là giao của AB và CD, AD và BC. Gọi giao điểm của FO’ với AB, CD lần lượt là
R và T.
Ta có EO’, FO’ là các đường phân giác của các góc  AED,  AFB . Dễ thấy NQ vuông góc với EO’
và MP vuông góc với FO’. Như vậy ta chỉ cần chứng minh EO’ vuông góc với FO’.
Thật vậy, theo tính chất góc ngoài của tam giác ta có
( RT , RE ) = ( RF , FA) + ( FA, RA) = ( FC , FT ) + (CT , CF ) = (TC , TF ) = (TE , TR) ( mod π )
Suy ra tam giác ERT cân tại E, do đó EO’ vuông góc với RT hay EO’ vuông góc với FO’. Vậy
khẳng định của bài toán đã được chứng minh.

Lời giải 3. (Dùng định lý bốn điểm)

Gọi O’A ∩ MN = H và O’C ∩ PQ = E. Ta có tứ giác ABCD nội tiếp đường tròn (O) nên
  = 1800 . Tứ giác ABCD lại ngoại tiếp đường tròn (O’) nên theo tính chất hai tiếp tuyến cắt
A+C
nhau, ta có:
 1 1
O ' AM =  A ; O  ' CQ = C
2 2

⇒O 
' AM + O ' CQ = 90 ⇒ O
0  
' AM = CO 'Q

O ' M MA
∆O ' MA ∼ ∆CQO ' ( g .g ) ⇒ =
CQ QO '
Đặt MA = AN = x ; BN = BP = y ; CP = CQ = z ; DQ = DM = t ; O’M = O’Q = r.
r x
Khi đó ta có := ⇒ r 2 = x ⋅ z Tương tự ta cũng có :
z r
O'N NB r y
∆O ' NB ∼ ∆DMO ' ⇒ = ⇒ = ⇒ r 2 = y ⋅ t . Suy ra : r 2 = x ⋅ z = y ⋅ t
DM MO ' t r
Do AM và AN là hai tiếp tuyến của đường tròn (O’) nên ta có O’A ⊥ MN tại M và H là trung
điểm của MN. Áp dụng hệ thức lượng vào ∆O ' MA vuông tại M , có đường cao MH được :
1 1 1 1 1 1 4x2r 2 4x2 ( x ⋅ z )
= + ⇒ = + ⇒ MN 2
= ⇒ MN 2
=
MH 2 MA2 O ' M 2 1 
2
x2 r 2 x2 + r 2 x2 + ( x ⋅ z )
 MN 
2 

9
2 4x 2 ⋅ z
(do r = x.z ) . Suy ra : MN = 2
.
x+z
4 xz 2 4 y 2t 4 yt 2
Hoàn toàn tương tự, ta cũng có : PQ = 2
; NP =
2
; MQ =
2

x+ z y +t y+t
Suy ra

N
A
H

K M
O
O' J
D
P
I Q

C
4 x 2 z 4 xz 2 4 xz ( x + z )
MN + PQ =
2 2
+ = = 4 xz = 4r 2
x+z x+z x+z
(do r 2= x.z = y.t )
2
4 y t 4 yt 2
4 yt ( y + t )
NP 2 + MQ 2 = + = = 4 yt = 4r 2
y+t y +t y+t
Như vậy tứ giác ABCD có MN 2 + PQ 2 = NP 2 + MQ 2 . Vậy, MP ⊥ NQ.

Nhận xét. Đây là một tính chất hình học rất đẹp của tứ giác lưỡng tiếp.
- Lời giải 1 đã dùng đến phép nghịch đảo, đây là công cụ mạnh nên lời giải rất gọn gàng và đẹp
mắt.
- Lời giải 2 dùng biến đổi góc, đây cũng là một lời giải đẹp, kinh điển, thuần túy hình học.
- Lời giải 3 sau khi đặt MA = AN = x ; BN = BP = y; CP = CQ = z; DQ = DM = t; O’M = O’Q = r,
dùng định lý bốn điểm tính toán và cũng đưa đến kết quả. Tuy việc tính toán hơi cồng kềnh nhưng
hướng đi rõ ràng, trong khi đó hai lời giải trên tuy gọn gàng và đẹp mắt nhưng không dễ gì ai
cũng nghĩ đến.
10
- Bài toán có thể thay đường tròn (O’) nội tiếp tứ giác bởi đường tròn bàng tiếp thì kết quả vẫn
còn đúng.
Tương tự như vậy, ta xét tiếp bài toán sau, dù biến đổi và tính toán hơi phức tạp nhưng hướng đi
cũng rất rõ ràng.

2.2. Một số bài toán chứng minh song song, đồng quy, thẳng hàng
Nhiều bài toán chứng minh quan hệ song song, đồng quy hoặc thẳng hàng có thể đưa về
việc chứng minh hai đường thẳng vuông góc. Do vậy chúng ta cũng có thể vận dụng định lý bốn
điểm để tiếp cận những dạng toán như vậy.

Bài toán 6. Cho tam giác ABC và các điểm B’, C’ thuộc BC (khác B, C). Các điểm E, F theo thứ
tự thuộc AC, AB sao cho BE // B ' A;CF // C ' A. X là giao điểm thứ hai của đường tròn ngoại tiếp các
tam giác ABC, AB’C’. Chứng minh rằng AX // EF.

Lời giải 1. (Chứng minh hai góc đồng vị bằng nhau)


Gọi K, L theo thứ tự là giao điểm thứ hai của AE, AF và (AB’C’).

E F

A
X F'
E'

O'
K
O
L
B B' C
C'

Vì A, B’, C’, L, K đồng viên và BE / / B ' A;CF / / C ' A nên


BL BL.BA CA BB '.BC ' CA BB ' BC ' CA EA BA CA AE
= . = . = . . = . . = .
CK CK.CA BA CC '.CB ' BA CB ' CC ' BA CA FA BA AF
Dễ thấy ∆XLB ∼ ∆XKC (g − g)
11
BL XL
Do đó = .
CK XK
AE XL
Vậy, = .
AF XK
 = LAK
Kết hợp với EAF  = LXK,
 suy ra ∆AEF ∼ ∆XLK.
 = XKL
Do đó XAL  = EFA.

Điều đó có nghĩa là AX // EF.
Lời giải 2. (Dùng phương tích, độ dài đại số)
Ta cần có một bổ đề.
Bổ đề. Cho hai đường tròn (O1), (O2), trục đẳng phương ∆. M là điểm bất kì và H là hình chiếu
vuông góc của M trên ∆. Khi đó
PM /(O1 ) − PM /(O2 ) = 2O1O 2 .HM.
Trở lại giải bài toán 2.
Gọi (O), (O’) là đường tròn ngoại tiếp các tam giác ABC, AB’C’; E’, F’ theo thứ tự là hình chiếu
của E, F trên AX; K, L theo thứ tự là giao điểm thứ hai của AE, AF và (AB’C’) . Vì AX là trục
đẳng phương của (O), (O’) nên, theo bổ đề trên,

PE /(O') − PE /(O) 2O ' O.E ' E E ' E


= = (1).
PF /(O') − PF /(O) 2O ' O.F ' F F ' F
Mặt khác
CC '.CB '
PE/(O') - PE/(O) EA.EK − EA.EC
= = =
(
EA EK − EC
EA.CK
=
) CA
EA.
PF /(O') - PF /(O) FA.FL − FA.FB FA FL − FB (
FA.BL )
FA.
BB '.BC '
BA
EA BA CC ' CB ' BB ' BC ' CC ' CB '
= . . . = . . . = 1 (2).
CA FA BB ' BC ' CB ' CC ' BB ' BC '
Từ (1) và (2) suy ra E ' E = F ' F.
Điều đó có nghĩa là AX // EF.

Lời giải 3 (Dùng định lý bốn điểm)


Theo giả thiết thì AX là trục đẳng phương của hai đường tròn (O) và (O’), do đó AX ⊥ OO' . Ta cần
chứng minh FE cũng vuông góc với OO’.
Ta có
( ) (
OE 2 − OF 2 = OE 2 − R 2 − OF 2 − R 2 = EA.EC − FA.FB )
( ) ( )
O ' E 2 − O ' F 2 = O ' E 2 − R '2 − O ' E 2 − R '2 = EA.EK − FA.FL
Cần chứng minh
OE 2 − OF 2 = O ' E 2 − O ' F 2 ⇔ EA ( EC − EK ) = FA ( FB − FL )
KC FA
⇔ EA.KC = FA.LB ⇔ = (1)
LB EA
KC XK AF
Mặt khác hai tam giác XKC và XLB đồng dạng nên suy ra = = (2)
LB XL AE
12
Nhận xét. - Lời giải 1 chủ yếu sử dụng tam giác đồng dạng và định lý Thales để tính toán và đưa
đến chứng minh hai góc đồng vị bằng nhau. Đây là lời giải chỉ sử dụng kiến thức THCS và rất
kinh điển.
- Lời giải 2 sử dụng một kết quả quen thuộc về phương tích và dùng độ dài đại số để tính toán.
Tuy có dùng đến kiến thức lớp 10 nhưng lời giải này có tính tổng quát cho mọi trường hợp và áp
dụng được khi mở rộng bài toán.
- Lời giải 3 sử dụng định lý bốn điểm, hướng giải quyết rất rõ ràng và rất dễ áp dụng.

Bài toán 7. (APMO 2010). Tam giác ABC nội tiếp đường tròn (O) và có trực tâm H, (HAB) cắt
AC tại N và (HAC) cắt AB tại M. Chứng minh tâm đường tròn (HMN) thuộc HO.
Lời giải 1. (Dùng phép đồng dạng và định lý Thales)
U

A
N

M N'
Q O'
M' P
S O
R
H
B
C

Dễ chứng minh được HMM'B và HNN'C là những hình thoi đồng dạng. Gọi O' là tâm (HMN) và
gọi P, Q là hình chiếu của O' lên HM, HN thì P, Q là trung điểm HM, HN. Xét phép đồng dạng f
biến HMM'B thành HNN'C, khi đó biến HM thành HN và BM' thành CN', do đó biến P thành Q và
R thành S (R, S là hình chiếu của O lên HM, HN).

13
HP HQ
Do đó ta có =
HR HS
Gọi X, Y là giao của HO' với trung trực BM', CN' thì ta có
RX // PO' và SY // QO' nên
HX HR HS HY
= = =
HO ' HP HQ HO '
Suy ra X = Y do vậy X, Y là giao của trung trực BM' và CN' nên phải trùng O. Vậy H, O, O' thẳng
hàng.

Lời giải 2. (Dùng định lý bốn điểm kết hợp với phương tích và trục đẳng phương)
HM cắt AC tại U và HN cắt AB tại V, ta có UA.UC = UH.UM nên U thuộc trục đẳng phương của
(O) và (HMN). Tương tự, V cũng thuộc trục đẳng phương của (O) và (HMN).
Suy ra OO' vuông góc với UV. Ta cần chứng minh OH vuông góc với UV.
Ta có

ANV = 
ABH = 
ACH = 
AMU
nên suy ra MNUV nội tiêp suy ra
HM .HU = HN .HV
Do đó
OU 2 − OV 2 = ( OU 2 − R 2 ) − ( OV 2 − R 2 ) = UM .UH − VN .VH

( ) ( )
= UH UH + HM − VH VH + HN = HU 2 − HV 2
Vậy OH vuông góc với UV, từ đó suy ra đpcm.

Nhận xét. Việc phát hiện hai hình thoi HMM'B và HNN'C đồng dạng gợi cho ta hướng tiếp cận
bài toán bằng cách dùng phép đồng dạng. Tuy nhiên nếu phát hiện ra U, V có cùng phương tích
với hai đường tròn (O) và (HMN) thì bài toán quy về việc chứng minh OH vuông góc với UV, do
đó một cách rất tự nhiên, ta nghỉ đến phương tích và định lý bốn điểm.

=
Bài toán 8. Cho tứ giác ABCD có DAB  > 900 . Chứng minh rằng đường thẳng Euler
ABC = BCD
của ∆ABC đi qua D.

Lời giải 1 (Dùng biến đổi góc và định lý Ceva-sin)


 = A, 
Đặt BAC ABC = B, ACB = C . Khi đó do B > 900 nên B nằm trong tam giác AHC (H là trực tâm
tam giác ABC). Gọi O là tâm đường tròn ngoại tiếp tam giác ABC. Khi đó ta có
  = HCB
ACO = CAO  = HAB
 = B − 900
(do H, O là 2 điểm đẳng giác trong tam giác ABC)(1)
Suy ra O nằm ngoài tam giác AHC
Ta có
 = A + HAB
HAO  = A + 2 ( B − 900 ) = B − C = BCD
 + OAC  −C = 
ACD
Tương tự,
14
 = CAD
HCO  = B − A, HAD
 = HCD
 = 2 B − 900
Theo định lý Sin, ta lại có
sin   sin CAD
AHD sin HCD  AD HD CD
. . = . . =1
 sin CHD
sin HAD  sin 
ACD HD CD AD
 = HCD
Mà HAD  nên
sin  
AHD sin CAD
. =1
 sin 
sin CHD ACD

H C1 J
B1
I B N1
A M1 C
O

Từ đó ta có
sin   sin CAO
AHD sin HCO  sin   sin 
AHD sin HCO 
AHD sin CAD
. . = . = . =1
 sin 
sin CHD  sin CHD
ACO sin HAO  sin HAO
 sin CHD
 sin  ACD
Theo định lý Ceva sin cho tam giác AHC ta có CO, AO, DH đồng quy. Vậy, H, O, D thẳng hàng.

Lời giải 2. (Dùng định lý bốn điểm)


Gọi DA ∩ CB = M, AB ∩ DC = N. Các đường cao MM1, NN1 của các tam giác AMB và BNC cắt
nhau tại O.
=
Do DAB  nên các tam giác AMB , BNC là các tam giác cân tại M và N. Do đó OM và
ABC = BCD
ON lần lượt là các đường trung trực của AB và BC. Từ đó suy ra O là tâm đường tròn ngoại tiếp
∆ABC . Gọi H là trực tâm của tam giác ∆ABC . Vì 
ABC > 900 nên H nằm ngoài ∆ABC vµ ta cã HA
⊥ BC t¹i B1 vµ HC ⊥ AB t¹i C1 .

15
Như vậy, các tứ giác sau nội tiếp được: AM1B1M, AB1C1C, CN1C1N và MACN. Gọi I , J lần lượt là
trung điểm của MA và CN. Khi đó I , J lần lượt là tâm các đường tròn ngoại tiếp tứ giác AM1B1M
và CN1C1N. Do đó ta có
HB1 ⋅ HA = HI 2 − IA2 và OM 1 ⋅ OM = OI 2 − IA2
HC1 ⋅ HC = HJ 2 − JC 2 và ON1 ⋅ ON = OJ 2 − JC 2
HC1 ⋅ HC = HB1 ⋅ HA
Suy ra HI 2 − IA2 = HJ 2 − JC 2 ⇒ HI 2 − HJ 2 = IA2 − JC 2 (1)
Tương tự, OM 1 ⋅ OM = ON1 ⋅ ON ⇒ OI 2 − IA2 = OJ 2 − JC 2 ⇒ OI 2 − OJ 2 = IA2 − JC 2
Từ đó suy ra HI 2 − HJ 2 = OI 2 − OJ 2 ⇒ HI 2 + OJ 2 = OI 2 + HJ 2 .
Theo định lý 4 điểm thì OH ⊥ IJ (2)
Mặt khác, do tứ giác MACN nội tiếp được nên DA.DM = DC.DN
⇒ ( DI – IA ).( DI + IA ) = ( DJ – JC ).( DJ – JC )
⇒ DI 2 − IA2 = DJ 2 − JC 2 ⇒ DI 2 − DJ 2 = IA2 − JC 2 (3)
Từ (1) và (3) suy ra HI 2 − HJ 2 = DI 2 − DJ 2 ⇒ HI 2 + DJ 2 = DI 2 + HJ 2
Theo định lý 4 điểm thì DH ⊥ IJ (4). Từ (2) và (4) suy ra H, O, D thẳng hàng.
Do đó đường thẳng Euler của tam giác ABC đi qua D.

Nhận xét. Đây là một bài toán khó, vì giả thiết bài toán rất đơn giản và cũng rất khó khai thác.
Ở lời giải 1 ta đưa bài toán chứng minh ba điểm thẳng hàng về chứng minh ba đường thẳng đồng
quy, do đó ta sử dụng định lý Ceva-sin để công phá, từ đó đòi hỏi chúng ta phải biến đổi góc. Còn
ở lời giải 2, để chứng minh H, O, D thẳng hàng ta đưa về hai bài toán chứng minh vuông góc, đó
là chứng minh OH và DH cùng vuông góc với IJ, việc phát hiện ra các tứ giác nội tiếp AM1B1M,
AB1C1C, CN1C1N và MACN là rất quan trọng, vì nó gợi ý cho ta có thể sử dụng phương tích và
định lý bốn điểm để công phá bài toán.

Bài toán 9. (1997-USA) Cho tam giác ABC và dựng các tam giác cân BCD, CAE và ABF nằm bên
ngoài tam giác ABC với BC, CA và AB là các cạnh đáy. Chứng minh rằng các đường thẳng qua A,
B, C tương ứng vuông góc với EF, FD, DE thì đồng quy.

Lời giải 1. (Dùng tính chất của trục đẳng phương)


Xét các đường tròn có tâm là D, E, F và có bán kính lần lượt là DB, EC, FA. Khi đó đường thẳng
qua A và vuông góc với EF là trục đẳng phương của hai đường tròn (E, EC) và (F, FA), đường
thẳng qua B và vuông góc với FD là trục đẳng phương của hai đường tròn (D, DB) và (F, FA),
đường thẳng qua C và vuông góc với ED là trục đẳng phương của hai đường tròn (D, DB) và (E,
EC). Mà ba đường tròn này có tâm không thẳng hàng nên ba đường thẳng trên đồng quy tại tâm
đẳng phương của ba đường tròn.

Lời giải 2. (Dùng định lý Carnot)


Theo giả thiết ta có AE = CE , AF = BF , BD = CD . Do đó ta có

16
( AE 2
− AF 2 ) + ( BF 2 − BD 2 ) + ( CD 2 − CE 2 ) = ( AE 2 − CE 2 ) + ( BF 2 − AF 2 ) + ( CD 2 − BD 2 ) = 0

Áp dụng định lý Carnot cho tam giác DEF với các điểm A, B, C ta có các đường thẳng qua A, B, C
tương ứng vuông góc với EF, FD, DE thì đồng quy.

E
A

F
P

B C

Lời giải 3. (Dùng định lý bốn điểm)

Để chứng minh FE, FD, ED đồng qui, ta gọi P là giao điểm của đường thẳng qua B vuông góc với
FD với đường thẳng qua C vuông góc với DE, ta cần chứng minh PA vuông góc với FE.
Áp dụng định lý bốn điểm và sử dụng tích chất các tam giác cân, ta có:
PB ⊥ FD ⇔ PF 2 − PD 2 = BF 2 − BD 2 = AF 2 − CD 2 (1)
PC ⊥ DE ⇔ PD − PE = CD − CE = CD − AE
2 2 2 2 2 2
( 2)
Từ (1) và (2) suy ra: PF − PE = AF − AE ⇔ PA ⊥ FE (đpcm).
2 2 2 2

Nhận xét. Bài toán có nội dung rất đơn giản nhưng ý tưởng rất hay, những dữ kiện của nó có thể
dễ dàng gợi cho ta hướng tiếp cận bằng định lý bốn điểm hoặc định lý Canort. Tuy vậy nhưng lời
giải bằng cách sử dụng phương tích, trục đẳng phương cũng khá bất ngờ và độc đáo.

2.3. Một số bài toán về chứng minh điểm thuộc đường thẳng cố định hoặc đường
thẳng luôn đi qua điểm cố định

Về bản chất, bài toán chứng minh điểm thuộc đường thẳng cố định chính là chứng minh
tính thẳng hàng của các điểm mà ở đó đường thẳng chứa các điểm đó (giá của hàng điểm) có tính
cố định khi một số yếu tố thay đổi. Tương tự, bài toán chứng minh đường thẳng qua điểm cố định
chính là bài toán về sự đồng quy mà điểm đồng quy (tâm của chùm đường thẳng) có tính cố định

17
khi một số yếu tố khác thay đổi). Vì vậy, với dạng toán này chúng ta cũng có thể đưa về việc
chứng minh quan hệ vuông góc bằng cách vận dụng định lý bốn điểm như đã nêu trên.

Bài toán 10. Cho hai điểm B, C cố định trên đường tròn tâm O và A là một điểm thay đổi trên (O)
khác B và C. Gọi B’ là điểm đối xứng với B qua AC và C’ là điểm đối xứng với C qua AB. Chứng
minh rằng đường thẳng qua A và vuông góc với B’C’ luôn đi qua một điểm cố định.
C'
B'
A

E
D

O
B C

Lời giải 1. (Dùng tích vô hướng)


Do D, E lần lượt là trung điểm BB’, CC’ nên
     
B ' C ' = 2 DE − BC , AX = AO + OX
Do đó
     
( )(
B ' C '. AX = 2 DE − BC AO + OX )
       
= 2 AO.DE − AO.BC + 2OX .DE − OX .BC
Mà OX vuông góc với BC và AO vuông góc với DE nên ta có

18
         
( )
B ' C '. AX = OA.BC + 2OX .DE = OA.BC.cos OA, BC + 2OX .DE.cos OX , DE ( )
       
( ) ( )
= OA.BC.cos  OA, DE + DE , BC  + 2OX .DE.cos  OX , BC + BC , DE 
   ( ) (
 )
BC 2 π    BC 2 π   
=
2sin A 2
(
cos  + DE , BC  + ) 
 2sin BOC
(
cos  − DE , BC  = 0
2 
)
Lời giải 2. (Dùng định lý bốn điểm)
Gọi F là giao điểm của BC’ và B’C, ta có
 = 1800 − CBF
BFC  − BCF

(  − 1800 − 2 
= 1800 − 1800 − 2 ABC ) (
ACB )
 + 2
= 2 ABC ACB − 1800
 = 2 BAC
Mà BOC  nên từ đây suy ra tứ giác BOCF nội tiếp.
B ' X 2 − C ' X 2 = ( B ' X 2 − R2 ) − (C ' X 2 − R2 )
= B ' C.B ' F − C ' B.C ' F = B ' C ( B ' C + CF ) − C ' B ( C ' B + BF )
= BC ( CF − BF )
Dễ thấy A là tâm đường tròn bàng tiếp góc F của tam giác BCF nên
B’A2 – C’A2 = AB2 – AC2 = BC(CF – BF).
Như vậy ta có B ' X 2 − C ' X 2 = B ' A2 − C ' A2 . Từ đó suy ra AX vuông góc với B’C’.

Nhận xét. 1/ Việc dự đoán điểm cố định là tâm X của đường tròn (OBC) không quá khó, bài toán
quy về việc chứng minh AX vuông góc với B’C’. Tuy nhiên với bài toán này, việc biến đổi góc
gặp nhiều khó khăn, vì vậy có thể tiếp cận bài toán này bằng công cụ tích vô hướng hoặc định lý
bốn điểm. Việc phát hiện tứ giác BOCF nội tiếp và A là tâm bàng tiếp góc F của tam giác BCF là
điểm then chốt giúp ta tiếp cận bài toán bằng định lý bốn điểm và thu được lời giải đơn giản hơn
lời giải 1 rất nhiều.
2/ Nếu gọi Y, Z lần lượt là tâm các đường tròn, (OCA), (OAB) thì AX, BY, CZ đồng quy tại một
điểm gọi là điểm Kosnita của tam giác ABC. Kết quả của bài toán trên là một trong rất nhiều tính
chất đẹp của điểm Kosnita của tam giác.

Bài toán 11. Trên đường tròn (O) cố định lấy hai điểm B, C cố định và lấy điểm A thay đổi khác
B, C vẽ các đường cao BE, CF. Gọi M, S, N lần lượt là trung điểm của BF, FE, EC. Đường thẳng
qua M và vuông góc với BS cắt đường thẳng qua N và vuông góc với CS tại K. Chứng minh K
luôn chạy trên một đường thẳng cố định khi A thay đổi.

Lời giải 1. (Dùng tính chất trục đẳng phương và định lý Canort)
Gọi T là trung điểm BC và B’, C’ lần lượt là hình chiếu của B, C lên E, F. Dễ thấy B, C, E, F cùng
thuộc đường tròn đường kính BC, tâm T. Vì TS vuông góc với EF nên S là trung điểm B’C’, do đó
ta có SE.SC’ = SF.SB’ (*)

19
Ngoài ra ta cũng có AF.AB = AE.AC nên kết hợp với (*) ta suy ra AS là trục đẳng phương của
hai đường tròn (BF) và (CE). Suy ra AS vuông góc với MN. Xét hai tam giác SBC và TMN. Vì các
đường thẳng qua B, C, S lần lượt vuông góc với TM, TN, MN đồng quy tại A nên các đường
thẳng qua M, N, T và lần lượt vuông góc với BS, CS, BC cũng đồng quy tại K.
Vậy, K luôn nằm trên trung trực của BC.
A

E C'
F S
B'

N
O
M

T C

Lời giải 2. (Dùng định lý 4 điểm)


Vì MK vuông góc với BS và NK vuông góc vơi CS nên theo định lý bốn điểm ta có
BK 2 − SK 2 = BM 2 − SM 2
CK 2 − SK 2 = CN 2 − SN 2
Suy ra

BK 2 − CK 2 = ( BM 2 − SM 2 ) − ( CN 2 − SN 2 ) = ( BF 2 − BE 2 ) − ( CE 2 − CF 2 ) 
1
4 

= ( BF 2 + CF 2 ) − ( BE 2 + CE 2 )  = ( BC 2 − BC 2 ) = 0
1 1
4 4
Do đó K luôn cách đều B và C. Vậy K luôn nằm trên trung trực của đoạn BC.
Nhận xét. Bằng phương pháp giới hạn (cho A tiến đến điểm chính giữa cung BC) ta có thể dự
đoán đường thẳng cố định là trung trực của BC.
Trong lời giải 1, điều quan trọng là phát hiện ra AS là trục đẳng phương của hai đường tròn (BF)
và (CE), từ đó mới có ý tưởng dùng định lý Carnot thuận và đảo cho các tam giác SBC và TMN.
Còn ở lời giải 2, vì MK vuông góc với BS và NK vuông góc vơi CS nên dùng định lý bốn điểm ta
có thể dễ dàng chứng minh K cách đều B và C, từ đó suy ra K thuộc trung trực của BC.

20
Bài toán 12. Cho đường tròn (O) và đường thẳng d cố định không có điểm chung. Gọi H là hình
chiếu của O lên d, một điểm P thay đổi trên (O), tiếp tuyến tại P cắt d tại M, gọi N đối xứng với M
qua H, vẽ tiếp tuyến NQ với (O) sao cho MP và NQ không đối xứng nhau qua OH. Chứng minh
giao điểm K của MP và NQ nằm trên một đường thẳng cố định.

O Q

I
A B
K
P

M H N

Lời giải 1. Áp dụng Menelaus cho tam giác MNK ta chứng minh được H,P,Q thẳng hàng. Gọi I là
giao của PQ với OK thì OK vuông góc với PQ và do H,P,Q thẳng hàng nên suy ra I thuộc đường
tròn đường kính OH.
Ta có KQ2 = KI.KO, do đó K có cùng phương tích với hai đường tròn (O) và (OH).
Vậy, K thuộc trục đẳng phương của hai đường tròn cố định.

Lời giải 2. Gọi giao điểm của (O) và đường tròn (OH) là A, B, ta sẽ chứng minh AK vuông góc
với OH. Ta có
KM 2 + KN 2 MN 2
KH 2 − KO 2 = − − KO 2
2 4
( KM + KN ) 2 + ( KM − KN )2 − 4 MH 2
= − KO 2
4
= MP + KP − MH − KO 2 = MP 2 − ( MO 2 − OH 2 ) − AO 2
2 2 2

= ( OH 2 − OP 2 ) − AO 2 = AH 2 − AO 2
Vậy, AK vuông góc với OH, suy ra đpcm.

Nhận xét. Dễ phát hiện K, A, B thẳng hàng nên ta dự đoán đường thẳng cố định là trục đẳng
phương của hai đường tròn (O) và (OH). Nếu trong lời giải 1 ta cần đến định lý Menelaus và trục
đẳng phương thì ở lời giải 2 ta chỉ cần công thức tính độ dài trung tuyến và kết hợp định lý bốn
điểm để suy ra kết luận của bài toán.
21
Bài toán 13 (TST 2012). Trên mặt phẳng, cho đường tròn (O) và hai điểm cố định B, C trên
đường tròn này sao cho BC không là đường kính của (O) . Gọi A là một điểm di động trên đường
tròn (O) và A không trùng với hai điểm B,C . Gọi D, K, J lần lượt là trung điểm của BC, CA, AB
và E, M, N lần lượt là hình chiếu vuông góc của A,B,C trên BC, DJ, DK. Chứng minh rằng các
tiếp tuyến tại M, N của đường tròn ngoại tiếp tam giác EMN luôn cắt nhau tại điểm T cố định khi
điểm A thay đổi trên (O) .

Lời giải 1. (Dùng tính chất tứ giác điều hòa)

M' T

J K
X
M O
H
N'
S
N
Q C
B E D

Vì H là trực tâm của tam giác ABC nên các điểm D, H, E, M, N cùng thuộc đường tròn đường
kính DH .
Đường thẳng song song với BC và qua H cắt OD tại S, HS// BC và D là trung điểm BC nên ta có

H ( B, C , D, S ) = −1 ⇒ H ( M , N , D, S ) = −1
Suy ra DSNM là tứ giác điều hòa, suy ra T thuộc DO (1).
Gọi giao điểm của AD và (EMN) là D, X và BH cắt AC tại M’, suy ra B, H, M, M’ thẳng hàng; CH
cắt AB tại N’ suy ra C, H, N, N’ thẳng hàng. Vì DM’, DN’ là tiếp tuyến tại M’, N’ của đường tròn
đường kính AH nên AXN’M’ là tứ giác điều hòa. Suy ra
H ( M ', N ', A ', X ) = −1 ⇒ H ( M , N , E , X ) = −1
Suy ra tứ giác MNXE là tứ giác điều hòa, do đó T thuộc EX
Dễ thấy hai tam giác AEX và ADH đồng dạng và AE//TD suy ra

22
DX . AE DX . AD
DT = =
XA AH

XDC =  
AHX = CM 'X

Suy ra D, X, M’, C đồng viên, do đó


 = DM
DXC  
' C = DCA

Suy ra hai tam giác DCA và DXC đồng dạng do đó DX. DA = DC2, từ đó

DC 2
TD = (2)
2OH
Từ (1) và (2) suy ra T cố định.

Lời giải 2 (Dùng phép vị tự và biến đổi góc để chứng minh hai điểm trung nhau)

K
P' T N' N
D
T' O
P

M
A
H E
J
M' B
Gọi P là giao điểm của các tiếp tuyến tại B và C của (O) và P’ là ảnh của P qua phép đối xứng
trục BC. Bằng cách xét trường hợp giới hạn khi tam giác ABC cân tại A ta dự đoán được điểm cố
định là trung điểm T’ của P’D. Ta sẽ chứng minh điều này.

Xét phép vị tự tâm D tỷ số k = 2 biến M, N, T’ lần lượt thành M’, N’, P’. Dễ chứng minh được hai
tam giác P’BM’ và P’CN’ bằng nhau, suy ra hai tam giác P’M’N’ và P’BC đồng dạng và
T
' MN = T 
' NM = BAC
Ngoài ra dễ thấy các điểm M, N, E, D và H đồng viên nên

 = MEN
TMN  = MDN
 = BAC
 = T
' MN

23

 = T
TNM ' NM
Từ đó suy ra T’ trùng T. Mà T’ cố định, vậy T là điểm cố định.

Lời giải 3. (Dùng định lý bốn điểm kết hợp với tính chất trục đẳng phương và biến đổi góc)

B' T

J K
M
O
C'
H

I N

B E R D S C

Gọi R, S lần lượt là trung điểm của DB, DC thì R, S lần lượt là tâm đường tròn ngoại tiếp các tam
giác BMD, CND. Ta có
1 1 1
TM = TN , MR = DB ' = BC = DC ' = NS
2 4 2
Bằng biến đổi góc, ta thu được
 = TNS
TMR  ⇒ ∆TMR = ∆TNS ⇒ TR = TS

Do đó T nằm trên đường trung trực của BC.


Gọi X là tâm đường tròn ngoại tiếp tam giác HBC thì X cố định. Ta sẽ chứng minh T nằm trên
trục đẳng phương của đường tròn (S) và (X) hay phải chứng minh TC vuông góc với SX. Gọi U
là trung điểm của OD. Ta thấy
TC ⊥ SX ⇔ TX 2 − XC 2 = TS 2 − SC 2
⇔ TX 2 = TS 2 + XD 2 + CD 2 − SC 2 = TD 2 + SD 2 + XD 2 + CD 2 − SC 2 = TC 2 + XD 2
⇔ (TD + XD ) = TC 2 + XD 2 ⇔ CD 2 = 2TD. XD ⇔ DS 2 = DU .DT
2

24
Điều này tương đương với tam giác TSU vuông tại S. Hơn nữa, ta thấy
 = 900 ⇔ STU
TSU  + SUT
 = 900
 + BXC
⇔ RTS  = 1800 ⇔ MTN
 + MIN
 = 1800

Đẳng thức cuối đúng nên suy ra T nằm trên trục đẳng phương của (S) và (X). Do hai đường tròn
này cố định nên trục đẳng phương của chúng cũng cố định. T là giao điểm của hai đường thẳng cố
định nên T là điểm cố định. Ta có đpcm.
Nhận xét. Đây là bài hình TST ở ngày thi thứ nhất và ở vị trí là Bài 1, tuy nhiên đây là một bài
toán khó. Nếu giải bằng hình học thuần túy thì đòi hỏi phải kẻ thêm đường phụ. Ở lời giải thứ
nhất ta phải dùng đến tính chất của tứ giác điều hòa và hàng điểm điều hòa, ở lời giải 2 ta lại dùng
phép vị tự và biến đổi góc. Còn lời giải 3, sau khi phát hiện T thuộc trung trực của BC cố định ta
cón phải chỉ ra T thuộc một đường cố định khác, đó là trục đẳng phương của hai đường tròn (S) và
(X), tuy nhiên điều này lại quy về việc chứng minh TC vuông góc với SX nên định hướng ta nghĩ
đến định lý bốn điểm và đã thu lại kết quả.
Ngoài 3 lời giải trên, bài toán còn có thể giải bằng phương pháp vectơ và phương pháp tọa
độ, tuy hướng đi rõ ràng nhưng việc tính toán tương đối phức tạp.

3. Bài tập.

Bài 1. Cho tam giác ABC, gọi E, F đối xứng với B, C qua AC, AB. Gọi D là giao điểm CE và BF
và K là tâm đường tròn ngoại tiếp tam giác DEF. Chứng minh AK vuông góc BC.
Hướng dẫn giải.
KB 2 − KC 2 = PB /( K ) − PC /( K ) = BC ( CD − BD )

Để ý rằng
AB 2 − AC 2 = ( p − DB ) 2 − ( p − DC ) 2 = BC.( DC − DB ) (với p = (BC+CD+DB)/2 ).

Bài 2. Cho tam giác ABC có BC < AB < AC có tâm nội tiếp là I. Gọi E thuộc AC, F thuộc AB sao
cho CE = BC = BF.
a) Gọi K là tâm ngoại tiếp tam giác AFE. Chứng minh IK vuông góc với BC.

b) Gọi O là tâm ngoại tiếp tam giác ABC. Chứng minh OI vuông góc với EF.
Hướng dẫn giải
a) Tương tự bài 1
b) OE 2 − OF2 = (OE 2 − RO 2 ) − (OF2 − RO 2 ) = EA.EC − FA.FB = BC ( AB − AC )
IE 2 − IF 2 = PE /( I ) − PF /( I ) = EM 2 − FN 2 = ( BC − ( p − AB ))2 − ( BC − ( p − AC )) 2
= ( AB − AC ).BC (M,N lần lượt là tiếp điểm của (I) lên AC, AB).

Bài 3. Cho tứ giác lồi ABCD có AB = AC = BD = a. Gọi P là giao các đường chéo; O và I là tâm
nội tiếp, tâm ngoại tiếp tam giác PAB. Chứng minh OI vuông góc với CD.
Hướng dẫn giải.
Giả sử (I) tiếp xúc PA, PB tại E, F.

25
OC 2 − OP 2 = CP.CA; OD 2 − OP 2 = DP.DB ⇒ OC 2 − OD 2 = a ( PC − PD)
IC 2 − ID 2 = CE 2 − DF 2 = a ( PC − PD)

Bài 4. Cho tam giác ABC có tâm ngoại tiếp (O). Điểm P trên phân giác góc BAC. Các đường
tròn (PAB), (PAC) cắt CA, BA tại E, F khác A. Chứng minh rằng PO vuông góc EF.
Hướng dẫn giải.

OE 2 − OF 2 = PE /(O ) − PF /(O )
= EA.EC − FA.FB = EP.EM − FP.FN
= PE 2 − PF 2 + ( PM .PE − PF .PN )
= PE 2 − PF 2

Bài 5. Cho tam giác ABC nội tiếp (O) cố định, B và C cố định. Hai điểm E, F đối xứng với B, C
qua AC, AB. Chứng minh rằng đường thẳng qua A vuông góc EF đi qua điểm cố định khi A di
chuyển.

Hướng dẫn giải. Điểm cố định là tâm đường tròn (OBC).


Gọi CE ∩ BF ≡ D ⇒ D ∈ (OBC ), K là tâm (OBCD). Cần chứng minh AK ⊥ EF
KE 2 − KF2 = PE /( K ) − PF /( K ) = EC.ED − FB.FD
= BC ( ED − FD ) = BC ( DC − DB ) = AB 2 − AC 2 = AE 2 − AF2

Bài 6. Cho tam giác ABC nội tiếp (O) cố định. B và C cố định. (K) cố định đi qua B, C cắt AB, AC
tại F, E. Đường BE giao CF tại H và HB, HC cắt (HCA), (HAB) tại M, N khác H. Chứng minh
đường thẳng qua A vuông góc MN đi qua điểm cố định.

Hướng dẫn giải. Điểm cố định là tâm (BCD).


Gọi NB ∩ MC ≡ D , L là tâm (BCD), chứng minh AL vuông góc MN.
LM 2 − LN 2 = MC.MD – NB.ND
AM 2 − AN 2 = ( AM 2 − AD 2 ) − ( AN 2 − AD 2 ) = ( PM − PD ).MD − (QD − QN ).ND

Bài 7. Cho tam giác ABC, đường tròn nội tiếp (I) tiếp xúc AC, AB tại E, F. Điểm P di chuyển trên
EF, PB cắt CA tại M và đường thẳng qua C vuông góc với AC cắt MI tại N. Chứng minh rằng
đường thẳng qua N vuông góc với PC luôn đi qua một điểm cố định khi P di chuyển.
Hướng dẫn giải.
Ta cần bổ đề sau: Cho tứ giác ABCD ngoại tiếp đường tròn (I). Các điểm M, N lần lượt thuộc IB,
ID sao cho AM vuông góc với AB và AN vuông góc với AC. Khi đó MN vuông góc với BD.
Để chứng minh bổ đề trên ta chỉ cần sử dụng định lý bốn điểm và để ý tính chất phân giác
và tính chất tứ giác ngoại tiếp.
Trở lại bài toán, gọi Q thuộc AB sao cho MQ tiếp xúc (I). Để ý CQ, MB, FE đồng quy tại
P. Đường thẳng qua C vuông góc với BC căt BI tại R. Áp dụng bổ đề cho tứ giác BQMC ngoại
tiếp suy ra NR vuông góc với CP.
Bài 8. Cho tam giác ABC, điểm M thuộc tia AB và điểm N thuộc tia CB sao cho AM = CN = p (p
là nửa chu vi tam giác ABC). Gọi I là tâm đường tròn nội tiếp tam giác ABC và BK là đường kính
đường tròn (ABC). Chứng minh KI vuông góc với MN.

26
Hướng dẫn giải. Ta chứng minh
IM 2 − IN 2 = BC 2 − BA2 = KM 2 − KN 2
Bài 9. Cho tam giác ABC nhọn, O là tâm đường tròn ngoại tiếp tam giác và ba đường cao AD, BE,
CF gặp nhau tại H. Gọi M, N lần lượt là giao điểm của ED với AB và FD với AC. Các đường
thẳng BE, CF cắt (O) tại E', F', giả sử E'F' cắt BC tại X và BF' cắt CE' tại Y. Chứng minh XY//MN.
Hướng dẫn giải. Áp dụng định lý Brocard cho tứ giác E’F’BC nội tiếp ta suy ra OH vuông góc
với XY. Theo kết quả Bài toán 3 ta suy ra đpcm.

Tài liệu tham khảo

1. V.V. Praxolov, Các bài toán về Hình học phẳng, Tập II, NXB Hải Phòng, 2002.
2. Nguyễn Minh Hà, Bài giảng tại trường Xuân toán học 2015.
3. Trần Quang Hùng, Bài giảng các đội tuyển VMO tại Phú yên, 2014.
4. Đỗ Thanh Sơn, Bài giảng các đội tuyển VMO tại KonTum, 2012.
5. Tài liệu từ internet, các diễn đàn mathlinks.ro và mathscope.org.
6. Tạp chí Mathematical Excalibur.

27
PHÉP NGHỊCH ĐẢO

VÀ ỨNG DỤNG TRONG GIẢI TOÁN HÌNH HỌC


GV. Phạm Đoan Ngọc – Trường THPT chuyên Lý Tự Trọng – Cần Thơ

M biến hình
ế Ph p Ngh h o Ph p
ngh h o

Tr Ph p ngh h ov ng ng trong gi i to n hình học

Mục lục
1. C
1.1. ……………………………………………………………………….....Trang 2
1.2. ……………………………………………………………………………Trang 2
1.3. ết qu khác hình h c……………………………..Trang 10
2. Ứ ụ
2.1. ………………………… .Trang 11
2.2. ……………………………………… 15
2.2.1. ……………………………………… 15
2.2.2. cm ò ……………………………… 16
2.2.3. m th …………………………………………… 20
2.2.4. ng th ồ ……………………………………… 24
2.2.5. ………………………………… 28
2.3. ế …………………………… …… 31
2.4. ………………………………………………………………… ang 35
3. ụ ……………………………………………………………………………Trang 37
4. Tài liệu tham kh o………………………………………………………………………...Trang 40

1
Nội dung

1. C
i m t s kiến th m
ế ng ng ph p
ngh h o trong gi i to n h nh họ

1.1. Đ .

I k 0 I k t

ng v i MI M’ sao cho . ' k .


IMIM
M’ M I k.

NIk : M M' hay NIk (M) M'


(H) (H') {M'/ M'  NIk (M), M(H)} (H) qua NIk

1.2. t.
1.2.1. NIk k 0 ò I

k. ò ò

C
M NIk (M) M M

. IM2 k IM k M(I, k)


IMIM
M’ ò (I, k)
IM'  k IM'2 k IM'.IM k NIk (M') M'
hay M’ NIk
NIk ò I k.
1.2.2. NIk k 0 ế M'  NIk (M) M'  M ò
MM’ ò

2
 M ò (I, k) :
A ế ế ế M ò I)
. ' k IA2
IMIM nên AM’ OAMAM'
M ò (I, k)
 M ò (I, k)
1.2.3. I k ế 1–1
1.2.4. ế M’ M I k M, I, M'
1.2.5. T nh h t ih p ế I k ế M M’

M’ ế M NIk NIk  I hay NIk   1


 NIk )
Chú ý : Do tính ch i h p nên ta sẽ dùng ký hi u NIk : M M' chỉ phép ngh o
biế m M m M' c l i.
k2
1.2.6. NIk1 NIk2 tâm I
k1
1.2.7. Cho A', B' A, B NIk
i) ế A, B, I A', B', A, B
ế A, B, I A', B', A, B ò

C
i) Hi n nhiên
ii) A, B, I

. '  k IA  IB' IAB IB' A' I BAIA' B'


. '  IBIB
IAIA
IB IA'
A', B', A, B ò
3
k
1.2.8. ế A', B' A, B NIk A' B'  .AB
IAIB
.
C
 Trường h p I, A, B th ng h ng

. ' k nên A' B'  IB' IA'  k  k k IAIB k. AB


. ' IBIB
IAIA
IB IA IAIB
. IAIB
.
k
A' B'  .AB
IAIB
.

 Trường h p I, A, B h ng th ng h ng

. '  IBIB
IAIA . '  k IAIA
. '  IBIB
. ' k
IAB IB' A'
k
 A' B'  IA'  IA'.IA 
AB IB IAIB . IAIB
.
AB
' ' mAB
1.2.9. Ả
M t trong t nh h t quan trọng nh t a ph p i n h nh t nh h t v nh a
h nh qua ph p i n h nh ta h t nh a hai h nh qu n thu ường th ng v
ường tr n qua ph p ngh h o

Đ 1.
C
NIk d I d
Ad, A I A'  NIk (A) I,A,A' A'd

4
NIk (d) d

Đ . ò
C

NIk d I d
ẽ IH d H
H'  NIk (H)
Md, M  H, M'  NIk (M)
. '  IMIM
IHIH . ' M, M', H, H' ò IM' H' M'
M' ò IH'
Khi M d M’ ò IH' M M' I
ế M’ ò IH’ ( M'  I
M  NIk (M') . . ' IHIH
IMIM . ' M,M', N, N' ò

IM' M' H' nên HM  IH d  IH, d qua H nên Md


V

C NIk
 ế d I d d' d
 ế d I
1: ế H I lên d
H'  NIk (H)
5
 d ò IH’
2 A, B d
A’, B’ A, B NIk
 d ò (IA' B')

Đ 3. ò (O) I d I d
ế ế (O) I.

NIk ò (O) I (O)


M, N, I (O) M', N', I ' NIk
ẽ M', N', I '
Khôn IM, IN IP.

OMN ON'M' MON IM  IN' ) IN' M'  IMN


IN IM'
IN' P'  IPN
IMNP ế (O)
IPN IMN 1800
Nên
M', N', P'

C NIk
+ A, B ò O)
6
+ A’, B’ NIk
 A’, B’ O)
Đ 4. ò (O) I NIk

ò O’ ò O’ O VI tâm I   k
p
p I ò O)

NIk ò O I (O)
M ò O N IM O)
k
M'  N (M) IMIM
k
. ' k IMIN
k
.  PI /(O)  p nên IM'  IN M' VI p (N)
I
p
M’ ò O’

C NIk
+ A, B, C ò O)
+ A’, B’, C’ A, B, C qua NIk
 ò O) qua NIk ò ế A’B’C’

1.2.9.
N u như ph p ời h nh như ph p t nh ti n, ph p i ng tr , ph p i ng t m,
ph p qua , o to n ho ng h gi a hai i m t ph p v tự, ph p ng ạng th ại o
to n t s ho ng h gi a hai i m th , ph p ngh h o ại t nh h t o to n g
gi a h nh Trư ti n, ta nh ại m t s h i ni m v g gi a h nh qu n thu

7
Đ . d ò O M d
ò O d ế ế ’ O) M. (d,(O))

Ví dụ ế d O d O 00

Đ .C ò O O’ ò O O’
ế ế O O’

Ví dụ - ế ò ế 00

- ò (O; R) (O'; R')


00 ế ế
8
Đ 4. NIk hai
ò ò
C

NIk

d qua I d O I O
’ ế ế O I
 d O d ’

NIk

O O’ I d
’ I.
d ế ế O I O’ ế ế O’ I

9

NIk
d I d qua NIk ò O’ I ế
ế O’) I d.
O1 O) qua NIk O) không qua I nên (O1 ò
O VI
d O M O O’ M’ M’ M
NIk
d1 ế ế O M, d2 d1 VI d2 / /d1 d2 ế
O1 M’.
ế ế ò O1 O’ M’ d
(O M 

NIk .(C
Hệ :
i) ế ò ế ế

ii) ế ò ế ế
ế
C

1.3. ộ khác :
1.3.1. ò ồ
1.3.2. ồ
ò
1.3.3. ò
ò
1.3.4. ò ò
ò ẽ
1.3.5.
10
2. Ứ ụ
2.1. C
ng ng ph p ngh h o trong i to n h ng minh h th h nh họ , ta
thường họn ph p ngh h o th h h p v p ng t nh h t

.( ABCD ế
ò .  ABCD
ACBD . ADBC
.

() ABCD ế ò O ò ế ABCD


ẽ .  ABCD
ACBD . ADBC
.
(O) d
B B'
NAk , (k 0) C C' B',C', D'

DD'
k BC k CD k BD
C' B’ D’ nên ' 'C' D' B' D' 
BC   
ABAC
. ACAD
. ABAD
.
() ABCD .  ABCD
ACBD . ADBC
. 1 ẽ ABCD ế

BB'
N , (k 0) C C'
k
A 

DD'

(1)  BC  CD  BD B'C'C' D'  B' D' B',C', D' C’


ABAC
. ACAD
. ABAD
.
B,C, D ò O A.

11
.( ) Cho (O; R),(I;r) ò ế ò ế
ABC. OI2  R2 2Rr

(I,r) ế AB, BC, AC F, E, D.

M, N, P IA EF, IB DF, IC DE

r2 ID2 IE2 IF2 IMIA


. INIB
. IPIC
.
2
NIr

DD, E E, F F, AM, B N, C P và (ABC) (MNP)


R’ ò ế MNP
2 2 2
R'  r R 2r 2 R 2 r 2 R (1)
PI/(O) d R R OI
SDEF  4SMNP

DEDFEF
. .  4 MNM
. PNP
. 2MN.2MP.2NP  4MNM . PNP
. R'  1 r (2)
4r 4R' 4r 4R' 2
2 1 (R2 OI2) 1 r  rR2 OI2  R2 2Rr
2

ABCD ò ế ABC
ò ế ABD ỉ AB2.CD2  AC2.BD2  AD2.BC2

12
BB'
N (k 0) C C' (ABC) B'C'
k
 (ABD) B'D'

A

DD'
(ABC) (ABD) B'C' B'D' C'D'2  B'C'2B' D'2
 k CD   k BC   k BD 
2 2 2
  
.   ABAC
 ACAD .   ABAD
. 
 k CDAB
.   k BCAD
.   k ABAD
. 
2 2 2

AB2.CD2  AC2.BD2  AD2.BC2

. ABC ế ò (O) M (O) .


MA, MB, MC (O) A', B',C' .
SABC
' ' '  MA'.MB'.MC'
SABC MAM . BM
. C

SABC  ABBCCA
. . SA'BC' '  A' B'.B'C'.C' A'
4R 4R
SABC
' ' '  A' B'.B'C'.C' A' (1)
SABC ABBCCA
. .
NMk k MAM
. A' MBM
. B'
(O) (O), AA', B B',C C'

13
k k k
A' B'  .AB; B'C'  BC ; C' A'  .CA (2)
MAM
. B MCM
. B MCM
. A
1 2

. ò (O1),(O2),(O3),(O4) sao cho (O1) (O3) ế P,


(O2) (O4) ế P, (O1) (O2) A, (O2) (O3
B, (O3) (O4) C, (O4) (O1) D ( A, B,C, D P).

.  PB2
ABBC
ADDC
. PD2

NPk (k 0)
AA', B B', C C', DD' (O1) d1, (O2) d2, (O3) d3,(O4) d4
(O1), (O3) ế i nên d1 / /d3 d2 //d4
ABC
' ' ' D' AB
' ' C' D' ' ' BC
AD ' '
k k k k
AB A' B', AD A' D', BC  B'C', DC  D'C'
PA'.PB' PA'.PD' PB'.PC' PD'.PC'
k k .  PD'2 . A' B'.B'C'  PB2
PB , PD  ABBC
PB' PD' ADDC
. PB'2 A' D'.D'C' PD2

14
2.2. C
2.2.1. C
:Đ h ng minh hai ường th ng vu ng g v i nhau, ta h ng minh nh
a h ng qua ph p ngh h o vu ng g v i nhau

1. ABC ế ò O BH, CK
ABC OAHK

k  AKAB
.  AHAC BK (O) KH
NAk . CH

A ò O) nên AOKH

. ABC ế ò (O1) ò O) qua A, C AB BC


K, N. G i O2 ng tròn ngo i tiếp tam giác KBN. Ch ng minh t giác
BOOO
1 2 là hình bình hành.

15
O2 ò KBN PB/(O)  BKBA
.  BNBC
. k

AK
C N (O) NK
k
N
B  1

BO1 BO1
BO1 (O1)nên BO1  NK 1  KN BO1 //OO2
OO (1)

(BNK) AC
NBk BO BO
 2 2

BO2 (O2) nên BO2  AC 1  ACnên BO2 //OO1


OO (2)

1 2 BOOO
1 2

2.2.2. C ộ ộ
Đ h ng minh A, B, C, D ng thu m t ường tr n, ta s h ng minh
B, C, D nh B’, C’, D’ qua ph p ngh h o ự A th a B’, C’, D’ thu m t ường th ng
h ng i qua ự A

1. ò O AB. G i M m b t k thu ng th ng AB và n m
ng tròn (O) sao cho MA < MB. Vẽ cát tuyến MCD ến (O) sao cho MC < MD, vẽ tiếp
tuyến MP (P là tiế ng tròn (OAC) c ng tròn (OBD) t m th hai là K. Ch ng
minh MKOP n i tiế c trong m ng tròn.

16
2
NOR
AA, B B, C C, DD, M M', P P
(OAC) AC, (OBD) BD
K ò OAC OBD K'  NOR (K)  ACBD
2

AB AB, CD(OCD) nên M'  NOR (M)  AB(OCD)


2

K’AB K' ABC K' B ADnên (OCD) ò


K’AB, mà (OCD) AB  M' K' M' AB

OP2 OM'.OM PM'  ABPK' M'


P, K', M' P, K, M, O ò

2. ò (O1),(O2),(O3),(O4) (O1) ế (O2) A, (O2) ế


(O3) B, (O3) ế (O4) C, (O4) ế (O1) D
A, B,C, D ò

17
NAk, k0
(O1) d1, (O2) d2, (O3) (O3' ), (O4) (O4' )
(O1) ế (O2) A nên d1 / /d2
(O3) ế (O4) nên (O3/ ) ế (O4/ )
(O2) ế (O3) nên (O3/ ) ế d2
18
(O4) ế (O1) nên (O4/ ) ế d1
D'  NAk (D), B'  NAk (B), C' NkA(C)
OB /
4 //OD
3
/
OB
4
/
d1, OD
3
/
d2,d1 //d2 )
B'OC
4 '  D'O3 C' O4 C' B' OC
3 ' D' D',C', B' 
DD', C C', B B' (qua NAk ) (BCD) A(BCD)

. A, B, C d P d
O1,O2,O3 ò ế ABP, BCP, CAP. inh : OOOP
1 2 3

ế ò

NPk , (k0)
AA', BB',CC',O1 O1/ ,O2 O2/ , O3 O3/
AC (A' B'C') (1), (ABP) A' B', (BCP) B'C'
1 P(A' B'C')  ế P A’B’C’

O1/ ,O2/ ,O3/ P qua A’B’, B’C’, A’C’

19
Nên O1/ ,O2/ ,O3/
1 2 3) OO NP1
/ /
(OOO 1 2

P(OOO
1 2 3)

2.2. . Ứ ụ

Cách 1. Đ h ng minh A, B, C th ng h ng, ta h ng minh h ng nh a A’, B’, C’


qua ph p ngh h o ự I v i I thu ường tr n i qua a i m A’, B’, C’
Cách 2. Đ ch ng minh A, B, C th ng hàng, ta ch ng minh qua phép ngh h o cực A,
i m B bi n th nh i m C.

. Cho O, A, A’ A O A’ C ò OA, d
OA A’ ế a A’ C P Q P’
OP d, Q’ d sao cho . '  A' AAO
A' PAQ . ' O, Q, Q’

APP' A' ế
20
NAk' k  PA'/(O)  A' PA
. 'Q A' AA
. 'O

AO
PQ (AA' P) OQ P'(AA' P) nên Q'OQ
T 

P' Q'
O, Q, Q’

. ò O BC A O B0,C0
giao AC, AB O H BB0 CC0 M, N ế
ế ế A ế ò O H, M, N

A0 ế A lên BC

H ABC
NAk k  AB0.AC AC0.AB AM2  AN2
M M, N N, H A0 , m OMAONAOAA
0 90 nên A0 (AMN)
0

H, M, N
ò O K O ế ế
KM, KN ế O M, N ế d1,d2 K (O
A, D B, C G AC BD M, N, G

21
3. ABC O), (I
ò ế ế ò I ế BC, CA, AB
D, E, F H DEF OI.

M, N, P EF, FD, DE
AI EF nên M AI hay A, M, I
B, N, I C, P, I
2
NIr ò I).

IEA vuôn E EM . IE2 r2 .


IMIA
M A
N B .
Suy ra qua N r2
I 

PC
MNPABC.
NIr : K O, suy ra I, K, O
2
K ò ế MNI
I ò ế DEF, K ò ế MNP
ồ ò DEF nên K, I, H
H, I, O

22
Bài 4. (Serbia TST 2009) I ng tròn n i tiếp tam giác ABC không cân, tiếp xúc v i
BC, CA, AB l t t i P, Q, R. QR c t BC t i M. M ò ếp xúc v i (I)
t ng tròn (MNP) c t AP t m th hai là L. Ch ng minh: I, L, M th ng hàng.

Gi i

Xét phép ngh o NPk

BB', CC', M M', AA', I I ', QQ', RR'


BCBC, (I) QR
' ' (/ / BC), AB(PAB
' '), AC(PAC
' ')
ng tròn (QR' ' P) c t BC t i M'; N' là hình chiếu c a M' trên Q' R' ; L' i x ng
v i M' qua Q' R' . Do I ' i x ng v i Pqua Q' R' nên t giác L' I ' PM' là m t hình ch nh t.
G i E là tâm c a hình ch nh t này.

Do ER'  EQ', mà Q' R' tiế ng tròn (PR' B') và (PQC


' ') nên E n m trên tr ng
A' P c ng tròn này.

23
Ta th ng tròn(I ' PM') có tâm là E , do I 'PBC ' ' mà E n ng th ng A' Pnên
ng th ng A' Pvuông góc v ng tròn (I ' PM') .

QuaNPk , ta có : (I ' PM') IM, AP


' AP
' , suy ra AP vuông góc v i IM.
APML nên I, M, L th

2.2. . Ứ ụ

ế ò O A1, B1,C1 ế A, B, C
lên BC, CA, AB H ABC A2, B2,C2
HA, HB, HC BC
1 1,CA
1 1, AB
1 1 da,db,dc A,B, C
BC
2 2,C2A2, AB
2 2 da,db,dc ồ ò
ABC.

24
ẽ da ò ABC

(Oa),(Ob),(Oc) ò ế BHC,CHA, AHB


NAk k  AC1.AB AHAA
. 1  AB1.AC
BC1, B1 CBB1 (ACC1).
H A1 nên AC
1 1 (HAB) B2 B' B’ ò (AAC
1 1)

(HAB)
, C2 C' v i C' mc a 1 1) và (HAC)
(AAB
2 2 (AB'C')
BC
Ia ò ế AB'C' Ia da
2 1  BHBB
B2A1.BC 2 . 2  BB
2 '.B2 A

PB2/(Ia)  PB2/(HBC)
PC2/(Ia) PC2/(HBC)
X, Y (AB'C') (HBC) XY  BC
2 2

25
X X
NAk Y Y

H A
BC (HBC) (ABC ) X,Y(ABC )
N k
A  1 1 1 1 1 1 1

C B1
(ABC
1 1 1),(Ia ),(HBC) ò

(ABC
1 1 1) ò ABC

ế E ò E, Ia,Oa IaE  XY  BC
2 2

2 2 nên Eda
AIa BC
Edb, Edc

. ABC P APB ACB APC ABC D,


E ò ế APB, APC. C AP, BDCE
, ồ

X AP BD, Y AP CE ẽ X Y
XA  BA YA  CA
XP BP YP CP

26
BB'
C C'
N k
A k0 

PP'

ABC AC' B' ABC  AC' B'


APB AB' P' (1) APB  AB' P'
 
APC AC' P' APC  AC' P'

B'C' P'  AC' P'  AC' B'  APCABC  APBACB  AB' P' AB'C' C' B' P'
BC
' ' P' P’ P' B'  P'C' (2)
1 2
XA  BA  P' A  P' A  CA  YA
XP BP P' B' P'C' CP YP
 XA  YA X Y
XP YP

Bài 3. (IMO 1995) 4 m A, B, C, D th ng hàng theo th t ò ng kính


AC, BD c t nhau t i X, Y. Z là giao c ng th ng XY v ng th ng BC, P là m mn m
trên XY, khác Z ng th ng CP c ò ng kính AC t i C và M ng th ng BP c t
ò ng kính BD t i B và N. Ch ng minh r ng XY, DN và AM ồng quy.

Gi i

G ò ng kính AC ng tròn (I ò ng kính BD ng tròn (J).

27
t A'  AP(I), D' DP(J) .
Do P n m trên tr XY c a (I) và (J) nên P ng tròn.
Th c hi n phép ngh o NPk k  PP/(I )  PP/(J) , ta có :

AA', DD', N B, M C, X Y


AM (A'CP), DN (D' BP), XY XY

ng tròn (A’CP) và (D’BP) l m c a BP và CP PAC


' và
PDB
' vuông. Mà PZ l i vuông góc v i AD, suy ra Z n m trê ng tròn (A’CP) và (D’BP).

G i Z’ nh c a Z qua NPk . Do Z n ng tròn (A’CZ) và (D’BZ) nên Z’ a hai

ng th ng AM và DN. M t khác, Z ng th ng XY, suy ra Z'XY. V y ba


ng th ng XY, AM, DN ồng quy t i Z’

2.2. . . Ứ ụ

. ò O PQ, (O’ ế O ế PQ C
mA ò O), B CQ sao cho ABPQ AB ế O’

minh : AC PAB

NCk k0 P P', QQ', AA', PQPQ

28
CAP CP' A' CAPCP' A'
CAB CB' A' CABCB' A'
 
O’ ế PQ C nên (O’ ế ’ ’ // PQ

O ế O’ O)  PQ nên (O ế O’ ế ’
P'Q
AB ế O AB  PQ nên AB ế CA’B’ ế ’
CB’

CA’B’), (O’) CQ’ nên CP' A' CB' A' CAP CAB

. ABC ế AM BD, CE P

DE AM ế AM BC 3 PA PM
2

29
N  AM (ABC)
BEDC ế .  ADAC
AEAB . k
BE (ABC) DEN P ANAP
. k
NAk CD

PM/(ABC)  MBM
. C  MAM
. N
2
BC  BC 3 MN MN  BC
4 2 23
BEDC ế M)
2 2 2
PA/(M)  AM2  BC AEAB . AP(AMMN)  AM2  BC  BC
.  k  ANAP
4 4 2

AP BC 3  1 AM
4 2
P AM.

Bài 3. (Luxembourg, 1980) ng tròn (I), (J) tiếp xúc nhau t i A. T m m M khác
A n m trên (I), k tiếp tuyến v i (I), c t (J) t i B, C. Ch ng minh r ng th ng AM ng
phân giác c a góc t o b ng th ng AB, AC.

Gi i

K AH vuông góc v i MB t i H.

Xét phép ngh o NAAH t


2
(I) a, (J) b, MB(K), MM', BB', CC'
LCC'a
30
Vì (I) tiếp xúc ngoài v i (J) và (I) tiếp xúc v i MB nên a // b và a tiếp xúc v i (K) t i M’

b c t (K) t i B’ C’

H là hình chiếu c a A trên MB nên K n m trên AH.

Do a // b nên hai cung nh M’C’ và M’B’ c a (K) có s y

LM'C' M' AB' do chúng l t ch n hai cung này.

M t khác, LM' AM'C' A nên suy ra hai tam giác LM’C’ và LAM’ ồng d ng.

V y LM'C' M' AL. T

Bài 4. (IMO 2015) Cho tam giác nh n ABC v i AB AC. G iO ng tròn ngo i tiếp
tam giác, H là tr c tâm và D ng cao k t A c a tam giác ABC. G i I m
c a BC. L y m m Q thu ng tròn (O) sao cho HQA900 và l m K thu c (O) sao

cho HKQ900. Gi s m A, B, C, K, Q phân bi t và thu ng tròn (O) theo th t


Ch ng tròn ngo i tiếp tam giác KQH và DKI tiếp xúc v i nhau.
Gi i

31
Xét phép ngh o NHk v i k  HAH
. D ta có: AQ, QI, K F
G i A’, Q’ l i tâm c a A, Q I m c a HA’ nên F là trung
m c a HQ’. T (HKQ) IF, (HDK) (QAF)
Ta có : IF // A'Q'// AQ, AQ'// A'Q nên AQA'Q' là hình ch nh t.
G i J  IF AQ' F m c a IJ nên IF vuông góc v ng trung bình c a
hình ch nh t AJQI IF tiếp xúc v i (QAF

.
Đ t m qu t h i m M, ta t m t ph p ngh h o v t m nh M’ aM
qua ph p ngh h o Khi , n u M’ hạ tr n h nh H’ th ta su ra M hạ tr n h nh H
v i H nh a H’ qua ph p ngh h o

. ò O O’ ế A, BC ế ế
O O’ B(O), C(O') D nh trên (O), M O’
MBC MAD)

32
G iI m th hai c a (MBC)và (MAD)
2
NBBC
(O) (O'), AA', M M', DD', C C
(MBC) CM'
(MAD) (M' A' D')

BAD BD' A' BA' D'  BDA ABCA' D'/ /BC
I ' CM'(M' A' D') I ' M' A' 900 A' I ' M’A’D’)

I ' D' D' A' I ' D'/ /OB


D’ I’ ’ OB
I ò D ế OB B

. ò O Ox, Oy ế ế M trên
(O Ox, Oy A, B O OAB Ox, Oy C,
D I CD.

33
E, F O OAB)  O OAB EF
2
NOR (R = OM
M M, E E, F F, MOMOOx
, Ox,Oy ,(OAB) EF
AOx(OAB), C OxEF A NOOM (C)
2

BOy(OAB), DOyEF B NOOM (D)


2

OAOC
. OM2 MC OA
 
. OM2 MDOB
OBOD
OCMD I OM

O; OM 
I ò  2 

Bài 3. (Đề ề ngh Olympic 30/4/2008) ng tròn (O) tiếp xúc v ng th ng d t m


Hc nh. M, N m trên d th a . N k const , (k 0). T
HMH M, N k các tiếp
tuyến MA, NB t i (O), A, B là các tiế m. Ch ng minh r ng th ng AB m
c nh.

34
Gi i

G iI m c a AB v ng th ng d.

Xét phép ngh ch NHk k HMH


. N
I I ', AA', BB', M N, d d, AB(HAB
' ') v i I ' (HA' B')d
MA(HA' N), MB(HB'M), (O) AB
' '
d tiếp xúc v i (O) nên A’B’ song song v i d.

B’ A’ m chính gi a c a các cung tròn MB'H c ng tròn (MB’H) và

NA' H c ng tròn (NA’H). V y MB’ = HB’; HA’ = NA’, suy ra các tam giác MHB’ và NHA’

l t cân t i B’ và A’ ồng th ng tròn (A’B’H B'H b ng

s AI
' '.
: B'MH  B' HM  HAB
' ' I 'B' A'  AI
' ' N MB'/ /AI
' '
B’I’ // NA’. Mà d // A’B’ nên hai t giác MI’A’B’ và NI’B’A’ u là hình
bình hành. V y MI’ = NI’ = A’B’, nên I’ n MN.

Do M, N c nh nên I’ mc nh, suy ra I mc nh.

35
V ng th ng AB mc nh I là giao c a AB v i d.

3. B
. ò (O1; R1),(O2; R2) ò O
A ò (O1; R1),(O2; R2) .

 ò O
(O1;R1) (O;1 R1)
NAk k  AO12 R12  PA/(O1) (O ; R ) (O/ ; R/ )
 2 2 2 2

O ò A d A

(O) (O1) d (O1)


(O) (O ) d (O/ ) d O1 O2/ d
 2  2

 :
1 ò (O2/ )  NAk (O2) k  AO12 R12
2 d O1,O2/
3 ò (O)  NAk (d)
(O) ò

 :
B3 suy ra (O) A.

d (O1)
1 (O2) (O2/ ) 2 d O1,O2/ nên  / .
d (O2)
d (O1)
d (O )
 2


ế AO
, 1,O2 O1 O2/  d 
ò O .
ế AO
, 1,O2 O1 O2/  d 
ò O

36
d m A, B ò O A, B
ế d.

 T ế d O).
+ Nếu AB//d thì T d AB.
+ ế d AB C
Do d ế ế C ò O)

NCk k CT2 CACB


. 0 d d CT2  k nên T
ò C; k T O

C :
1 C ABd
2 ò NCk k CACB
.
ò CA
a CA B a ò CA D
(C;CD) ò

3 T  d (C;CD)
4 ò O A, B, T O ò

C : Ta có CT2 CD2 CACB


. k nên d ế ế ò ABT).
 ệ
ế AB//d T d AB 1

ế A, Bd
ế dABC C AB d ò C; CD

ế dABC C AB

37
4. ụ

.  O
M  ẽ N OM sao cho OMON
. 1
a) N ò C O
b) Cho A  ẽ ò A
O ò C P O 
hai Q A PQ ò C)

38
. ò O A, B ò M
O N ò C C1 M
ế AB A, B mN

. A, B, C O d
AB ò O A, B d D, E CD, CE
O D’, E’ m D’, E’ khi O d.

. ò C A B C
AB ế ế CD, CD’ ò D, D’
I a DD’

. A, B, C B A C. Qua A ế ế AT
ò C ế AC C, BT C M M.

. ò (O), (O’ A, B. M ò O
ế MA, MB O’ A’, B’

a) ' ' MO


AB
b) A’B’ khi M ò O)

. Cho ò O R P ò I
ỉ I O A, B ò (O1),(O2) I
ế O A B M (O1),(O2) .

. ABC ế ò O M O
MA, MB, MC O m A’, B’, C’

a) M ò O
SABC
' ' '  MA'.MB'.MC'
SABC MAM . BM
. C
b) M A’B’C’ vuông.

. ò I R O OI 2R (O1),(O2)
ò O ế I M
(O1),(O2) M.

39
P ò (O1),(O2) P
ò ế ò

A ò ế d
a

i 12 ò (O1),(O2) A, B ò
C ế AB A (O1),(O2) P, P’ PP’
ò BPP’ ế AB B.

ò O AB  AB H
ò A C ò O  D AC
 C’ AD D’ ’ ’

. Cho N, S ò O ế O S
ò O ế ế N ế ế
OA, OB O’, A’, B’ ế N lên l O, A, B
O’ A’B’

.( Feurbach) ò ABC ế
ò I I ò ế ABC ế ò ế
(Ia),(Ib),(Ic) ABC.

. ò O AB O). P
A’, B’ PA, PB O

a) A’B’
b) ò PA’B’

. ABCD AC, BD
O ò ế OAB OCD X, O
ò ế OAD, OCB Y, O ò AC
Z, T. X, Y, Z, T ò

40
K t Lu n

Tài liệu tham kh o

1 2008 C ph p i n h nh trong m t ph ng
2 1 4 Ph p i n h nh trong m t ph ng
3 200 M ts hu n h nh họ ph ng i ư ng họ sinh gi i

4 2005 Nh ng nh họn ọ trong h nh họ ph ng qua thi


Olympic
5 2014 B i ư ng họ sinh gi i m n To n

[6] Tr n Quang Hùng (2014), Phép ngh h o và ng d ng.

41
CHUYÊN ĐỀ: PHÉP VỊ TỰ QUAY 1 LÝ THUYẾT

1. LÝ THUYẾT
Trong họ hàng các phép biến hình của hình học, có một viên đá quý với một tên gọi rất tinh tế: vị tự
quay. Sự bí ẩn của phép biến hình này, với những hiểu biết và các kỹ thuật một cách sâu sắc, chúng ta
sẽ đưa những bài toán hình học khó về các bài toán đơn giản hơn nhiều.

Như tên gọi, phép vị tự quay bảo toàn "hình dáng" của hình, nhưng sẽ được quay đi một góc.

Định nghĩa 1.1. Cho trước một điểm S, một số dương k, và một góc ϕ (khác 00 và 1800 ). Phép vị tự
quay, tâm S, co dãn k, quay góc ϕ là một phép biến hình biến điểm A thành điểm A′ sao cho:

1. SA′ = k.SA;

2. (SA, SA′) = ϕ .

Phép biến hình như trên ta thường ký hiệu là S(S, k, ϕ ).

S(S, 43 , ϕ )
b
B
b
C
B′
b

b
A′

C′
b
b

A
S

Nếu góc ϕ = 00 hoặc ϕ = 1800 thì phép vị tự quay trở thành phép vị tự. Nếu k = 1, thì nó trở thành
phép quay. Trong trường hợp tổng quát, phép vị tự quay là tích của hai phép biến hình này.

Như ta đã biết, phép vị tự biến một hình thành một hình đồng dạng với nó, và phép vị tự quay thì
sau khi vị tự, nó tiếp tục bởi một phép quay, do đó nó vẫn biến một hình thành một hình mới đồng dạng
với nó. Hơn nữa, hai hình này đồng dạng thuận với nhau. Điều này có nghĩa các điểm tương ứng của
hai hình này sẽ được sắp xếp theo cùng một hướng (ngược chiều hoặc cùng chiều kim đồng hồ).

Bổ đề 1.2. Cho S(S, k, ϕ ) là một phép vị tự quay. Khi đó

1. Ảnh của đường thẳng l là một đường thẳng, ký hiệu là l ′ thì (l, l ′) = ϕ .

2. Ảnh của tam giác ABC là tam giác A′ B′C′ đồng dạng thuận theo tỉ số k. Rõ hơn là
A′ B′ A′C′ B′C′
= = = k.
AB AC BC
Ths Trần Minh Hiền - Trường THPT chuyên Quang Trung 1
CHUYÊN ĐỀ: PHÉP VỊ TỰ QUAY 1 LÝ THUYẾT


(AB, A′B′ ) = (AC, A′C′ ) = (BC, B′C′ ) = ϕ .

3. Ảnh của một đường tròn bán kính R là một đường tròn bán kính k.R.

Chứng minh. Ta chứng minh phần 1, các phần còn lại coi như những bài tập dễ (học sinh tự chứng
minh).
Ảnh của đường thẳng l qua phép vị tự H(S, k) là đường thẳng l2 song song với l. Ảnh của đường thẳng
này qua phép quay lại tiếp tục là một đường thẳng.

P X2
b b

l2

X
b b

l
X′ b

l′ ϕ
S
b

Ký hiệu X2 là hình chiếu của S lên l2 . Do phép quay bảo toàn góc, do đó ảnh X ′ của X2 dưới phép quay
tâm S, góc quay ϕ là hình chiếu của S lên l ′ . Gọi P là giao diểm của l2 và l ′. Khi đó

(l, l ′) = (l2 , l ′) = (PX2 , PX ′ ) = (SX2 , SX ′ ) = ϕ

vì S, X2 , P, X ′ đồng viên. Ý 1 được chứng minh.

Ta sẽ nhận thấy một ứng dụng đầu tiên qua một bài toán quen thuộc dưới đây.

Định lý 1.3 (Đường thẳng Simson). Cho tam giác ABC và X là một điểm nằm trong mặt phẳng. Gọi
P, Q, R lần lượt là hình chiếu của X lên các đường BC,CA, AB. Khi đó ba điểm P, Q, R thẳng hàng nếu
và chỉ nếu X nằm trên đường tròn ω ngoại tiếp tam giác ABC.

Chứng minh. Giả sử X ∈ ω . Nếu X trùng với một trong các đỉnh của tam giác thì ta có ngay kết quả.
Ngoài ra nếu X đối xứng (chẳng hạn với A) qua tâm đường tròn ω thì Q = C, R = B, bài toán hiển
nhiên. Xét trường hợp khác

Ths Trần Minh Hiền - Trường THPT chuyên Quang Trung 2


CHUYÊN ĐỀ: PHÉP VỊ TỰ QUAY 1 LÝ THUYẾT

R
b

X
b

A
b

Q
b

Q′
b

B
b

P b

Xét hai tam giác vuông XPC và XRA. Do ABCX nội tiếp nên

(XA, AB) = (XC,CB),

dẫn đến hai tam giác này đồng dạng thuận. Xét phép vị tự quay, tâm tại X, biến P thành C, ký hiệu là
S(X, k, ϕ ). Khi đó

S(X, k, ϕ ) : P → C
:R→A (vì hai tam giác XPC và XRA đồng dạng thuận)
: PR → AC
: Q → Q′ .

\′ = 900 ,
Vì phép vị tự quay bảo toàn hình dạng, do đó tam giác XQQ′ phải là tam giác vuông, hay XQQ
dẫn đến Q′ thuộc vào AC. Vì ba điểm A, Q′ ,C thẳng hàng nên ba điểm R, Q, P cũng thẳng hàng.
Việc chứng minh chiều ngược lại là tương tự.

Một điều cần ghi nhớ với phép vị tự quay là chúng đi theo cặp. Khi chúng ta thực hiện một phép vị
tự quay, thì luôn có thêm một phép vị tự quay khác kèm với nó.

Mệnh đề 1.4. Giả sử S(S, k, ϕ ) là phép vị tự quay biến A thành A′ và B thành B′ . Khi đó

1. ∆SAB ∼ ∆SA′ B′ ;

2. ∆SAA′ ∼ ∆SBB′ ;

3. Phép vị tự quay S′ (S, k′ , ϕ ′ ) biến A thành B và A′ thành B′ với việc chọn phù hợp k′ và ϕ ′ .

Ths Trần Minh Hiền - Trường THPT chuyên Quang Trung 3


CHUYÊN ĐỀ: PHÉP VỊ TỰ QUAY 1 LÝ THUYẾT

b
B b
B

b
A b
A
b
A′ b

A′
B′
b
b

B′
ϕ ϕ
b b

S S

Chứng minh. 1. Kết quả này là hệ quả do S biến tam giác SAB thành tam giác SA′ B′ .

2. Điều này suy trực tiếp từ định nghĩa phép vị tự quay.

3. Đây chỉ là hệ quả của ý trên.

Chú ý rằng mặc dù hai phép vị tự quay ở trên có cùng tâm S, nhưng chúng không trùng nhau.
Chúng khác nhau bởi hệ số co dãn, cũng như góc quay.
Tính chất này của phép vị tự quay giúp chúng ta chứng minh một định lý rất nổi tiếng của Ptoleme,
định lý cho ta đặc trưng về khoảng cách của tứ gi1c nội tiếp.

Định lý 1.5 (Bất đẳng thức Ptoleme). Cho tứ giác ABCD. Ký hiệu độ dài AB, BC,CA, DA là a, b, c, d
và các đường chéo AC, BD là e, f tương ứng. Khi đó

ac + bd ≥ e f .

Dấu bằng xảy ra khi ABCD nội tiếp.

b
C
c
D b

d b
f

A
b

a
B
d. bc
b

f . ce

Chứng minh. A′ b

Xét phép vị tự quay S, với tâm tại C, biến D → B, và gọi A′ là ảnh của A dưới S. Ta viết lại

S(C, k, ϕ ) : D → B
: A → A′ .

Ths Trần Minh Hiền - Trường THPT chuyên Quang Trung 4


CHUYÊN ĐỀ: PHÉP VỊ TỰ QUAY 1 LÝ THUYẾT

CB b
Do đó hai tam giác ∆CDA ∼ CBA′ (với hệ số co dãn bằng = ), dẫn đến
CD c
b
BA′ = d. .
c
CA e
Vì phép vị tự quay đi theo cặp, do đó ta cũng có ∆CDB ∼ ∆CAA′ (với hệ số co dãn bằng = ), dẫn
CD c
đến
a
AA′ = f . .
c
Áp dụng bất đẳng thức trong tam giác ABA′ suy ra
b e
a + d. ≥ f . ⇒ a.c + d.b ≥ e. f
c c
Dấu bằng xảy ra nếu và chỉ nếu A, B, A′ thẳng hàng, tức

d = 1800 − A[
CBA d
′ BC = 1800 − ADC,

điều này tương đương với tứ giác ABCD nội tiếp.

Bây giờ ta sẽ nghiên cứu, khi nào thì tồn tại phép vị tự quay biến hai điểm cho trước thành hai điểm
cho trước. Câu trả lời được cho dưới định lý sau.

Định lý 1.6. Cho bốn điểm A, B, A′ , B′ nằm trong mặt phẳng, sao cho không có ba điểm nào thẳng
hàng. Giả sử hai đường thẳng AB và A′ B′ cắt nhau tại P. Khi đó tồn tại duy nhất một phép vị tự quay
biến A thành A′ và B thành B′ . Tâm của phép vị tự quay này là giao điểm thứ hai của hai đường tròn
ngoại tiếp AA′ P và BB′ P.

Chứng minh.
Sb

b
B
b
B
b

A
A′
b

P
b

B′
A′
b

B′
b b

b
A S b

Ths Trần Minh Hiền - Trường THPT chuyên Quang Trung 5


CHUYÊN ĐỀ: PHÉP VỊ TỰ QUAY 1 LÝ THUYẾT

Gọi S là tâm của phép vị tự quay S(S, k, ϕ ) mà biến AB thành A′ B′ . Khi đó

(SA, SA′) = (SB, SB′ ) = (AB, A′ B′ ) (bổ đề 26.2 ý 1).

Dẫn đến S thuộc vào cả hai đường tròn ngoại tiếp hai tam giác AA′ P và BB′ P.
Phần còn lại ta sẽ chứng minh hai tam giác SAA′ và SBB′ đồng dạng thuận. Ta đã có

(SA, SA′) = (SB, SB′).

Tuy nhiên điều này được biến đổi

(A′ A, AS) = (A′ P, PS) = (B′ P, PS) = (B′ B, BS).

Định lý được chứng minh.

Nếu hai đường tròn ngoại tiếp hai tam giác AA′ P và BB′ P tiếp xúc với nhau, khi đó phép vị tự quay
quy biến thành phép vị tự tâm P.

Ví dụ 1.1. Cho tam giác ABC nhọn có trực tâm H và tâm đường tròn ngoại tiếp O. Đường trung trực
[
của AH cắt các cạnh AB và AC tại D và E. Chứng minh rằng OA là phân giác của góc DOE.

Dấu hiệu nhận biết: Hai tam giác ADH và AOC đồng dạng.

A b
b

Db b
b
E

b
H
b
O
b

F
Bb

C
Chứng minh.
b

Ta có BAH
d = OAC. d Do đó hai tam giác cân ADH và AOC đồng dạng thuận. Xét phép vị tự quay S
biến D → H, O → C. Khi đó A là tâm quay và giao điểm F của DO với CH có tính chất: AOFC, ADHF
nội tiếp. Vì tứ giác AOFC nội tiếp nên

d = ACF
DOA [
d ≡ ACH.

Chứng minh tương tự cho tam giác AEO và AOB, ta cũng có

[
d = ABH.
EOA

[ = ABH
Mà ACH [ nên DOA [
d hay OA là tia phân giác của góc DOE.
d = EOA,

Ths Trần Minh Hiền - Trường THPT chuyên Quang Trung 6


CHUYÊN ĐỀ: PHÉP VỊ TỰ QUAY 1 LÝ THUYẾT

Kết quả trên không áp dụng cho trường hợp ba trong bốn điểm thẳng hàng. Mệnh đề trên có thể
phát biểu lại để thuận tiện hơn trong việc áp dụng phép vị tự quay liên quan đến hai đường tròn cắt
nhau.

Định lý 1.7. 1. Giả sử hai tam giác SAB, SA′B′ đồng dạng thuận, với các đường tròn ngoại tiếp
ω , ω ′ tương ứng. Khi đó ω , ω ′ và các đường thẳng AA′ , BB′ cùng đi qua một điểm chung.

2. Cho hai đường tròn ω , ω ′ cắt nhau tại P và S. Khi đó trong phép vị tự quay S với tâm S, biến
đường tròn ω thành đường tròn ω ′ , thì điểm A′ ∈ ω ′ là ảnh của A ∈ ω khi và chỉ khi P ∈ AA′ .

b
A′ A′
b

b
B P B
b
P
B′
b
b

B′
b

A
b

A b
b

b
b

S S

Ví dụ 1.2 (CHINA 1992). Cho tứ giác lồi ABCD nội tiếp đường tròn tâm O. Đường chéo AC cắt BD
tại P. Đường tròn ngoại tiếp các tam giác ABP và CDP cắt nhau tại P và Q phân biệt khác O. Chứng
[ = 900 .
minh OQP

Dấu hiệu nhận biết: Đường tròn ngoại tiếp các tam giác ABP và CDP cắt nhau tại P và Q
A
b

B
Q N b

b
b

P
Mb

O b
C

Giải. D
Ta nhận thấy Q là tâm vị tự quay S biến A → C, B → D. Khi đó theo mệnh đề 26.4 ý c thì Q cũng
tâm vị tự quay S′

S(Q, k, ϕ ) : A → B
:C →D
: AC → BD.

Ths Trần Minh Hiền - Trường THPT chuyên Quang Trung 7


CHUYÊN ĐỀ: PHÉP VỊ TỰ QUAY 3 MÔ HÌNH ĐIỂM MIQUEL

Do đó nó biến N thành M (với N là trung điểm AC, M là trung điểm BD.) Vậy ham tam giác QAB và
QNM đồng dạng nên
[ = AQB
NQM d = APB [
d = 1800 − NPM.

Chứng tỏ P, N, Q, M nội tiếp. Vì O, P, N, M nội tiếp đường tròn đường kính OP nên Q ∈ (OP) hay
[ = 900 .
OQP

2. CÁC BÀI TOÁN MỞ ĐẦU


Ví dụ 2.1. Cho tam giác ABC và hai điểm M, N di chuyên trên hai cạnh AB và AC sao cho
BM AN
= .
MA NC
Chứng minh đường tròn ngoại tiếp tam giác AMN luôn đi qua một điểm cố định khác A.

Ví dụ 2.2 (RUSSIAN 2001). Cho tam giác ABC và điểm D nằm trong tam giác. Các điểm E, F nằm
ngoài tam giác sao cho
∆AFB ∼ ∆CEA ∼ ∆CDB.
Chứng minh AEDF là hình bình hành.

Ví dụ 2.3 (NAPOLEON). Cho tam giác ABC. Người ta dựng ra bên ngoài tam giác này các tam
giác đều BCD,CAE, ABF. Chứng minh tam giác A1 B1C1 đều, với A1 , B1 ,C1 lần lượt là tâm đường tròn
ngoại tiếp các tam giác BCD,CAE, ABF.

Ví dụ 2.4 (APMO 1998). Cho tam giác ABC và D là chân đường cao hạ từ đỉnh A. Gọi E, F là hai
điểm nằm trên đường thẳng đi qua D sao cho

d = ACF
AEB d = 900 .

[ = 900 .
Gọi M, N lần lượt là trung điểm của BC và EF. Chứng minh ANM

3. MÔ HÌNH ĐIỂM MIQUEL


Định lý dưới đây có thể chứng minh bằng cách liên kết góc, nhưng với kết quả của phép vị tự quay, ta
đưa ngay một chứng minh hiển nhiên cho nó.

Mệnh đề 3.1 (Điểm Miquel cho tứ giác). Cho tứ giác ABCD. Các tia BC và AD cắt nhau tại Q, và
các tia BA và CD cắt nhau tại R. Khi đó bốn đường tròn (RAD), (DBC), (QAB), (QCD) cùng đi qua
một điểm M. Điểm M gọi là điểm Miquel của tứ giác ABCD.

Ths Trần Minh Hiền - Trường THPT chuyên Quang Trung 8


CHUYÊN ĐỀ: PHÉP VỊ TỰ QUAY 3 MÔ HÌNH ĐIỂM MIQUEL

b
Q

M b

b
C
b

R b

A b

Chứng minh.
Theo định lý 26.6 thì giao điểm thứ hai M(M 6= R) của (RAD) và (RBC) là tâm phép vị tự quay
biến A → D, B → C. Theo mệnh đề 26.4.3 thì nó cũng là tâm của phép vị tự quay biến A → B, D → C.
Do đó lại áp dụng định lý 26.6 thì M lại nằm trên (QAB), (QCD). Ta có điều phải chứng minh.

Ví dụ 3.1 (USAMO 2006). Cho tứ giác ABCD có các cặp cạnh đối không song song và E, F lần lượt
là các điểm trên cạnh AD và BC tương ứng sao cho
AE BF
= .
ED FC
Tia FE cắt tia BA và CD tại S, T . Chứng minh rằng các đường tròn (SAE), (SBF), (TCF), (T DE)
cùng đi qua một điểm.

Dấu hiệu nhận biết: Có một tỉ số và mô hình tứ giác toàn phần.

C
b

M
b

D
b

b
F

E
b

Tb

S b

B
A
b
b

Chứng minh.
Gọi M là tâm phép vị tự quay S biến A → B, D → C. Khi đó AD → BC và vì các điểm E, F chia
các đoạn thẳng này theo cùng tỉ số, nên nó biến E → F. Do đó S biến AE → BF và ED → FC. Vì vậy
M là điểm Miquel chung của các tứ giác ABFE, EFCD. Theo mệnh đề 26.8 thì M nằm trên tất cả các
đường tròn đề bài yêu cầu.

Bài toán sau đây là hệ quả của bài toán trên.

Ths Trần Minh Hiền - Trường THPT chuyên Quang Trung 9


CHUYÊN ĐỀ: PHÉP VỊ TỰ QUAY 3 MÔ HÌNH ĐIỂM MIQUEL

Ví dụ 3.2 (VIETNAM TST 2013). Cho tứ giác ABCD có các cạnh đối không song song nội tiếp đường
tròn (O; R). Gọi E là giao điểm hai đường chéo và đường phân giác góc AEBd cắt các đường thẳng
AB, BC,CD, DA lần lượt tại các điểm M, N, P, Q. Chứng minh rằng các đường tròn (AQM), (BMN), (CNP), (DPQ
cùng đi qua một điểm.

Thật kỳ lạ, ý tưởng trên tiếp tục được khai thác trong một bài toán thi IMO 2005, khác chăng là thứ
tự các đỉnh qua phép vị tự quay.

Ví dụ 3.3 (IMO 2005). Cho ABCD là tứ giác lồi với AD = BC và BC, AD không song song. Gọi E, F
là các điểm trên cạnh BC, AD tương ứng sao cho BE = DF. Đường thẳng AC và BD cắt nhau tại P,
đường thẳng BD và EF cắt nhau tại Q, đường thẳng EF và AC cắt nhau tại R. Chứng minh rằng đường
tròn ngoại tiếp tam giác PQR đi qua một điểm cố định khác P.
D
b

b
C

P
b

F b

R
b

Q
b

b
E

S
b b
B
A

Tóm tắt cách giải.


Xét phép vị tự quay

S(S, k, ϕ ) : D → B
:A→C
:F →E (do DF
AF = BE
EC theo giả thiết.)

Theo định lý 26.6 thì điểm S là giao của hai đường tròn (DPA) và (BPC). Xét tứ giác toàn phần
DFRPAQ, thì theo mệnh đề 26.8, điểm S cũng thuộc đường tròn (PQR). Vì (DPA) và (BPC) cố định
nên S cố định. Bài toán được chứng minh.

Ý tưởng này tiếp tục được khai thác trong một bài toán thi quốc tế nữa, đó là IMO 2007, nhưng
việc sử dụng tinh tế hơn.

Ví dụ 3.4 (IMO 2007). Cho hình bình hành ABCD. Gọi l là một đường thẳng đi qua A, cắt cạnh BC
và đường thẳng DC tại F, G. Gọi E là tâm đường tròn ngoại tiếp tam giác CFG. Chứng minh rằng nếu
d
DCED nội tiếp thì l là phân giác DAB.
BF AB DC
Dấu hiệu nhận biết: Có tỉ số = = .
FC CG CG
Ths Trần Minh Hiền - Trường THPT chuyên Quang Trung 10
CHUYÊN ĐỀ: PHÉP VỊ TỰ QUAY 3 MÔ HÌNH ĐIỂM MIQUEL

b
E
b
G

b
J
Bb b
I b b

F C

b b

Chứng minh. A D
BF AB DC
Ta có = = , nên xét phép vị tự quay
FC CG CG
S(O, k, ϕ ) : B → D
:C →G
: BC → DG
BF DC
:F →C (do = ).
FC CG
: FC → CG
:I→J (với I và J là trung điểm FC và CG.)

Ta có BI cắt DJ tại C. Theo định lý 26.6 thì tâm của phép vị tự quay trên là giao điểm thứ hai của
đường tròn (BDC) và đường tròn (CIJ). Nhưng hai đường tròn này cùng đi qua E (E ∈ (CIJ) vì CE
chính là đường kính của đường tròn này). Do đó O ≡ E. Theo tính chất vị tự quay thì ∆EBD ∼ ∆EFC,
mà tam giác EFC cân tại E (EF = EC), nên tam giác EBD cân tại E.

[ =E
ECG [BD (tứ giác ECDB nội tiếp)
=E[DB (tam giác EBD cân tại E)
d
= ECB.

Do đó hai tam giác cân ECF và ECG bằng nhau, nên CF = CG, tức tam giác CFG cân tại C. Suy ra

[ = CGF.
CFG [

[ = DAF
Mà CFG [ = BAF
d (đồng vị) và CGF d (so le trong) nên

d = DAF
BAF d

hay AF là phân giác của góc BAD.


d

Ths Trần Minh Hiền - Trường THPT chuyên Quang Trung 11


CHUYÊN ĐỀ: PHÉP VỊ TỰ QUAY 4 CÁC BÀI TOÁN OLYMPIC

Chúng ta đã nghiên cứu qua các tính chất của phép vị tự. Vậy thì khi nào dùng phép vị tự quay.
Bất cứ khi nào ta thấy hai đường tròn cắt nhau cùng với một số đường thẳng đi qua một trong các
giao điểm, khi đó một phép vị tự quay có thể xem xét. Và ngược lại, các đường thẳng nối các điểm
tương ứng trong phép vị tự quay thông thường sẽ đi qua một giao điểm của hai đường tròn.

4. CÁC BÀI TOÁN OLYMPIC


Ví dụ 4.1 (IMO SHORTLIST 2006). Cho ngũ giác lồi ABCDE sao cho
d = CAD
BAC d = DAE,
d d = DCA
CBA d = EDA.
d

Gọi P là giao điểm của BD và CE. Chứng minh rằng đường thẳng AP đi qua trung điểm của CD.

Dấu hiệu nhận biết: Hai tam giác ABC và ADE đồng dạng, P là giao điểm của BD và CE.

A
b

B b

b
E
b

C
b

D
Chứng minh.
Ta có hai tam giác ABC và ADE đồng dạng với nhau, nên tồn tại phép vị tự quay biến tam giác
ABC thành tam giác ADE.

1. Theo định lý 26.7.1 suy ra P là giao điểm thứ hai của hai đường tròn (ABC) và (ADE).

2. Theo giả thiết thì ba tam giác ABC, ACD, ADE đồng dạng nên
d = DCA,
CBA d [
d = AED.
ADC

Suy ra CD là tiếp tuyến chung của hai đường tròn (ABC) và (ADE).

3. Vì AP là trục đẳng phương của (ABC) và (ADE) nên AP đi qua trung điểm của CD.

Ví dụ 4.2 (USA TST 2007). Hai đường tròn (O1 ) và (O2 ) cắt nhau tại P và Q. AC, BD tương ứng là
những dây cung của (O1 ) và (O2 ) sao cho đoạn thẳng AB và tia CD cắt nhau tại P. Tia BD cắt đoạn
AC tại X. Điểm Y nằm trên (O1) sao cho PY k BD. Điểm Z nằm trên (O2) sao cho PZ k AC. Chứng
minh rằng các điểm Q, X,Y, Z cùng nằm trên một đường thẳng.

Ths Trần Minh Hiền - Trường THPT chuyên Quang Trung 12


CHUYÊN ĐỀ: PHÉP VỊ TỰ QUAY 4 CÁC BÀI TOÁN OLYMPIC

Dấu hiệu nhận biết: Các dây cung CD, AB đi qua một giao điểm P của hai đường tròn.

B
P
b

A
b
b

Y′
b

b
D
b
b

Xb

Q
b

Chứng minh. Z′ Theo định lý 26.7.2 thì do p ∈ CD, P ∈


b

AB nên xét phép vị tự quay tâm Q biến D → C, B → A. Theo định lý 26.6 thì Q lại nằm trên đường tròn
(XPC). Gọi Y ′ , Z ′ lần lượt là giao điểm của XQ với (O1 ), (O2). Ta chứng minh Y ′ ≡ Y, Z ′ ≡ Z. Thật vậy

[ = DCQ
DXQ [
[ = PY ′ Q ⇒ PY ′ k DB ⇒ Y ′ ≡ Y ;

[′ = DQX
DPZ [ = DCA
d ⇒ PZ ′ k QC ⇒ Z ≡ Z ′ .

Ví dụ 4.3 (IMO SHORTLIST 2006). Trên các cạnh BC,CA, AB của tam giác ABC lấy các điểm
A1 , B1 ,C1 . Đường tròn ngoại tiếp các tam giác AB1C1 , BC1 A1 và BC1A1 cắt đường tròn ngoại tiếp tam
giác ABC tại điểm thứ hai A2 , B2 ,C2 . Gọi A3 , B3 ,C3 lần lượt là các điểm đối xứng với A1 , B1,C1 qua
trung điểm của BC,CA, AB. Chứng minh rằng các tam giác A2 B2C2 và A3 B3C3 đồng dạng.

Dấu hiện nhận biết: Hai đường tròn (AC1 B1 ) và (ABC) cắt nhau tại A2 .

b
A C2
b

A2
B1
b

C3 b

C1
B3
b

B
b b

A3
b b

A1 C
b

B2

Chứng minh.

Ths Trần Minh Hiền - Trường THPT chuyên Quang Trung 13


CHUYÊN ĐỀ: PHÉP VỊ TỰ QUAY 4 CÁC BÀI TOÁN OLYMPIC

Ta có BC1 và CB1 cắt nhau tại A. Hai đường tròn (AC1 B1 ) và (ABC) cắt nhau tại A2 . Do đó theo
định lý 26.6 thì

S(A2 , k, ϕ ) : C1 → B1
: B → C.

Do đó hai tam giác ∆A2C1 B ∼ ∆A2 B1C. Dẫn đến


A2C1 C1 B
= .
A2 B1 B1C

Do tính chất đối xứng mà C1 B = AC3 , B1C = AB3 nên


A2C1 AC3
= .
A2 B1 AB3

Ngoài ra C\1 A2 B1 = C\3 AB3 nên hai tam giác ∆A2C1 B1 ∼ ∆AC3 B3 . Lại theo tính chất "theo cặp" của
phép vị tự quay, ta cũng có ∆A2C1 B1 ∼ ∆A2 BC. Từ đây suy ra

∆A2 BC ∼ ∆AC3 B3 ⇒ A[ \
2 BC = AC3 B3 .

Tương tự thì
\
BC [
3 A3 = B [
2 AC, B [
2 AC = B \
2 BC ⇒ BC [
3 A3 = B 2 BC.

Từ đây dẫn đến

B\ 0 \
3C3 A3 = 180 − AC \
3 B3 − BC
0 [
3 A3 = 180 − A [
2 BC − B
0 \
2 BC = 180 − B 2 AA2 = A\
2C2 B2 .

Chứng minh tương tự ta cũng có C\


3 B3 A3 = A\
2 B2C2 . Chứng tỏ hai tam giác A2 B2C2 và A3 B3C3 đồng
dạng.

Ví dụ 4.4 (PTNK TST 2013). Tam giác nhọn ABC có H là trực tâm và P là điểm di động bên trong
[ Đường thẳng qua B và vuông góc với AB cắt PC tại M, đường
d = BHC.
tam giác ABC sao cho BPC
thẳng qua C và vuông góc với AC cắt PB tại N. Chứng minh trung điểm I của MN luôn thuộc một
đường thẳng cố định.

Dấu hiệu nhận biết: Các đường thẳng cùng đi qua điểm P là giao điểm của hai đường tròn (BHPC)
và (PNM).

Ths Trần Minh Hiền - Trường THPT chuyên Quang Trung 14


CHUYÊN ĐỀ: PHÉP VỊ TỰ QUAY 4 CÁC BÀI TOÁN OLYMPIC

A
b

b N

P
b

H b

I
Bb

T
b

C
Db

E M
b

Giải.

• Với giả thiết bài toán thì P ∈ (BHC). Vì các đường thẳng MC và NB cùng đi qua giao điểm P
của hai đường thẳng nên tồn tại phép vị tự quay biến M → C, N → B. Theo tính chất "đi theo
cặp", thì cũng có phép vị tự quay biến M → N,C → B. Theo định lý 26.6 thì tâm vị tự quay
trên là điểm E (giao điểm thứ hai của (BHPC) và (PNM).). Vậy đó phép vị tự quay này biến
∆EMN → ∆ECB. Do đó trung điểm I của MN biến thành trung điểm T của BC.

• Gọi D là giao điểm của hai đường thẳng vuông góc trong đề bài thì H, T, D thẳng hàng và
D ∈ (ABC).

• Ta có
d + BPC
NPC d = 1800 ⇒ NPC [ = 1800 .
d + BHC
[ = 1800 , dẫn đến NPC
Vì A + BHC [ = A. Từ đó suy ra
d = A. Theo tính chất góc ngoài thì NDM
[ = NPM
NDM [ hay D ∈ (PMN).

• Ta chứng minh E cố định. Thật vậy

(ED, EP) = (ND, NP) = (BH, BP)(so le trong) = (EH, EP).

Do đó E thuộc đoạn HD cố định. Vì E là giao điểm của HD (cố định) và (BHC) (cố định) nên
E cố định.

• Theo tính chất phép vị tự quay thì


EM EC
(EM, EI) = (EC, ET ) = α (cố định), = = k = const.
EI ET
Do đó phép vị tự quay
S (M, k, ϕ ) : M → I.
Vì M thuộc đường thẳng BD cố định nên I thuộc đường thẳng cố định.

Ths Trần Minh Hiền - Trường THPT chuyên Quang Trung 15


CHUYÊN ĐỀ: PHÉP VỊ TỰ QUAY 4 CÁC BÀI TOÁN OLYMPIC

Ví dụ 4.5 (VIETNAM TST 2009). Cho tam giác nhọn ABC nội tiếp đường tròn (O). Gọi A1 , B1 ,C1
và A2 , B2 ,C2 lần lượt là chân đường cao của tam giác ABC hạ từ các đỉnh A, B,C và các điểm đối xứng
với A1 , B1 ,C1 qua trung điểm các cạnh BC,CA, AB. Gọi A3 , B3 ,C3 lần lượt là các giao điểm của các
đường tròn ngoại tiếp các tam giác AB2C2 , BA2C2 ,CA2 B2 với (O). Chứng minh rằng A1 A3 , B1 B3 ,C1C3
đồng quy.
Dấu hiện nhận diện: Giả thiết tương tự bài 26.5

A b A3
b

C2 b

b
B1
b
C3
b

b
B2
b

C1
b

A1
b

B
b

C
b
b

A2

B3
b

Giải.
Cách chứng minh bài này tương tự như bài 26.5. Vì BC2 cắt CB2 tại A và A3 là giao điểm thứ hai
của (AC1 B2 ) với (ABC) nên

S(A3 , k, ϕ ) : B → C
C2 → B2 .

Do đó tam giác ∆A3 B2C2 ∼ ∆A3CB. Và ta cũng có ∆A3 BC2 ∼ ∆A3CB2 nên
A3 B BC2 A3 B AC1
= , BC2 = AC1 ,CB2 = AB2 (tính đối xứng) ⇒ = .
A3C CB2 A3C AB1
Do đó ∆AB1C1 ∼ ∆A3CB, dẫn đến
\
AB [
1C1 = A 3CB.
\
Vì BCB1C1 nội tiếp nên AB 1C1 = ABC. Vì vậy
d

d = A[
ABC 3CB.

Tứ giác AA3CB nội tiếp có ABCd = A[ 3CB nên là hình thang cân. A1 là hình chiếu của A trên BC, theo
tính chất hình thang cân thì A3 A1 đi qua trọng tâm G của tam giác ABC. Tương tự cho B3 B1 ,C3C1 cũng
đi qua G. Ta có điều phải chứng minh.

Ths Trần Minh Hiền - Trường THPT chuyên Quang Trung 16


CHUYÊN ĐỀ: PHÉP VỊ TỰ QUAY 4 CÁC BÀI TOÁN OLYMPIC

Ví dụ 4.6 (IMO SHORTLIST 2012). Cho tam giác nhọn ABC và các điểm D, E, F lần lượt là chân
đường cao hạ từ các đỉnh A, B,C tương ứng. Gọi I1 , I2 lần lượt là tâm đường tròn nội tiếp các tam giác
AEF, BDF, và O1 , O2 lần lượt là tâm đường tròn ngoại tiếp các tam giác ACI1 và BCI2 . Chứng minh
I1 I2 k O1 O2 .

Dấu hiệu nhận biết: Hai tam giác chung đỉnh FEA và FBD đồng dạng.

b
O1
A b

I1
b

b
E

F b

I3
b

I2
b

C
B
b

O2 b

Giải.
Dễ dàng có ∆FEA ∼ FBD. Do đó

S(F, k, ϕ ) : E → B
:A→D
: I1 → I2 .

Dẫn đến ∆FEB ∼ FI1 I2 . Ta có

[
AI [
1 I2 = AI [
1 F + FI 1 I2
d
AEF
= 900 + [
+ FI1 I2 (do I1 là tâm nội tiếp AFE)
2
B d d = B và FI [1 I2 = FEB do tam giác FI1 I2 và FEB đồng dạng.)
= 900 + + FEB (AEF d
2
B d (do BE ⊥ AC)
= 900 + + 900 − FEA
2
B B d2 .
= 1800 + − B = 1800 − = 1800 − ABI
2 2

Ths Trần Minh Hiền - Trường THPT chuyên Quang Trung 17


CHUYÊN ĐỀ: PHÉP VỊ TỰ QUAY 4 CÁC BÀI TOÁN OLYMPIC

Do đó AI1 I2 B nội tiếp. Tương tự nếu ta gọi I3 là tâm nội tiếp tam giác CDE thì ta cũng chứng minh
được AI1 I3C, BI2 I3C nội tiếp. Hay I3 ∈ (O1), I3 ∈ (O2). Do đó CI3 là trục đẳng phương của (O1 ), (O2 )
nên
CI3 ⊥ O1 O2 .
Dễ thấy AI1 , BI2 ,CI3 đồng quy tại tâm nội tiếp I của tam giác ABC, và dễ dàng chứng minh

CI3 ⊥ I1 I2 .

Từ đó ta được O1 O2 k I1 I2 .

Ví dụ 4.7 (USA 2013). Trên các cạnh BC,CA, AB của tam giác ABC lấy các điểm P, Q, R. Đường tròn
ngoại tiếp các tam giác AQR, BRB và CPQ cắt AP tại điểm thứ hai X,Y, Z tương ứng. Chứng minh
YX BP
rằng = .
XZ PC
Dấu hiệu nhận biết:Hai đường thẳng BC và Y Z đi qua P là giao điểm của hai đường tròn.

A
b

R b

b
Y
Q
b

X
b b

Z b

B
b
b

P
b
C

Chứng minh.
Ba đường tròn (ARQ), (BPR), (CPQ) cắt nhau tại điểm O (Định lý Miquel). Ta có O là tâm vị tự
quay

S(O, k, ϕ ) : C → B
: Z → Y.

XY PB
Do đó hai tam giác ∆OCB ∼ OZY . Do đó để có kết luận = của bài toán ta chỉ cần chứng minh
XZ PC
∆OXY ∼ ∆OPB. Thật vậy

[ ≡ OXA
OXY d
[ (tứ giác AXIQ nội tiếp)
= OQC
d (tứ giác OQCP nội tiếp)
= OPB

Ths Trần Minh Hiền - Trường THPT chuyên Quang Trung 18


CHUYÊN ĐỀ: PHÉP VỊ TỰ QUAY 4 CÁC BÀI TOÁN OLYMPIC

[
và OY d (do hai tam giác ∆OXY ∼ ∆OPB). Vậy hai tam giác ∆OXY ∼ ∆OPB. Ta có điều phải
X = OBP
chứng minh.

Ví dụ 4.8 (USA TST 2006). Cho tam giác nhọn ABC với các đường cao AD, BE,CF và H là trực tâm
của tam giác. Đường tròn tâm O qua A, H cắt AB, AC một lần nữa tại các điểm Q và P tương ứng.
CR ED
Đường tròn ngoại tiếp tam giác OQP tiếp xúc với BC tại R. Chứng minh rằng = .
BR FD
Dấu hiệu nhận biết: Hai đường tròn (AFHE) và (AQHP) cắt nhau tại A.
A b

Q M O b
b

Fb

Eb

b
b

P
H

Bb

R D
b
b

C
Chứng minh.
Ta có hai đường tròn (HEF) và (HQP) cắt nhau tại A nên

S(A, k, ϕ ) : Q → F
:P→E
: H → H.

[ = 2BAC.
Vì (AFHE), tâm M trung điểm AH nên FME [ = 2BAC.
d Vì (AQHP) tâm O nên QOP d Do đó
[ Do đó hai tam giác cân FME và QOP đồng dạng. Do đó
[ = QOP.
FME

S(A, k, ϕ ) : M → O.

\ = 900 nên HDR


Gọi S(A, k, ϕ ) : D → R′ . Khi đó tam giác ∆HMO ∼ ∆HDR′ . Vì OMH \′ = 900 hay
R′ ∈ BC. Ngoài ra bốn điểm F, M, E, D thuộc một đường tròn (đường tròn Euler) nên Q, O, P, R′ cũng
cùng thuộc một đường tròn. Dẫn đến R ≡ R′ . Vì ba điểm M, H, D thẳng hàng nên O, H, R cũng thẳng
hàng. Ta có

d = PQR
CRQ d (cùng bằng một nửa số đo cung RP)
[
= EFD (do tính chất phép vị tự quay)
d − BFR
= 1800 − AFE d = 2(900 −C).

Ths Trần Minh Hiền - Trường THPT chuyên Quang Trung 19


CHUYÊN ĐỀ: PHÉP VỊ TỰ QUAY 4 CÁC BÀI TOÁN OLYMPIC

Dẫn đến tam giác CRP cân tại R nên CR = RP. Tương tự ta có BR = RQ. Mà ∆QPR ∼ ∆FED (do tính
chất phép vị tự quay) nên
ED PR CR
= = .
FD QR BR

Kết thúc chuyên đề này, ta sẽ giải một bài toán, với quá trình suy ngược (rất có thể đây là con
đường tác giả tạo ra bài toán đó). Và sự tiềm ẩn của phép biến hình cho thấy mức độ khó của bài toán.
BÀI TOÁN KẾT THÚC: Cho tam giác nhọn ABC, trực tâm H, trung điểm D của BC. E là hình chiếu
của A trên BC. Như ta đã biết H ′ , đối xứng của H qua BC, nằm trên (ABC).
B
b

H′ Eb

M′
b

X b

Q I
b

A
b

T
N′
b
b

D O
b

b S

D′
b

1. Đường tròn (DEH ′ ), đường kính DH ′ , cắt BH ′ ,CH ′ tại M ′ , N ′ . Gọi Q là giao điểm thứ hai của
(DEH ′ ) và (ABC).

2. Khi đó

S(Q, k, ϕ ) : M ′ → B
: N′ → C
:E →A
: D → D′ (D, H ′ , D′ thẳng hàng, do đó D, H ′ , D′ , I thẳng hàng)
:I→O (I là tâm ngoại tiếp (DEH ′ ) và O là tâm ngoại tiếp (ABC)
hai tiếp tuyến tại M ′ , N ′ của (I) → hai tiếp tuyến tại B,C của (O)).
:T →X (T là giao của hai tiếp tuyến tại M ′ , N ′ của (I) và
O là giao của hai tiếp tuyến tại B,C của (O)).

Ths Trần Minh Hiền - Trường THPT chuyên Quang Trung 20


CHUYÊN ĐỀ: PHÉP VỊ TỰ QUAY 5 10 BÀI TẬP LUYỆN TẬP

3. Gọi S là giao của XD′ với (ABC). Tứ giác điều hòa D′ BSC (vì hai tiếp tuyến tại B,C cắt nhau tại
X trên D′ S). Theo tính chất của tứ giác điều hòa thì D′ X là đường đối trung. Do D′ D′ là đường
[
trung tuyến nên XD [
′C = BD ′ D, hay CS = BH ′ .

4. Dẫn đến ∆XSC = ∆XH ′ B nên XH ′ = XS, do đó H ′ , S đối xứng nhau qua XO, dẫn đến AS là
đường kính của (ABC).

5. Ta có S(Q, k, ϕ ) : T[ [
DE → XD d = 900 (do AS là đường kính). Nên T[
′ A = SDA DE = 900 . Chứng
tỏ X, T, D, O thẳng hàng.

6. Ta có S(Q, k, ϕ ) : IT
d \′ = OXS
D → OXD d = OXH\′ , mà hai góc IT d \′ ở vị trí đồng vị nên
D và OXH
IT k XH ′ . Vậy IT là đường trung bình của tam giác DH ′ X nên T là trung điểm của DX.

7. Nếu B,C cố định, đường tròn (O) cố định thì D, X cố định, nên T cũng cố định theo. Do đó bài
toán trên cho A thay đổi và ẩn đi đường tròn (DEH ′ ) bởi đường tròn đối xứng với nó qua BC
thì ta có bài toán khó (được đánh giá là bài khó nhất trong lịch sử TST. Việc thực hiện chi tiết
dành cho học sinh.)

Ví dụ 4.9 (VIETNAM TST 2009). Trên mặt phẳng, cho đường tròn (O) và hai điểm cố định B,C trên
đường tròn này sao cho BC không là đường kính của (O). Gọi A là một điểm di động trên đường tròn
(O) và A không trùng với hai điểm B,C. Gọi D, K, J lần luợt là trung điểm của BC,CA, AB và E, M, N
lần luợt là hình chiếu vuông góc của A, B,C trên BC, DJ, DK. Chứng minh rằng các tiếp tuyến tại M, N
của đường tròn ngoại tiếp tam giác EMN luôn cắt nhau tại diểm T cố dịnh khi điểm A thay đổi trên
(O).

5. 10 BÀI TẬP LUYỆN TẬP


Bài 5.1 (ROMANIAN TST 1999). Hai đường tròn (O1 ) và (O2) cắt nhau tại A và B. Một đường thẳng
l đi qua A, cắt lại hai đường tròn (O1 ) và (O2 ) tại C và D tương ứng. Gọi M, N lần lượt là trung điểm
của các cung BC, BD của hai đường tròn (O1 ) và (O2 ) (cung không chứa A). Gọi K là trung điểm CD.
Chứng minh MKN[ = 900 .

Ths Trần Minh Hiền - Trường THPT chuyên Quang Trung 21


CHUYÊN ĐỀ: PHÉP VỊ TỰ QUAY 5 10 BÀI TẬP LUYỆN TẬP

b
D

Ab

K
b

b O2
b

O1 b

N
Cb

B b

M
b
D′
b

Chứng minh. C′
Gọi C′ , D′ là các điểm đối xứng của C, D qua M, N tương ứng. Khi đó

CM = BM (giả thiết) = C′ M.

Do đó tam giác BCC′ vuông tại B. Tương tự tam giác BDD′ vuông tại D. Ta có

[ + BAC
BMC d = 1800 , [
B d = 1800 , BAC
ND + BAD d + BAD
d = 1800

dẫn đến
[ +B
BMC [ [
ND = 1800 ⇒ 2(BC [
′C + BD ′ D) = 1800

[
⇒ BC [
′C + BD [
′ D = 900 ⇒ BC [′ .
′C = BDD

Do đó ∆BC′C ∼ ∆BDD′ . Từ đó tồn tại phép vị tự quay

S(B, k, 900) : C → C′
: D′ → D.

Dẫn đến CD′ ⊥ C′ D. Mà MK, NK là đường trung bình của tam giác AC′ D, DCD′ nên MK k C′ D, NK k
CD′ . Từ đó ta được MK ⊥ NK.

Bài 5.2 (BANKAL 2009). Cho tam giác ABC. M, N là hai điểm trên AB, AC sao cho MN song song
với BC. BN cắt CM tại P. Đường tròn ngoại tiếp các tam giác BMP và CNP cắt nhau tại P và Q. Chứng
minh rằng BAQ
d = PAC.
d

Bài 5.3 (CHINA TST 2010). Cho tam giác nhọn ABC, AB > AC có I là tâm đường tròn nội tiếp tam
giác. Gọi M, N lần lượt là trung điểm của AC, AB. Hai điểm D, E lần lượt nằm trên hai cung AC và AB
sao cho BD k IM,CE k IN. Đường thẳng qua I song song với DE cắt BC tại P. Gọi Q là hình chiếu của
P trên AI. Chứng minh Q nằm trên đường tròn ngoại tiếp tam giác ABC.

Ths Trần Minh Hiền - Trường THPT chuyên Quang Trung 22


CHUYÊN ĐỀ: PHÉP VỊ TỰ QUAY 5 10 BÀI TẬP LUYỆN TẬP

Bài 5.4 (BA LAN 1999). Cho tam giác nhọn ABC có các đỉnh lần lượt nằm trên các cạnh của tam giác
d = A\
A1 B1C1 và ABC 1 B1C1 ; ACB = A\
d 1C1 B1 ; CAB = C\
d 1 A1 B1 . Chứng minh rằng tâm đường tròn ngoại
tiếp tam giác ABC cách đều trực tâm hai tam giác ABC và A1 B1C1 .

Bài 5.5 (IMO 2004). Cho tứ giác lồi ABCD, đường chéo BD không là phân giác các góc ABC,
d CDA.
d
Điểm P nằm trong tứ giác ABCD sao cho PBC
d = DBA;
d PDC d Chứng minh rằng ABCD nội tiếp
d = BDA.
được nếu và chỉ nếu AP = CP.

Bài 5.6 (IRAN TST 2010). Hai đường tròn (O1 ) và (O2 ) cắt nhau tại P và K. XY là tiếp tuyến chung
gần P của hai đường tròn (X thuộc (O1 ) và Y thuộc (O2 )). XP cắt (O2 ) lần thứ hai tại C; Y P cắt (O1 )
lần thứ hai tại B. A là giao điểm của BX và CY . Gọi Q là giao điểm thứ hai của hai đường tròn ngoại
tiếp các tam giác ABC và AXY . Chứng minh rằng QXA[ = QKP.[

Bài 5.7. Cho hai đường tròn (O1 ) và (O2 ) cắt nhau tại A và B. Một cát tuyến thay đổi qua A cắt
(O1 ), (O2 ) tại D và E. Tiếp tuyến của (O1 ) tại D và tiếp tuyến của (O2 ) tại E cắt nhau tại P. Chứng
minh đường trung trực của BP luôn tiếp xúc với một đường tròn cố định.

Bài 5.8 (CANADA 2013). Cho tam giác vuông ABC với C = 900 và G là trọng tâm tam giác. P là điểm
trên tia AG sao cho CPA
d = CAB d và Q là điểm trên tia BG sao cho CQB
d = ABC.d Chứng minh đường
tròn ngoại tiếp của hai tam giác AQG và BPG cắt nhau tại một điểm trên cạnh AB.

Bài 5.9 (CANADA 2013). Cho tam giác nhọn ABC nội tiếp đường tròn (O). Trên cạnh AB lấy điểm
P sao cho BOP
d = ABC.d Q là điểm trên cạnh AC sao cho COQ [ = ACB. d Chứng minh rằng đường thẳng
đối xứng với BC qua PQ tiếp xúc với đường tròn ngoại tiếp tam giác APQ.

Bài 5.10 (VIETNAM TST 2009). Cho đường tròn (O) có đường kính AB và M là một điểm bất kì
nằm trong (O), M không nằm trên AB. Gọi N là giao điểm của phân giác trong góc M của tam giác
AMB với đường tròn (O). Đường phân giác ngoài góc AMB cắt các đường thẳng NA, NB lần lượt tại
P, Q. Đường thẳng MA cắt đường tròn đường kính NQ tại R, đường thẳng MB cắt đường tròn đường
kính NP tại S và R, S khác M. Chứng minh rằng đường trung tuyến ứng với đỉnh N của tam giác NRS
luôn đi qua một điểm cố định khi M di động phía trong đường tròn.

Ths Trần Minh Hiền - Trường THPT chuyên Quang Trung 23


SỐ PHỨC VỚI HÌNH HỌC PHẲNG – Lê Thị Cẩm Thủy – THPT chuyên Nguyễn Đình Chiểu (Đồng Tháp)

SỐ PHỨC VỚI HÌNH HỌC PHẲNG

Mỗi điểm trong hệ tọa độ vuông góc tương ứng với một số phức. Quan hệ tập hợp các số phức và tập hợp
các điểm trong mặt phẳng tương ứng một - một. Điểm Z  x; y  ứng với số phức z  x  iy . Ta gọi số phức z

là nhãn của điểm Z . Trong bài viết này một điểm trong mặt phẳng luôn luôn kí hiệu là một chữ cái hoa và
nhãn của nó là chữ cài thường tương ứng.
Khi xem số phức như là những điểm trên mặt phẳng với hệ tọa độ Oxy thì có thể xem số phức như là
những vectơ trong mặt phẳng này. Chính vì vậy mà có thể áp dụng được số phức vào giải những bài toán
trong hình học phẳng.
Có thể nói phương pháp số phức là sự kết hợp giữa phương pháp tọa độ với phương pháp véctơ. Chính vì
vậy nó có nhiều thế mạnh trong chứng minh một lớp không ít các bài toán trong hình học phẳng.

I. Độ đo góc của hai tia


1. Góc định hướng
a) Góc định hướng  tạo bởi hai tia đi qua gốc tọa độ O y
Z2
Chọn Z1 , Z 2 nằm trên mỗi tia. Khi đó Z2

z2 Z1
  arg z2  arg z1  arg Z1
z1 Z0

b) Góc định hướng  tạo bởi hai tia đi qua điểm Z 0


O x
z z
  arg  z2  z0   arg  z1  z0   arg 2 0
z1  z0

c) Góc định hướng  tạo bởi hai tia bất kì


uuuuur uuuuur
Cho hai vectơ Z1Z 2 , U1U 2 , với Z1 , Z 2 , U1 , U 2 lần lượt có nhãn z1 , z2 , u1 , u2 . Khi đó, ta chỉ cần quay
uuuuur
vectơ đơn vị của Z1Z 2 một góc  theo chiều dương, nghĩa là

z2  z1 u u
 cos  isin    2 1
z2  z1 u2  u1

Suy ra
u2  u1 z2  z1 u2  u1 u2  u1
 cos  isin    :   cos   isin    : p
u2  u1 z2  z1 z2  z1 z2  z1

Vậy góc  phải tìm thỏa

1
SỐ PHỨC VỚI HÌNH HỌC PHẲNG – Lê Thị Cẩm Thủy – THPT chuyên Nguyễn Đình Chiểu (Đồng Tháp)

 p p
  
 z  z  u  u   u2  u1  z2  z1
cos   2 1 2 1
 
cos    2 z2  z1 u2  u1
 2
 hay  .
sin   p  p


sin  
 
  z2  z1  u2  u1   u2  u1  z2  z1  
2i 2 z2  z1 u2  u1


Nhận xét

 
1) Z1Z 2  U1U 2   z2  z1  u2  u1   u2  u1  z2  z1  0  
 
Z1Z 2 PU1U 2   z2  z1  u2  u1   u2  u1  z2  z1  0  
2) Nếu z1  u1 và z1 z2  u1u2 thì khi biết z2 và  thì ta tìm được u2 theo z2

  900 , 2  iz2

1 3 
  600 , 2    i  z2
2 2 

 3 1 
  300 , 2    i  z2
 2 2 
z2  z0
3) Kí hiệu V  z2 , z1 , z0   là tỉ số đơn của các số phức z2 , z1 , z0 (theo thứ tự đó).
z1  z0
uuuuur uuuuur
Khi đó argumen của V  z2 , z1 , z0  chính là góc định hướng của các vectơ Z0 Z1 , Z0 Z 2 .
uuuuur uuuuur
Điều kiện cần và đủ để z2 , z1 , z0 thẳng hàng là góc định hướng của các vectơ Z0 Z1 , Z0 Z 2

bằng 0 hoặc bằng   . Nghĩa là tỉ số đơn V  z2 , z1 , z0  là một số thực.

2. Ví dụ
Ví dụ 1. Cho hình vuông ABCD. Điểm M là trung điểm CD, điểm P D M C
nằm trên đường chéo AC sao cho PC=3AP. Chứng ming rằng
µ  900 .
BPM
Giải
uuur
Chọn hệ tọa độ sao cho A  O và AB là vectơ đơn vị theo chiều P
dương của trục hoành. Khi đó A, B, C, D, M, P tương ứng có nhãn
A B
1 1 1 1 1
a  0, b  1, c  1  i, d  i , m   c  d   1  2i  , p  c   i .
2 2 4 4 4
m p 1  3i
Ta có: V  m, b, p    i
b p 3i


Vì arg i  nên MPB  900 .
2
Hơn nữa, i  1 ta được PM = PB nên tam giác MPB là vuông cân.

2
SỐ PHỨC VỚI HÌNH HỌC PHẲNG – Lê Thị Cẩm Thủy – THPT chuyên Nguyễn Đình Chiểu (Đồng Tháp)
Ví dụ 2. Cho ba hình vuông bằng nhau ABCD, BEFC, EPQF . Chứng D C F Q


minh rằng ·
ACD  ·
AFD  ·
AQD  .
2
Giải A B E P
uuur
Chọn hệ tọa độ vuông góc sao cho A  O và AB là vectơ đơn vị theo
chiều dương của trục hoành. Khi đó A, B, C, D, E, F, P, Q
tương ứng có nhãn a  0, b  1, c  1  i, d  i, e  2, f  2  i, p  3, q  3  i .

· ·
ACD  BAC  arg c, · ·  arg f , ·
AFD  EAF ·
AQD  PAQ  arg q
nên

·
ACD  · AQD  arg c  arg f  arg q  arg  c f q   arg 10i   .
AFD  ·
2
Ví dụ 3. Cho tam giác ABC. Trong nửa mặt phẳng bờ AB chứa G F
điểm C, dựng hình vuông ABDE. Trong nửa mặt phẳng bờ BC chứa A
điểm A, dựng hình vuông BCFG. Chứng minh GA vuông góc CD
và GA = CD.
Giải
E
uuur C
Chọn hệ tọa độ Oxy sao cho B  O , BC là chiều dương của trục B
hoành. Khi đó, g  ic , d  ia . Gọi  là góc giữa GA và CD, ta
có D

ag ag   a  ic a  ic  i 
  arg  :   arg  :   arg 
 d c d c   ia  c ia  c  i 2

Vậy GA vuông góc CD và GA  a  g  a  ic  ia  c  d  c  CD .

 
Ví dụ 4. Cho tam giác ABC BAC  600 . Ở miền ngoài của tam
E
giác vẽ các tam giác đều ABD, ACE. Dựng hình bình hành AEFD. F
Chứng minh tam giác BFD đều.
Giải D A
Chọn hệ tọa độ vuông góc sao cho A  O . Khi đó A, B, C tương
ứng có nhãn a  0, b, c .
B C
Do cách dựng tam giác đều, ta có
1 3  1 3 
 
e  c cos600  i sin 600  c    
i  , d  b cos600  i sin 600  b   i 
2 2  2 2 

3
SỐ PHỨC VỚI HÌNH HỌC PHẲNG – Lê Thị Cẩm Thủy – THPT chuyên Nguyễn Đình Chiểu (Đồng Tháp)
1 3
f ed  c  b  i c  b
2 2
1 3 1 3
BF  f  b   c  b   i c  b  b  c  b  i  c  b   c  b  BC
2 2 2 2

1 3 1 3
CF  f  c   c  b   i  c  b   c    c  b   i  c  b   c  b  BC
2 2 2 2

Vậy tam giác BCF đều.

3. Bài tập
Bài 1.1. Cho hình vuông ABCD. Gọi N là điểm trên cạnh BC sao cho NC  2 NB , M là điểm trên cạnh BD
sao cho MB  2MD . Chứng minh rằng tam giác AMN vuông tại M.
Bài 1.2. Cho tam giác ABC, vẽ về phía ngoài của tam giác các hình vuông ABEF, ACGH . Chứng minh rằng
đường trung tuyến AM vẽ từ đỉnh A của tam giác ABC là đường cao của tam giác AHF.
Bài 1.3. Cho tam giác ABC, M là trung điểm BC. Trên cạnh AB lấy điểm D sao cho BD = 2AD. Các đoạn
thẳng AM và CD cắt nhau tại I . Chứng minh rằng:
a) I là trung điểm AM.
b) CI = 3DI.
Bài 1.4. Về phía ngoài của tam giác ABC vẽ các hình vuông ABDE, ACFG. Gọi H, K, L lần lượt là trung điểm
các đoạn BE, BC, CG.
a) Chứng minh rằng tam giác HKL vuông cân.
b) Có nhận xét gì về vị trí thứ tư của hình vuông có ba đỉnh H, K, L.
Bài 1.5. Về phía ngoài của tan giác ABC vẽ các tam giác cân MAB, NAC, PCB theo thứ tự nhận các điểm M,
N, P làm đỉnh góc vuông. Chứng minh rằng các đoạn thẳng AP và MN bằng nhau và vuông góc nhau.
Bài 1.6. Cho tứ giác lồi ABCD. Gọi M, N theo thứ tự là trung điểm AB, CD. Gọi E, F lần lượt là giao điểm
của các đường thẳng AD và BC với MN. Chứng minh rằng nếu AD = BC thì góc AEM bằng góc BFM.
Bài 1.7. Về phía ngoài của một tứ giác ABCD trên các cạnh AB, BC,CD, DA dựng các hình vuông, tâm lần
lượt của các hình vuông này là E, F, G, H. Chứng minh rằng EG = FH và EG  FH .

4
SỐ PHỨC VỚI HÌNH HỌC PHẲNG – Lê Thị Cẩm Thủy – THPT chuyên Nguyễn Đình Chiểu (Đồng Tháp)

II. Phương trình đường thẳng


1. Đường thẳng qua hai điểm
uuuur uuuur
- Điều kiện cần và đủ để ba điểm phân biệt z0 , z1 , z2 thẳng hàng là góc giữa hai vectơ z0 z2 , z1 z2 bằng

0 hoặc   . Nói cách khác tỉ số đơn V  z0 , z1 , z2  là một số thực, hay

z0  z 2 z 0  z 2
 .
z1  z2 z1  z2

- Như vậy, một đường thẳng đi qua hai điểm z1 , z2 là tập hợp các điểm z sao cho

z  z2 z  z2

z1  z2 z1  z2

hoặc là

z  z z z  z z  z z
1 2 1 2 1 2  z1 z2  0

- Phương trình trên được gọi là phương trình đường thẳng.


Nếu đặt B  z1  z2 và C  z1 z2  z1 z2 thì B z  B z  C  0,  B  0  , vì C  z1 z2  z1 z2  C

nên C là số ảo.
- Ngược lại, mọi phương trình có dạng trên biểu diễn một đường thẳng trên mặt phẳng.
2. Phương trình tham số
- Ba điểm phân biệt z, z1 , z2 nằm trên một đường thẳng khi và chỉ khi tỉ số đơn V  z, z1 , z2  là một số

thực. Vì vậy, với mỗi số thực t thì số phức z  z2  t  z1  z2   t z1  1  t  z2 được biểu diễn bởi một

điểm thuộc đường thẳng đi qua Z1 , Z 2 và ngược lại. Như vậy, với t R , phương trình

z  z2  t  z1  z2   t z1  1  t  z2

được gọi là phương trình tham số của đường thẳng Z1 Z 2 .


- Lưu ý: Trong ứng dụng giải bài tập hình học, ta cần xem xét khi t thay đổi thì ảnh hưởng của z như
thế nào đối với z1 , z2

z  z2 uuuuur uuuur
 Nếu t  là số thực dương thì các vectơ Z 2 Z1 , Z 2 Z cùng chiều.
z1  z2

z  z2 uuuuur uuuur
 Nếu t  là số thực âm thì các vectơ Z 2 Z1 , Z 2 Z ngược chiều.
z1  z2

 Đối với vị trí điểm Z xác định như sau


0  t  1 thì Z nằm trong đoạn Z1Z 2

t  1 thì Z nằm ngoài đoạn Z1Z 2 về phía Z1

t  0 thì M nằm ngoài đoạn Z1Z 2 về phía Z 2 .

5
SỐ PHỨC VỚI HÌNH HỌC PHẲNG – Lê Thị Cẩm Thủy – THPT chuyên Nguyễn Đình Chiểu (Đồng Tháp)
- Giá trị tuyệt đối của t bằng tỷ số đoạn thẳng Z0 Z 2 , Z1Z 2 . Trong thực tế, ta thường tìm trên đường

Z1Z z  z1
thẳng Z1Z 2 một điểm Z sao cho  t cho trước. Khi đó  t . Nghĩa là, nếu Z thuộc đoạn
ZZ 2 z2  z

z1  tz2 1 t
Z1Z 2 thì z   z1  z2 , t R .
1 t 1 t 1 t
3. Ví dụ
Ví dụ 1. Cho hai hình vuông cùng hướng OABC và OA1B1C1 . Chứng minh rằng các đường thẳng

AA1 , BB1 , CC1 đồng quy.


Giải
Chọn hệ tọa độ vuông góc có góc tại O. C B
Khi đó c  ai , b  a  c  a 1  i  , c1  a1i ,

b1  a1  c1  a1 1  i  .

Phương trình các đường thẳng O A

 
AA1 : a  a1 z   a  a1  z  aa1  aa1  0 (1)
C1
BB :  b  b  z   b  b  z  bb  bb  0
1 1 1 1 1 (2)

CC :  c  c  z   c  c  z  cc  cc  0
1 1 1 1 1 (3) A
1


Ta có cc1  cc1  aa1  aa1 và bb1  bb1  2 aa1  aa1 . 
B1

Cộng (1) và (2) ta được (3). Điều này chứng tỏ điểm chung của AA1 và CC1 nằm trên BB1 . W
Ví dụ 2. Trên các cạnh AB và AC của tam giác đều ABC lấy các điểm E và D tương ứng sao cho
AD BE 1
  . Chứng minh rằng nếu P là giao điểm của BD và CE thì góc APC bằng 900 .
DC EA 2

Giải C
Chọn hệ tọa độ vuông góc có góc tại A, chiều dương của
uuur 2 1 i 3
trục hoành là AB và b  1 . Khi đó e  , c   ,
3 2 2 D
P
1 i 3
d  . Phương trình của BD và CE là:
6 6 A E B

 5 i 3 5 i 3 i 3
 BD :    z     z  0
 6 6  6 6  3

  1 i 3   1 i 3  2i 3
CE :  6  2  z   6  2  z  3  0
    

6
SỐ PHỨC VỚI HÌNH HỌC PHẲNG – Lê Thị Cẩm Thủy – THPT chuyên Nguyễn Đình Chiểu (Đồng Tháp)
Giao điểm P là nghiệm của hệ phương trình trên. Giải hệ ta được
z p

c p
Xét V  c, a, p    là số ảo. Vậy ·
APC  900 . W
a p

A2 B1
Ví dụ 3. Cho tam giác A1 A2 A3 . Trên cạnh A2 A3 , A3 A1 lấy các điểm B1 , B2 sao cho  t1 và
B1 A3

A3 B2 a t a t t a
 t2 . Chứng minh rằng, nếu M là giao điểm của A1 B1 và A2 B2 thì m  2 1 3 1 2 1
B2 A1 1  t1  t1t2

Giải. A3
a2  t1a3 a t a
Từ giả thiết, ta có b1  , b2  3 2 1 B2
1  t1 1  t2 M B1
a2  t1a3  t1t2 a1
Nếu đặt z  thì ta nhận được A1 A2
1  t1  t1t2

z
1  t1 a t a tt a
 2 1 3  12 1 
1  t1
 b1 
t1t2
 a1 
1  t1  b1  t1t2 a1
1  t1  t1t2 1  t1 1  t1  t1t2 1  t1  t1t2 1  t1  t1t2 1  t1  t1t2

điều này suy ra Z có nhãn z nằm trong đoạn A1 B1 .

a2 t 1  t2   a3  t2 a1  a2   t1  t1t2  b2
Tương tự, z   1   , suy ra Z nằm trong đoạn A2 B2 .
1  t1  t1t2 1  t1  t1t2 1  t2 1  t1  t1t2

Vậy Z  M là giao điểm của A1 B1 và A2 B2 . W

Chú ý. Từ chứng minh trên ta có thể tính


a2  t1a3  t1t2 a1
 a1
A1M m  a1 1  t1  t1t2 1  t1   a2  t1a3   1  t1  a1  1  t1
    .
MB1 b1  m a2  t1a3  a2  t1a3  t1t2 a1 t1t2  a2  t1a3   1  t1  a1  t1t2
1  t1 1  t1  t1t2

a2  t1a3  t1t2 a1
 a2
A2 M m  a2 1  t1  t1t2 1  t2  t1  a3  t2 a1   1  t2  a2 
    t1 1  t2  .
MB2 b2  m a3  t2 a1  a2  t1a3  t1t2 a1  a3  t2 a1   1  t2  a2
1  t2 1  t1  t1t2

Trường hợp đặc biệt, t1  t2  1 thì A1 B1 , A2 B2 là đường trung tuyến của tam giác A1 A2 A3 và M là

a1  a2  a3 AM A M A M 2
trọng tâm của tam giác A1 A2 A3 với nhãn m  và 1  2  2  .
3 MB1 MB2 MB2 1

Tương tự ta có thể tính được những điểm đặc biệt khác trong tam giác như tâm đường tròn ngoại
tiếp, trực tâm, …

7
SỐ PHỨC VỚI HÌNH HỌC PHẲNG – Lê Thị Cẩm Thủy – THPT chuyên Nguyễn Đình Chiểu (Đồng Tháp)
Ví dụ 4. (Định lí Cheva) Cho ba điểm B1 , B2 , B3 nằm trên các cạnh tương ứng A2 A3 , A3 A1 , A1 A2 của

A2 B1 AB AB
tam giác A1 A2 A3 và  t1 , 3 2  t2 , 1 3  t3 . Chứng minh rằng, những đường thẳng
B1 A3 B2 A1 B3 A2

A1B1 , A2 B2 , A3 B3 cắt nhau tại một điểm khi và chỉ khi t1t2t3  1 .
Giải A3
a1  t3 a2 a2  t1t2 a1
1) Nếu t1t2t3  1 . Khi đó, b3   . B1
1  t3 1  t1t2 B M
2
Gọi M là giao điểm của A1B1 , A2 B2 . Theo ví dụ 3, ta có
A1 B3 A2
a t a t t a
m 2 1 3 12 1
1  t1  t1t2

m  a3
Ta cần chứng minh M thuộc A3 B3 , muốn vậy ta cần chứng minh là một số thực.
b3  m

a2  t1a3  t1t2 a1
 a3
m  a3 1  t1  t1t2 1  t1t2
Thật vậy,   .
b3  m a2  t1t2 a1  a2  t1a3  t1t2 a1 t1
1  t1t2 1  t1  t1t2

Suy ra điểm M nằm trên đường thẳng A3 B3 .

Do đó A1B1 , A2 B2 , A3 B3 cắt nhau tại M.

2) Ngược lại, nếu A1B1 , A2 B2 , A3 B3 cắt nhau tại M. Khi đó, ta tính được

 a2  t1a3  t1t2 a1
m  1  t  t t
 1 1 2

a 
m  3 2 1 t2t3 a2
t a 
 1  t3  t1t3
 1  t1t2t3   a2  a3  t1  a1  a2    0

Vì A1 , A2 , A3 không thẳng hàng nên a2  a3  t1  a1  a2   0

Suy ra t1t2t3  1. W

Ví dụ 5. (Định lí Menelai) Cho ba điểm B1 , B2 , B3 nằm trên các cạnh tương ứng A2 A3 , A3 A1 , A1 A2 của

A2 B1 AB AB
tam giác A1 A2 A3 và  t1 , 3 2  t2 , 1 3  t3 . Chứng minh rằng, ba điểm B1 , B2 , B3 thẳng
B1 A3 B2 A1 B3 A2

hàng khi và chỉ khi t1t2t3  1 .


Giải
1) Nếu t1t2t3  1 . Ta có

8
SỐ PHỨC VỚI HÌNH HỌC PHẲNG – Lê Thị Cẩm Thủy – THPT chuyên Nguyễn Đình Chiểu (Đồng Tháp)
a2  t1a3 a1  t3 a2 A3

b1  b3 1  t1 1  t3 1  t2
   B1
b3  b2 a1  t3 a2  a3  t2 a1 1  t1  t2t3 B2
1  t3 1  t2

Là một số thực. Suy ra ba điểm B1 , B2 , B3 thẳng hàng. A1 A2


B3

2) Ngược lại, nếu ba điểm B1 , B2 , B3 thẳng hàng. Ta chọn một số t3' sao cho t1t2t3'  1 .

a1 + t3' a2
Theo chứng minh ở ví dụ trên thì điểm B3' có b3' = nằm trên B1B2 . Như vậy, B3' º B3 .
1 + t3'

Suy ra t3' = t3 . Vậy t1t2t3  1 . W

4. Bài tập
Bài 2.1. (Định lí Shainer) Chứng minh rằng giao điểm của hai đường chéo một hình thang, giao điểm hai
cạnh bên và những trung điểm của hai cạnh đáy, nằm trên cùng một đường thẳng.
Bài 2.2. Điểm D chia cạnh AC của tam giác ABC, đáy lớn AB. M là trung điểm cạnh BC và P là giao điểm
của AM và đường thẳng song song với BC xuất phát từ D. Chứng minh rằng, nếu AP = PM

thì những đường thẳng AC, DP, MN cắt nhau tại một điểm, với N là trung điểm cạnh AD.
Bài 2.3. Qua điểm nằm trong hình bình hành ABCD ta kẻ các đường thẳng song song với các cạnh của
chúng. Chúng cắt các cạnh AB, BC, CD, DA lần lượt tại các điểm P, Q, R, S. Chứng minh rằng
nếu đường thẳng PQ và RS cắt nhau, thì giao điểm của chúng nằm trên AC.
Bài 2.4. Về phía ngoài của tam giác ABC vẽ các hình vuông ABEF và ACGH, N là trung điểm FH, M là
giao điểm của các đường BG và CE. Chứng minh rằng các điểm N, M, A thẳng hàng.
Bài 2.5. Cho hình thang ABCD đáy lớn AB. Kẻ đường thẳng song song với hai đáy, lần lượt cắt các cạnh
bên BC, AD tại N và M, cắt các đường chéo AC, BD tại P và Q. Chứng minh rằng MD = QN.
AM DN
Bài 2.6. Trên cạnh AB và DC của tứ giác ABCD lấy điểm M và N sao cho = = k . Các điểm
AB DC
P, Q, S lần lượt là trung điểm của các đoạn thẳng AD, BC, MN. Chứng minh rằng P, Q, S cùng
PS
nằm trên một đường thẳng và = k.
PQ
Bài 2.7. Cho tam giác ABC lấy K là trung điểm AB, từ K kẻ hai đường song song lần lượt với trung
tuyến AA1 , BB1 cuả tam giác ABC và cắt các cạnh AC tại M, BC tại N. Chứng minh rằng các

đường trung tuyến AA1 , BB1 chia đường thẳng MN thành ba phần bằng nhau.

Bài 2.8. Cho A1 , B1 , C1 là trung điểm các cạnh BC, CA, AB của tam giác ABC, M là một điểm trong

tam giác ABC, lấy M1 , M 2 , M 3 lần lượt là các điểm đối xứng của M qua các điểm A1 , B1 , C1 .

Chứng minh rằng những đường thẳng AM1 , BM 2 , CM 3 cắt nhau tại một điểm.

9
SỐ PHỨC VỚI HÌNH HỌC PHẲNG – Lê Thị Cẩm Thủy – THPT chuyên Nguyễn Đình Chiểu (Đồng Tháp)

III. Phương trình đường tròn


1. Phương trình tổng quát
Chúng ta tìm điều kiện cần và đủ để 4 điểm Z3

Z0 , Z1, Z 2 , Z3 nằm trên một đường tròn. Ở đây ta có thể Z


2

coi đường thẳng như là đường tròn tâm vô tận. Nếu


Z0 , Z1, Z 2 , Z3 nằm trên đường tròn thì hiệu giữa góc định
·Z Z và Z
hướng Z ·Z Z là 0 hoặc ± p . Suy ra tỷ số Z
0
Z
1
0 2 1 0 3 1

V (z0 , z1 , z2 ) z0 - z2 z0 - z3
= : là một số thực.
V (z0 , z1 , z3 ) z1 - z2 z1 - z3

V (z0 , z1 , z2 )
Ngược lại, nếu tỷ số là một số thực, thì z0 , z1 , z2 , z3 là nhãn của những điểm trên
V (z0 , z1 , z3 )

V (z0 , z1 , z2 )
đường tròn hoặc đường thẳng. W (z0 , z1 , z2 , z3 ) = gọi là tỷ số kép của 4 điểm
V (z0 , z1 , z3 )

z0 , z1 , z2 , z3 (theo thứ tự này). Như vậy

Điều kiện cần và đủ để 4 điểm Z0 , Z1 , Z 2 , Z3 nằm trên đường thẳng hoặc đường tròn là tỷ số kép

của nhãn z0 , z1 , z2 , z3

V (z0 , z1 , z2 ) z0 - z2 z0 - z3
W (z0 , z1 , z2 , z3 ) = = :
V (z0 , z1 , z3 ) z1 - z2 z1 - z3
là một số thực. Hoặc là

z0 - z2 z0 - z3 z0 - z2 z0 - z3
: = : .
z1 - z2 z1 - z3 z1 - z2 z1 - z3

Từ phương trình trên, để một điểm Z nằm trên đường tròn ngoại tiếp tam giác Z1Z 2 Z3 là phương

trình sau thỏa mãn

z - z2 z - z3 z - z2 z - z3
: = :
z1 - z2 z1 - z3 z1 - z2 z1 - z3

Ta có thể gọi đây là phương trình đường tròn xác định bởi 3 điểm Z1 , Z2 , Z3 . Khử mẫu số ta

được

(z - z2 )(z1 - z3 )(z - z 3 )(z1 - z 2 )- (z1 - z3 )(z - z 2 )(z1 - z2 )(z1 - z 3 )= 0

hoặc là a z z + b z - b z + g = 0 , ở đây

10
SỐ PHỨC VỚI HÌNH HỌC PHẲNG – Lê Thị Cẩm Thủy – THPT chuyên Nguyễn Đình Chiểu (Đồng Tháp)
a = (z1 - z3 )(z1 - z2 )- (z1 - z2 )(z1 - z3 )

b = - z3 (z1 - z3 )(z1 - z2 )+ z2 (z1 - z2 )(z1 - z3 )

g = z2 z3 (z1 - z3 )(z1 - z2 )- z3 z2 (z1 - z2 )(z1 - z3 ).

Rõ ràng, a = - a , b = - b .
Như vậy, mọi đường tròn có phương trình dạng

a z z + bz- b z+ g = 0
trong đó a , g hoàn toàn ảo.

Phương trình đường tròn là phương trình đường thẳng khi và chỉ khi a = 0 . Điều này cũng phù
hợp với điều kiện ba điểm Z1 , Z2 , Z3 thẳng hàng.

b g
Bây giờ, nếu a ¹ 0 , đặt a = - , b= thì phương trình đường tròn có dạng
a a
z z - az - az + b = 0 .
Từ đó suy ra
2 2
z- a = a - b

2
do đó a là nhãn của điểm tâm đường tròn và bán kính R = a - b.

Trường hợp đặc biệt, tâm của đường tròn trùng với gốc tọa độ và bán kính bằng 1, thì phương

trình đường tròn có dạng z z = 1 . Đường tròn này được gọi là đường tròn đơn vị.
2. Đường tròn đơn vị
1 1 1 1
Nếu Z1 , Z 2 , U1 , U 2 đều nằm trên đường tròn đơn vị thì z1 = , z2 = , u1 = , u 2 = . Và
z1 z2 u1 u2

do đó, Z1Z 2 ^ U1U 2 khi và chỉ khi z1 z2 + u1u2 = 0 .

Tương tự, Z1Z 2 PU1U 2 khi và chỉ khi z1 z2 = u1u2 .

3. Giao điểm hai cát tuyến


Điều kiện để ba điểm A, B, U nằm trên một đường thẳng

u- a u- a B
là = .
b- a b- a
U
1 1
Nếu A và B nằm trên đường tròn đơn vị, thì a = , b = , A
a b
khi đó điều kiện trên được viết lại là a + b = u + abu .
Đây cũng là điều kiện cần và đủ để U thuộc đường thẳng AB.

11
SỐ PHỨC VỚI HÌNH HỌC PHẲNG – Lê Thị Cẩm Thủy – THPT chuyên Nguyễn Đình Chiểu (Đồng Tháp)

Nếu Z1Z 2 , U1U 2 là hai cung của đường tròn đơn vị cắt U
1

nhau, thì giao điểm S của chúng thỏa hệ

z1 + z2 = s + z1 z2 s Z
2
S
u1 + u2 = s + u1u2 s Z
1 U
2
Từ đây ta có công thức tính nhãn s của giao điểm

(z1 + z2 )u1u2 - (u1 + u2 )z1z2


s=
u1u2 - z1 z2

Do Z1Z 2 , U1U 2 không song song nên u1u2 - z1 z2 ¹ 0 .

4. Chân đường vuông góc ở dây cung


Gọi S là chân đường vuông góc hạ từ đểm M xuống đường
M
thẳng AB, với A, B nằm trên đường tròn đơn vị.

Vì S nằm trên AB nên a + b = s + abs


B
Mặt khác, MS ^ AB , ta có S
A
(m - s)(a - b)+ (m - s)(a - b)= 0
Từ đó tính được nhãn s của chân đường vuông góc hạ từ M xuống dây AB
1
s=
2
(a + b + m - abm)
5. Giao điểm của hai tiếp tuyến
Gọi S là giao điểm của các đường tiếp tuyến với đường tròn tâm O tại A và B. Khi đó, do
SA ^ AO và SB ^ BO nên ta có
ìï (a - s )a + (a - s)a = 0 ìï sa + as = 2
ïï
í Û ïí
ïï (b - s )b + b - s b = 0 ï
ïî ( ) îï sb + bs = 2
Từ đó suy ra
2ab
s= ×
a+ b
6. Ví dụ
Ví dụ 1. Trong mặt phẳng cho 4 đường tròn k1 , k2 , k3 , k4 . Gọi X1 , Y1 là giao điểm của k1 , k2 ; X 2 , Y2 là

giao điểm của k2 , k3 ; X 3 , Y3 là giao điểm của k3 , k4 ; X 4 , Y4 là giao điểm của k4 , k1 . Chứng minh rằng

nếu X1 , X 2 , X 3 , X 4 nằm trên một đường tròn hoặc đường thẳng, thì Y1 , Y2 , Y3 , Y4 cùng nằm trên một

đường tròn hoặc đường thẳng.


Giải

12
SỐ PHỨC VỚI HÌNH HỌC PHẲNG – Lê Thị Cẩm Thủy – THPT chuyên Nguyễn Đình Chiểu (Đồng Tháp)
Do X1 , Y2 , X 2 , Y1 nằm trên đường tròn k 2 nên tỉ số kép X1

x1 - x2 x1 - y1
W (x1 , y2 , x2 , y1 ) = : X
4
y2 - x2 y2 - y1 Y1
Y4
là một số thực. X2 Y2 Y3

Tương tự, các tỷ số kép khác


x2 - x3 x2 - y2 X
3
W (x2 , y3 , x3 , y2 ) = :
y3 - x3 y3 - y2
x3 - x4 x3 - y3
W (x3 , y4 , x4 , y3 ) = :
y4 - x4 y4 - y3
x4 - x1 x4 - y4
W (x4 , y1 , x1 , y4 ) = :
y1 - x1 y1 - y4
là những số thực. Do đó

W (x1 , y2 , x2 , y1 )W (x3 , y4 , x4 , y3 ) éx - x x - x ù éy - y y - y ù
ê1
= 2
: 1 4úê 1
× 2
: 1 4ú
W (x2 , y3 , x3 , y2 )W (x4 , y1 , x1 , y4 ) êx - x x - x ú êy - y y - y ú
ë3 2 3 4ûë 3 2 3 4û

= W (x1 , x3 , x2 , x4 )W ( y1 , y3 , y2 , y4 ).

là một số thực. Nhưng từ giả thiết, X1 , X 2 , X 3 , X 4 nằm trên một đường tròn hoặc đường thẳng nên

W (x1, x3 , x2 , x4 ) là một số thực, và do vậy W ( y1, y3 , y2 , y4 ) cũng là một số thực, nghĩa là

Y1 , Y2 , Y3 , Y4 cùng nằm trên một đường tròn hoặc đường thẳng. W


Ví dụ 2. Từ các đỉnh của tứ giác ABCD nội tiếp trong đường tròn tâm O ta dựng các đường tiếp tuyến với
đường tròn đó và chúng cắt nhau tạo ra tứ giác PQRS. Chứng ming rằng những trung điểm các đường
chéo của tứ giác PQRS cùng thuộc một đường thẳng với tâm của đường tròn.
S
Giải
Chọn hệ tọa độ vuông góc sao cho đường tròn chính là đường
tròn đơn vị. Khi đó, ta có
2ab 2bc 2cd 2da
p= , q= , r= , p=
a+ b b+ c c+ d d+ a
Trung điểm M, N của đường chéo PR và QS có nhãn là N
A
1 abc + abd + acd + bcd D
m= ( p + r )= O
2 (a + b)(c + d ) P
M R
1 bcd + abc + abd + acd
n = (q + s )=
2 (b + c)(d + a) B C

m (b + c)(d + a ) Q
Suy ra = ×Nhưng
n (a + b)(c + d )

13
SỐ PHỨC VỚI HÌNH HỌC PHẲNG – Lê Thị Cẩm Thủy – THPT chuyên Nguyễn Đình Chiểu (Đồng Tháp)
æ1 1 öæ 1ö
çç + ÷
÷ çç 1 + ÷
÷
m
=
(b + c)(d + a)
=
çèb ÷
c øèç d ÷ (b + c)(d + a ) m

= = ×
n (a + b)(c + d ) æ çç 1 + 1 öæ
÷ çç1 + 1ö
÷ (a + b)(c + d ) n
çè a ÷
÷ ÷
÷
b øèçc dø
m
Do dó là số thực, nghĩa là các điểm M, N nằm trên đường thẳng đi qua O. W
n
Ví dụ 3. Cho hình chữ nhật ABCD. Từ một điểm K bất kì trên đường tròn ngoại tiếp hình chữ nhật, hạ
những đường thẳng vuông góc xuống AB, CD, AD, BC và cắt các cạnh này lần lượt tại P, Q, R, S. Chứng
minh rằng PR vuông góc với QS và PS vuông góc với QR.
Giải
Chọn hệ tọa độ vuông góc sao cho đường tròn chính là đường P K Q
tròn đơn vị. Khi đó: c = - a, d = - b và A R D
1
p=
2
(a + b + k - abk )
1 1
r = (a + d + k - ad k ) = (a - b + k + abk )
2 2
B S C
1 1
q = (c + d + k - cd k ) = (- a - b + k - abk )
2 2
1 1
s = (b + c + k - bck ) = (- a + b + k + abk )
2 2

1 1
p- r (a + b + k - abk )- (a - b + k + abk )
a- k
= 2 2 = ×
q - s 1 - a - b + k - abk - 1 - a + b + k + abk a+ k
2
( ) 2( )

1 1
-
p- r a- k a k = - a- k = - p- r ×
= =
q- s a+ k 1 + 1 a+ k q- s
a k
p- r
Suy ra hoàn toàn ảo. Hay PR ^ QS .
q- s

p- s b- k p- s p- s
Tương tự ta có = và =- .
q- r b+ k q- r q- r
Vậy PR ^ QS và PS ^ QR . W

14
SỐ PHỨC VỚI HÌNH HỌC PHẲNG – Lê Thị Cẩm Thủy – THPT chuyên Nguyễn Đình Chiểu (Đồng Tháp)
7. Bài tập
Bài 3.1. Trên một đường tròn có hai cung AB và CD. Nếu M là giao điểm của các đường vuông góc dựng từ A
đối với AB và từ C đối với CD, còn N là giao điểm của các đường vuông góc dựng từ B đối với AB và
từ D đối với CD. Chứng minh rằng MN đi qua giao điểm của hai đường thẳng BC và AD, nếu nó cắt
nhau hoặc MN song song với BC và AD, nếu chúng song song.
Bài 3.2. Trên đường tròn cho 4 điểm ABCD sao cho AB là đường kính, còn CD thì không. Đường thẳng AC
và BD cắt nhau tại điểm X. Từ mỗi điểm C và D dựng các đường tiếp tuyến với đường tròn và chúng
cắt nhau tại Y. Chứng minh rằng đường thẳng XY vuông góc với AB và đi qua giao điểm của hai
đường thẳng AD và CB.
Bài 3.3. Trên đường tròn k kẻ hai đường kính AB và CD. M là điểm bất kì trên đường tròn k, còn P và Q là
chân đường vuông góc hạ từ M xuống AB và CD. Chứng minh rằng độ dài đoạn PQ không phụ thuộc
vào vị trí của M trên đường tròn.
Bài 3.4. Từ trung điểm M của dây cung PQ trong một đường tròn, dựng hai dây cung bất kì AB và CD. Dây
cung AD và BC cắt PQ lần lượt tại X và Y. Chứng minh rằng M là trung điểm của đoạn XY.
Bài 3.5. Cho tứ giác ABCD nội tiếp đường tròn. Các tiếp tuyến tại A và C cắt nhau tại M; các tiếp tuyến tại B
và D cắt nhau tại N. Chứng minh rằng các điểm A, C, N nằm trên một đường thẳng khi và chỉ khi các
điểm B, D, M nằm trên một đường thẳng.
Bài 3.6. Cho hình bình hành ABCD. Dựng đường tròn đường kính AC. Gọi M và N là những điểm cắt thứ hai
của đường tròn vừa dựng với đường thẳng AB và AD. Chứng minh rằng đường thẳng BD, MN và
đường tiếp tuyến với đường tròn tại C cắt nhau tại một điểm.
»
Bài 3.7. Cho P là một điểm trên nửa đường tròn S với đường kính AB. Lấy hai cung bằng nhau trên S là BC
» . Chứng minh rằng nếu AC và BP cắt nhau tại E, AD và CP cắt nhau tại F, thì EF vuông góc
và CD
với AD.
Bài 3.8. Tại hai đầu đường kính AB của đường tròn dựng hai tiếp tuyến t A và t B . Qua điểm C trên t A , không

trùng với A dựng hai cung bất kì D1E1 và D2 E2 đối với đường tròn. Chứng minh rằng các tia

AD1 , AD2 cắt đường thẳng t B thành một đoạn thẳng có độ dài bằng độ dài đoạn thẳng do hai tia

AE1 , AE2 taoj ra trên t B .


Bài 3.9. Trên nửa đường tròn đường kính AB lấy hai điểm bất kì C và D. Những điểm P, Q, R là trung điểm
của những đoạn AC, CD, BD. Qua mỗi điểm P, Q lần lượt vẽ các đường thẳng vuông góc vơi AQ,
BQ, chúng cắt các tiếp tuyến tại A, B tại các điểm T, S. Chứng minh rằng ST và CD song song.

15
SỐ PHỨC VỚI HÌNH HỌC PHẲNG – Lê Thị Cẩm Thủy – THPT chuyên Nguyễn Đình Chiểu (Đồng Tháp)

LỜI GIẢI VÀ HƯỚNG DẪN BÀI TẬP

Bài 1.1. Cho hình vuông ABCD. Gọi N là điểm trên cạnh BC sao cho NC  2 NB , M là điểm trên cạnh BD
sao cho MB  2MD . Chứng minh rằng tam giác AMN vuông tại M.
Giải
Chọn tọa độ điểm gốc là C và a  1 , c  i , d  1  i . D C
i 2
Khi đó n  , m   1  i  . M
3 3
2
1   1  i  N
am 1  2i
V  a, n, m    3   i
n  m i  2 1  i 2i
 
3 3
A B
Vậy AM  MN .
Bài 1.2. Cho tam giác ABC, vẽ về phía ngoài của tam giác các hình vuông ABEF, ACGH . Chứng minh rằng
đường trung tuyến AM vẽ từ đỉnh A của tam giác ABC là đường cao của tam giác AHF.
Giải
bc F
Chọn tọa độ điểm gốc là A, do đó m  , h  ci ,
2 H E

f  bi . Xét:

 h  f a  m  h  f a  m
G A

b  c b  c bc
  ci  bi 
2
 ci  bi 2
i 2
c  b  b  c  0
C M B

Vậy AM  HF .
Bài 1.3. Cho tam giác ABC, M là trung điểm BC. Trên cạnh AB lấy điểm D sao cho BD = 2AD. Các đoạn
thẳng AM và CD cắt nhau tại N . Chứng minh rằng:
a) N là trung điểm AM.
b) CN= 3DN.
Giải
bc 1 C
Chọn tọa độ điểm gốc là A, ta có m  , d  b.
2 3
M
bc
Gọi N’ là trung điểm AM, ta có n '  . Xét: N
4
bc
c A D B
c  n' 4  3. 3c  b  3
V  c, d , n '   
d  n' b  b  c b  3c
3 4

16
SỐ PHỨC VỚI HÌNH HỌC PHẲNG – Lê Thị Cẩm Thủy – THPT chuyên Nguyễn Đình Chiểu (Đồng Tháp)
Suy ra C, N’, D thẳng hàng.
DN 1
Vậy N '  N và  
DC 4

Bài 1.4. Về phía ngoài của tam giác ABC vẽ các hình vuông ABDE, ACFG. Gọi H, K, L lần lượt là trung điểm
các đoạn BE, BC, CG.
a) Chứng minh rằng tam giác HKL vuông cân.
b) Có nhận xét gì về vị trí thứ tư của hình vuông có ba đỉnh H, K, L.
Giải
bc G
a) Chọn tọa độ điểm gốc là A, ta có k  , e  bi , P
2 E

b  bi c  ci A
F
g  ci , h  , l . Xét: H
L
2 2
D
h  k c  bi
V  h, l , k    i
l  k b  ci B K C

Suy ra HK  KL và HK = KL.
b) Đỉnh thứ tư của hình vuông có ba đỉnh H, K, L là trung điểm P của EG.
ci  bi h  b b  ci
Thật vậy, ta có p  và V  h, l , p     1
2 l  b b  ci

Bài 1.5. Về phía ngoài của tan giác ABC vẽ các tam giác cân MAB, NAC, PCB theo thứ tự nhận các điểm M,
N, P làm đỉnh góc vuông. Chứng minh rằng các đoạn thẳng AP và MN bằng nhau và vuông góc nhau.
Giải
Chọn tọa độ điểm gốc là A, ta có a  0 .
Theo điều kiện đã cho, ta có P
C
ib
0 - m = i (b - m) Þ m =
i- 1 N
ic - b
b - p = i (c - p) Þ p =
i- 1
c A B
c  n  i 0  n  n 
i 1
ib - c ci - b
Có m - n = , p- a =
i- 1 i- 1 M
m- n
Suy ra m - n = p - a Þ MN = PA và = - i , do đó MN ^ AP . W
p- a
Bài 1.6. Cho tứ giác lồi ABCD. Gọi M, N theo thứ tự là trung điểm AB, CD. Gọi E, F lần lượt là giao điểm

của các đường thẳng AD và BC với MN. Chứng minh rằng nếu AD = BC thì · ·
AEM bằng BFM .

17
SỐ PHỨC VỚI HÌNH HỌC PHẲNG – Lê Thị Cẩm Thủy – THPT chuyên Nguyễn Đình Chiểu (Đồng Tháp)
Giải
F
a+ b c+ d
Ta có m = ,n= ×
2 2 E C
(a - d )(m - n)+ (a - d )(m - n) D
cos ·
N
AEM =
2 m- n . a- d
A M B
·
(b - c)(m - n)+ (b - c)(m - n)
cos MFB =
2 m- n . b- c
2 2
( )
Nếu AD = BC thì a - d = b - c , lúc đó (a - d ) a - d = (b - c) b - c . ( )
(m - n)éë(a -
d )- (b - c)ù m - n)éê(a - d )- (b - c)ù
Cos · · =
AEM - cos MFB û+ ( ë ú
û
2 m- n b- c 2 m- n b- c

é(a - d )+ (b - c)ùé(a - d )- (b - c )ù é(a - d )- (b - c )ùé(a - d )- (b - c)ù


ê ú
ûë û ë ûêë ú
= ë + û
4 m- n b- c 4 m- n b- c
= 0.

Vậy · ·
AEM = BFM .W
Bài 1.7. Về phía ngoài của một tứ giác ABCD trên các cạnh AB, BC,CD, DA dựng các hình vuông, tâm lần
lượt của các hình vuông này là E, F, G, H. Chứng minh rằng EG = FH và EG  FH .
Giải
Ta có
G
a - e = i (b - e), b - f = i (c - f )
D
H C
c - g = i (d - g ), d - h = i (a - h)
Từ đó suy ra A F
a - bi b - ci c - di d - ai B
e= , f = , g= , h=
1- i 1- i 1- i 1- i E
Ta cần chứng minh g - e = i ( f - h). Thật vậy

i é bi + c - di - a 1 é
i ( f - h) = (b - ci )- (d - ai )ùû=
ë = (c - di )- (a - bi )ùû= g - e . W
1- i 1- i 1- i ë

18
SỐ PHỨC VỚI HÌNH HỌC PHẲNG – Lê Thị Cẩm Thủy – THPT chuyên Nguyễn Đình Chiểu (Đồng Tháp)
Bài 2.1. (Định lí Shainer) Chứng minh rằng giao điểm của hai đường chéo một hình thang, giao điểm hai
cạnh bên và những trung điểm của hai cạnh đáy, nằm trên cùng một đường thẳng.
Giải
Kí hiệu hình thang ABCD. Giao điểm hai đường chéo là Q; O
trung điểm AB và CD lần lượt là M và N. Chọn gốc O là giao
điểm của hai cạnh bên. D N C
Vì AB P CD nên d = l a, c = l b với l là một số thực tùy ý
Q
a+ b
Khi đó O, M, N nằm trên một đường thẳng, vì m = ,
2
A M B
c+ d a+ b
n= =l . Mặt khác, ta có
2 2
l 1 l 1 l
q= a+ c= b+ d= (a + b).
1+ l 1+ l 1+ l 1+ l 1+ l
Điều này chứng tỏ giao điểm Q của AC và BD nằm trên đường thẳng OM.
Bài 2.2. Cho hình thang ABCD, đáy lớn AB. M là trung điểm cạnh BC và P là giao điểm của AM và
đường thẳng song song với BC xuất phát từ D. Chứng minh rằng, nếu AP = PM thì những

đường thẳng AC, DP, MN cắt nhau tại một điểm, với N là trung điểm cạnh AD.
Giải
Chọn hệ tọa độ với điểm gốc O là giao điểm của hai cạnh bên O
AD và BC. Do DP P OM và P là trung điểm AM nên D là

trung điểm OA và OA = 2OD . Do đó a = 2d , b = 2c , D C

b + c 3c a + m 4d + 3c M
m= = , p= = N
2 2 2 4 X
P
Gọi X là giao của AC với đường trung bình của hình thang thì
A B
a + c 2d + c
x= = .
2 2
4d + 3
d+ 2
2d + c 4 = d + 2p .
Mặt khác x = =
2 3 3
Vậy X nằm trên đường thẳng DP.
Bài 2.3. Qua điểm M nằm trong hình bình hành ABCD ta kẻ các đường thẳng song song với các cạnh của
chúng. Chúng cắt các cạnh AB, BC, CD, DA lần lượt tại các điểm P, Q, R, S. Chứng minh rằng
nếu đường thẳng PQ và RS cắt nhau, thì giao điểm của chúng nằm trên AC.
Giải

19
SỐ PHỨC VỚI HÌNH HỌC PHẲNG – Lê Thị Cẩm Thủy – THPT chuyên Nguyễn Đình Chiểu (Đồng Tháp)
Chọn hệ tọa độ với điểm gốc O tại A. Đặt D R C

p s
l = , m= . Khi đó
b d S
M Q
q = b + s = b + md , r = d + p = d + l b . H
A
P B

Gọi H là giao điểm của PQ và RS, thì


l m(b + d ) l m- 1 m l - 1
h= = q+ p= r+ s
l + m- 1 l + m- 1 l + m- 1 l + m- 1 l + m- 1
Ta thấy H nằm trên AC. Tất nhiên l + m- 1 ¹ 0. Nếu ngược lại thì PS và QR sẽ song song nhau và

nó không cắt nhau. W


Bài 2.4. Về phía ngoài của tam giác ABC vẽ các hình vuông ABEF và ACGH, N là trung điểm FH, M là
giao điểm của các đường BG và CE. Chứng minh rằng các điểm N, M, A thẳng hàng.
Giải G

Chọn hệ tọa độ có gốc tại A. Ta có: a = 0 , h = ic , g = c (1 + i ) , M


H
i (c - b) C
f = bi , e = b (1- i ), n = .
2 B
A
Gọi M là giao điểm của EC và BG
N
( )
EC : (c - b (1+ i ))m - (c - b (1- i ))m + c (b (1+ i ))- cb (1- i ) = 0

( )
BG : (b - c (1+ i ))m - (b - c (1- i ))m + b (c (1+ i ))- bc (1- i ) = 0
F E
Hoặc

m m m m
= Þ = .
i (b - c) i (b - c) n n
2 2
và a = 0 suy ra M, A, N nằm trên một đường thẳng. W
Bài 2.5. Cho hình thang ABCD đáy lớn AB. Kẻ đường thẳng song song với hai đáy, lần lượt cắt các cạnh
bên BC, AD tại N và M, cắt các đường chéo AC, BD tại P và Q. Chứng minh rằng MP = QN.
Giải
D C
AM AP BQ BN
Do MN P AB, MN P DC nên = = = =l
MD PC QD NC
N M
a+ l d b+ l c a+ l c b+ l d
Vì vậy m = , n= , p= , q= P Q
1+ l 1+ l 1+ l 1+ l
A B

l (c - d )
Suy ra p - m = = n - q . Vậy MP = QN.
1+ l

20
SỐ PHỨC VỚI HÌNH HỌC PHẲNG – Lê Thị Cẩm Thủy – THPT chuyên Nguyễn Đình Chiểu (Đồng Tháp)
AM DN
Bài 2.6. Trên cạnh AB và DC của tứ giác ABCD lấy điểm M và N sao cho = = k . Các điểm
AB DC
P, Q, S lần lượt là trung điểm của các đoạn thẳng AD, BC, MN. Chứng minh rằng P, Q, S cùng
PS
nằm trên một đường thẳng và = k.
PQ
Giải
k AM DN a+ d b+ c C
Đặt l = , thì = = l , p= , q= , D N
1- k MB NC 2 2
Q
P S
a+ l b d+l c a + d + l (b + c)
m= , n= , s=
1+ l 1+ l 2 + 2l
A M B

p - s a + d + l (a + d )- (a + d )- l (b + c)
V ( p, q, s ) = = =-l
q- s b + c + l (b + c)- (a + d )- l (b + c)

PS PS
là một số thực, vậy P, S, Q nằm trên một đường thẳng và = l hay = k. W
SQ PQ
Bài 2.7. Cho tam giác ABC lấy K là trung điểm AB, từ K kẻ hai đường song song lần lượt với trung
tuyến AA1 , BB1 cuả tam giác ABC và cắt các cạnh AC tại M, BC tại N. Chứng minh rằng các

đường trung tuyến AA1 , BB1 chia đường thẳng MN thành ba phần bằng nhau.

Giải
a+ c b+ c a+ b 3a + c C
Ta có b1 = , a1 = , k= , m= ,
2 2 2 4
B1 A
3b + c 1
n= . Gọi P, Q lần lượt là giao điểm của AA1 , BB1 với M N
4
MN. Khi đó A K B

b+ c
+a
2 b + c + 2a a + c + 2b
p= = , q=
2 4 4

3 1 1 1
m- n = (a - b), m - p = (a - b), n - q = - (a - b), p - q = (a - b).
4 4 4 4
Suy ra PQ P AB . Và P, Q chia MN thành ba phần bằng nhau. W

Bài 2.8. Cho A1 , B1 , C1 là trung điểm các cạnh BC, CA, AB của tam giác ABC, M là một điểm trong

tam giác ABC, lấy M1 , M 2 , M 3 lần lượt là các điểm đối xứng của M qua các điểm A1 , B1 , C1 .

Chứng minh rằng những đường thẳng AM1 , BM 2 , CM 3 cắt nhau tại một điểm.

21
SỐ PHỨC VỚI HÌNH HỌC PHẲNG – Lê Thị Cẩm Thủy – THPT chuyên Nguyễn Đình Chiểu (Đồng Tháp)
Giải
C
b+ c a+ c a+ b
Ta có a1 = , b1 = , c1 = , M2 M
1
2 2 2
B1 A
1
m1 = b + c - m , m2 = a + c - m , m3 = a + b - m .
M
Do đó a - m2 = m - c = b - m1 , m2 - m1 = a - b . Suy ra A C1 M3 B

AM 2 P BM1, M1M 2 P AB .

Hay ABM1M 2 là hình bình hành. Nên AM1 , BM 2 cắt nhau tại trung điểm M ' của mỗi đường.

a + b+ c- m
Do đó m ' = . Vì a, b, c có vai trò như nhau nên M ' là giao của ba đường thẳng. W
2
Bài 3.1. Trên một đường tròn có hai cung AB và CD. Nếu M là giao điểm của các đường vuông góc dựng
từ A đối với AB và từ C đối với CD, còn N là giao điểm của các đường vuông góc dựng từ B đối
với AB và từ D đối với CD. Chứng minh rằng MN đi qua giao điểm của hai đường thẳng BC và
AD, nếu nó cắt nhau hoặc MN song song với BC và AD, nếu chúng song song.
Giải
Chọn hệ tọa độ vuông góc sao cho đường tròn làm đường tròn N

đơn vị. B

- abc + abd + acd - bcd D


m= C
cd - ab
abc - abd - acd + bcd
n=
cd - ab M A
Suy ra m + n = 0 , nghĩa là M, N đối xứng qua O.

(a + d )bc - (b + c)ad
Gọi P = BC Ç AD . Khi đó p = và
bc - ad
P
n bc - ad
= N
p cd - ab
B D
1 1
-
n bc - ad bc ad = bc - ad = n .
Có = =
p cd - ab 1 1 cd - ab p C
-
cd ab
Suy ra N, P,O thẳng hàng hay P nằm trên đường thẳng MN
bc M
Nếu BC P AD thì bc = ad Û d = . Khi đó A
a
bc bc bc
abc - ab - ac + bc
n= a a a = bc - ac
bc a+ c
c - ab
a
22
SỐ PHỨC VỚI HÌNH HỌC PHẲNG – Lê Thị Cẩm Thủy – THPT chuyên Nguyễn Đình Chiểu (Đồng Tháp)
1 1
-
n bc - ac n bc ac bc - ac n
= và = = =
b - c (a + c)(b - c) b- c æ öæ1 1 ö
çç 1 + 1 ÷ ç ÷ (a + c)(b - c) b - c
÷ ÷
÷ççb - c ø
÷
èç a c øè
Suy ra NO P BC hay NM P BC P AD . W
Bài 3.2. Trên đường tròn cho 4 điểm ABCD sao cho AB là đường kính, còn CD thì không. Đường thẳng
AC và BD cắt nhau tại điểm X. Từ mỗi điểm C và D dựng các đường tiếp tuyến với đường tròn và
chúng cắt nhau tại Y. Chứng minh rằng đường thẳng XY vuông góc với AB và đi qua giao điểm
của hai đường thẳng AD và CB.
Giải
Chọn hệ tọa độ vuông góc sao cho đường tròn làm đường tròn
đơn vị và đường kính AB nằm trên trục hoành. Khi đó
(- 1+ c)d + (1+ d )c 2cd + c - d Y
a = - 1 , b = 1, x = = ,
d+ c c+ d D

2cd d- c C
y= . Có y- x = và
c+ d d+ c X
A B
1 1
-
d- c d c = - d - c = - ( y - x)
y- x = =
d+ c 1+ 1 d+ c
d c
Suy ra y - x hoàn toàn phức hay XY vuông góc trục hoành. Suy ra XY ^ AB .

d- c d- c
Từ =- . Suy ra giao điểm của AD và BC cũng nằm trên đường thẳng XY. W
d+ c d+ c
Bài 3.3. Trên đường tròn k kẻ hai đường kính AB và CD. M là điểm bất kì trên đường tròn k, còn P và Q
là chân đường vuông góc hạ từ M xuống AB và CD. Chứng minh rằng độ dài đoạn PQ không phụ
thuộc vào vị trí của M trên đường tròn.
Giải
Chọn hệ tọa độ vuông góc sao cho đường tròn k làm đường M
D
tròn đơn vị. Khi đó b = - a , d = - c ,
Q
1 1
p=
2
(a - a + m + a 2 m) = (m + a 2 m) ,
2
A
P
B

1 1
q=
2
(c - c + m + c 2 m) = (m + c 2 m).
2 C

1 1
Suy ra p - q = m (a 2 - c 2 ) = (a 2 - c 2 )
2 2
Vậy độ dài đoạn PQ không phụ thuộc vào vị trí của M trên đường tròn. W

23
SỐ PHỨC VỚI HÌNH HỌC PHẲNG – Lê Thị Cẩm Thủy – THPT chuyên Nguyễn Đình Chiểu (Đồng Tháp)
Bài 3.4. Từ trung điểm M của dây cung PQ trong một đường tròn, dựng hai dây cung bất kì AB và CD.
Dây cung AD và BC cắt PQ lần lượt tại X và Y. Chứng minh rằng M là trung điểm của đoạn XY.
Giải
Chọn hệ tọa độ vuông góc sao cho đường tròn làm đường tròn
D B
đơn vị và PQ song song trục hoành. Ta có m hoàn toàn ảo,

m = - m , pq = - 1 (vì nếu p = cos j + i sin j thì

q = - cos j + i sin j ).
P X M Y Q
M Î AB Þ a + b = m + abm A C

Þ abm = a + b - m
M Î CD Þ c + d = m + cd m

Þ cd m = c + d - m
( p + q)ad - (a + d ) pq 2mad + a + d
x= =
ad - pq ad + 1
Tương tự
2mbc + b + c
y=
bc + 1
Xét
mad + a + d - m 2mbc + b + c - m
x + y - 2m = x - m + y - m = +
ad + 1 bc + 1
(mcd + c + d - m)+ (mab + a + b - m) 0+ 0
= = = 0.
(ad + 1)(bc + 1) (ad + 1)(bc + 1)
Vậy M là trung điểm của đoạn XY. W
Bài 3.5. Cho tứ giác ABCD nội tiếp đường tròn. Các tiếp tuyến tại D

A và C cắt nhau tại M; các tiếp tuyến tại B và D cắt nhau


tại N. Chứng minh rằng các điểm A, C, N nằm trên một
đường thẳng khi và chỉ khi các điểm B, D, M nằm trên A C N
một đường thẳng.
B
Giải
Chọn hệ tọa độ vuông góc sao cho đường tròn ngoại tiếp tứ
2ac
giác ABCD làm đường tròn đơn vị. Khi đó m = ,
a+ c
2bd
n= . M
b+ d

24
SỐ PHỨC VỚI HÌNH HỌC PHẲNG – Lê Thị Cẩm Thủy – THPT chuyên Nguyễn Đình Chiểu (Đồng Tháp)
n- a n- a
A, C, N nằm trên một đường thẳng khi và chỉ khi = .
c- a c- a
2bd 2bd 1
- a -
n- a n- a
Có = Û b+ d = b + d a Û 2(ac + bd )= (a + c)(b + d ).
c- a c- a c- a 1 1
-
c a
Biểu thức trên không đổi khi a = b và c = d . Do vậy điều kiện cần và đủ để A, C, N nằm trên một
đường thẳng có thể thay bằng B, D, M nằm trên một đường thẳng. W
Bài 3.6. Cho hình bình hành ABCD. Dựng đường tròn đường kính AC. Gọi M và N là những điểm cắt thứ
hai của đường tròn vừa dựng với đường thẳng AB và AD. Chứng minh rằng đường thẳng BD, MN
và đường tiếp tuyến với đường tròn tại C cắt nhau tại một điểm.
Giải
Chọn hệ tọa độ vuông góc có điểm gốc là giao của hai đường
chéo hình bình hành và đường tròn làm đường tròn đơn vị. Khi
đó, a = - 1 , c = 1 , b + d = 0 .
N D
1+ b
M Î AB Þ a + m = b + amb Þ m =
1+ b
1 + d 1- d
N Î AD Þ a + m = d + amd Þ n = = A C
1 + d 1- d
MN: m + n = z + mnz
M
Thay m, n vào phương trình MN ta được
B
2
(1- b )z + (1- b )z = 2(1- bb).
2

Tiếp tuyến tại C có phương trình z + z = 2 Þ z = 2 - z . Thay vào phương trình MN ta tìm được
giao điểm của MN và tiếp tuyến tại C

2 (b 2 - bb) 2
z= 2
= b.
b2 - b b+ b

Số b + b là số thực, suy ra giao điểm của MN và tiếp tuyến tại C nằm trên đường thẳng OB. Do vậy
DB cũng đi qua giao điểm hai đường trên. W
Bài 3.7. Cho P là một điểm trên nửa đường tròn S với đường kính AB. Lấy hai cung bằng nhau trên S là
» và CD
BC » . Chứng minh rằng nếu AC và BP cắt nhau tại E, AD và CP cắt nhau tại F, thì EF
vuông góc với AD.
Gợi ý

25
SỐ PHỨC VỚI HÌNH HỌC PHẲNG – Lê Thị Cẩm Thủy – THPT chuyên Nguyễn Đình Chiểu (Đồng Tháp)
Chọn hệ tọa độ có gốc là tâm nửa đường tròn và đường tròn
D
đó làm đơn vị, a = - 1 , b = 1 . Khi đó bd = c 2 .
F C
Tìm các nhãn của E, F. Ta cần chứng minh P

f- e f- e E
=- . A B
d- a d- a S

Bài 3.8. Tại hai đầu đường kính AB của đường tròn dựng hai tiếp tuyến t A và t B . Qua điểm C trên t A ,

không trùng với A dựng hai cung bất kì D1E1 và D2 E2 đối với đường tròn. Chứng minh rằng

các tia AD1 , AD2 cắt đường thẳng t B thành một đoạn thẳng có độ dài bằng độ dài đoạn thẳng do

hai tia AE1 , AE2 tạo ra trên t B .

D2
D1

E2

A B

E1

Bài 3.9. Trên nửa đường tròn đường kính AB lấy hai điểm bất kì C và D. Những điểm P, Q, R là trung
điểm của những đoạn AC, CD, BD. Qua mỗi điểm P, Q lần lượt vẽ các đường thẳng vuông góc
với AQ, BQ, chúng cắt các tiếp tuyến tại A, B tại các điểm T, S. Chứng minh rằng ST và CD song
song.
T
D
Q
C

S R
P

A
B

26
THPT Chuyên Hoàng Lê Kha

MINH HỌA CÁC PHƯƠNG PHÁP


CHỨNG MINH ĐƯỜNG ĐI QUA ĐIỂM CỐ ĐỊNH
Nguyễn Hoài Phúc
Trường THPT Chuyên Hoàng Lê Kha – Tây Ninh

I. Mở đầu
Trong những năm gần đây, bài toán chứng minh đường đi qua điểm cố định
xuất hiện nhiều trong các đề thi Olympic 30 – 4, học sinh giỏi vòng quốc gia... Thực tế
cho thấy học sinh thường vấp phải nhiều khó khăn khi giải quyết dạng toán này. Chuyên
đề được viết với mục đích là muốn các em hệ thống lại kiến thức, ứng dụng các phương
pháp đã được học như: phương pháp biến hình, phương pháp tọa độ, phương pháp vectơ
và phương pháp tổng hợp để giải dạng toán này, nhằm cho các em có một cách nhìn toàn
diện về bài toán, giúp các em tiếp cận nó tốt hơn và mở rộng tư duy.

II. Nội dung


1. Vận dụng phương pháp tọa độ

Ví dụ 1. (Poland 1992) Trong mặt phẳng cho trước hai điểm A, B. Xét điểm C
thay đổi trên một nửa mặt phẳng bờ AB. Dựng ra ngoài của tam giác ABC các hình
vuông ACED và BCFG. Chứng minh rằng đường thẳng DG luôn đi qua một điểm cố
định khi C thay đổi.

Giải

Chọn hệ trục Axy sao cho A(0;0) ; B(b;0) và C x0 ; y0 , với y0 > 0. Khi đó

D y0 ; x0 , G b y0; b x0 . y
F

Vậy DG b 2 y0 ; b 2 x0 và do đó E G
I
đường thẳng DG có phương trình :
D C
x y0 x x0
b 2 y0 b 2 x0 x
A B

Hay
b 2 x x0 b 2 y y0 b x y 0 (1)

Tổng hợp các phương pháp chứng minh đường đi qua điểm cố định Trang 1
THPT Chuyên Hoàng Lê Kha

Từ đó suy ra, DG luôn đi qua điểm cố định khi và chỉ khi (1) có nghiệm với mọi x0, y0

b 2x 0 x
b
b 2y 0 2 . Vậy DG luôn đi qua I b ; b cố định.
b 2 2
x y 0 y
2

Ví dụ 2. (Đề thi HSG quốc gia 2007-2008) Cho tam giác ABC, trung tuyến AD.
Cho đường thẳng (d) vuông góc với đường thẳng AD. Xét điểm M trên (d). Gọi E, F lần
lượt là trung điểm của MB và MC. Đường thẳng đi qua E và vuông góc với (d) cắt đường
thẳng AB tại P, đường thẳng đi qua F và vuông góc với (d) cắt đường thẳng AC tại Q.
Chứng minh rằng đường thẳng đi qua M và vuông góc với PQ luôn đi qua một điểm cố
định, khi M di động trên (d).
Giải
Chọn hệ trục như hình vẽ O D , Oy DA .

Khi đó Ox song song (d), A(0;a),


P
B(b; c) , C  b; c  , M ( xM ; d ) y M
Phương trình đường thẳng
AB : (a c)x by ab 0 A
F
AC : (a c)x by ab 0
R
Gọi d1 , d2 là các đường thẳng vuông góc E

với d và lần lượt đi qua E, F thì : C x


D
x b xM b Q
d1 : x  M và d2 : x B
2 2

xM b (a c)( xM b)
P = d1  AB  P ;a
2 2b

xM b (a c)( xM b) a.xM bc
Q = d2  AC  Q ;a . Suy ra : PQ b;
2 2b b

Phương trình đường thẳng  đi qua M(xM ; h) và vuông góc PQ là :

axM bc bc b2
b( x xM ) (y h) 0 hay b2 x (axM bc) y h 0.
b a a

bc b2
Vậy  đi qua điểm cố định R ;h khi M di động trên d.
a a
Tổng hợp các phương pháp chứng minh đường đi qua điểm cố định Trang 2
THPT Chuyên Hoàng Lê Kha

Ví dụ 3. (IMO – Shortlist) Cho ba điểm thẳng hàng theo thứ tự A, B, C. Một


đường tròn (I) thay đổi qua A và C nhưng không nhận AC làm đường kính. Hai tiếp
tuyến của (I) tại A và C cắt nhau tại D, BD cắt (I) tại E và F. Chứng minh rằng các đường

phân giác trong của các góc AEC; AFC luôn đi qua một điểm cố định.

Giải
Chọn hệ trục tọa độ như hình vẽ. Khi đó ta xây dựng tọa độ của các điểm như sau:

O(0; 0); A( 1; 0); C(1; 0); B(b; 0); I (0; - p) , phương trình (I): x2 (y p)2 1 p2

1
và do đó D 0; và M 0; p 1 p2 là điểm chính giữa của cung AB không chứa
p

điểm E.
x 1
Ta có (DB) : y và E là một giao điểm của DB và (I) nên ta có tọa độ của E
bp p

b(1 p2 ) bp (1 p2 )(1 b2 ) p(1 b2 ) (1 p2 )(1 b2 )


là: E ;
1 b2 p 2 1 b2 p 2

ED 1 p2
Từ đó ta có:
EB p 1 b2

y
D

A C x
O K B

Tổng hợp các phương pháp chứng minh đường đi qua điểm cố định Trang 3
THPT Chuyên Hoàng Lê Kha

MO p 1 p2 p
Mặt khác:
MD 1 1 p2
p 1 p2
p
Gọi K là giao của đường phân giác trong của góc AEC với AC thì ta có E, K, M thẳng

KO EB MD
hàng. Áp dụng định lý Menelaus cho tam giác DOB ta được: . . 1
KB ED MO

KO MO ED p 1 p2 1
Từ đó ta có: . .
KB MD EB 1 p2 p 1 b2 1 b2

Điều này chứng tỏ điểm K không phụ thuộc vào sự thay đổi của đường tròn (I). Do cách
lấy tọa độ điểm E là tổng quát nên đối với điểm F ta cũng có kết quả tương tự.
Vậy hai đường phân giác cùng đi qua điểm K cố định trên AC.
Ví dụ 4. Cho tam giác ABC vuông tại A nhưng không cân; trên cạnh AB và AC
lấy M và N sao cho BM = CN. Chứng minh rằng đường trung trực của MN luôn đi qua
một điểm cố định.
Giải
Dựng hệ trục tọa độ như hình vẽ.
Chọn A(0; 0); M(0; m); B(0; b) ; C(1; 0) y
với m thay đổi, 0 < m < b  1
B
Ta có MB = CN, suy ra N(1 + m – b ;0).
M
Suy ra trung điểm của MN có tọa độ

1 m b m
;
2 2 I
x
và MN 1 m b; m A N C

Suy ra phương trình đường trung trực của

1 m b m
MN là: 1 m b x m y 0
2 2

1 2
m x y 1 b 1 b x 1 b 0
2
1 b b 1
Ta thấy đường thẳng này luôn đi qua điểm cố định I ; . Và đó cũng chính là
2 2
điểm chính giữa của cung BAC trên đường tròn ngoại tiếp tam giác ABC.

Tổng hợp các phương pháp chứng minh đường đi qua điểm cố định Trang 4
THPT Chuyên Hoàng Lê Kha

2. Vận dụng phương pháp vectơ


Ví dụ 1. Cho các đường thẳng xx’ và yy’ cắt nhau tại O và hai số dương a, b. Các

điểm A, B theo thứ tự thay đổi trên xx’ và yy’ sao cho aOA bOB 1 . Chứng minh
rằng đường tròn ngoại tiếp tam giác OAB luôn đi qua một điểm cố định khác O.
Giải
Gọi I là tâm của đường tròn ngoại tiếp tam giác OAB; H, K là trung điểm của OA và OB;
e1, e2 là vectơ đơn vị trên x’Ox, y’Oy. Khi đó ta có:

OI .(ae1 be2 ) a.OI .e1 b.OI .e2 a. OH b.OK

1 1
a.OA b.OB
2 2

Dựng OT ae1 be2 OT cố định. Gọi I ' là hình chiếu của I trên đường thẳng OT.

1 1
Khi đó: OI .OT OI '.OT OI '.OT OI ' hay I ' là điểm cố định.
2 2.OT
Vậy I luôn thuộc đường thẳng ( ) (cố định) vuông góc OT tại I ' . Lấy đối xứng O qua
( ) được O ' thì ta có O ' cố định và thuộc đường tròn ngoại tiếp tam giác OAB.

Ví dụ 2. Cho góc xOy và hai số dương a, b. Các điểm A, B theo thứ tự thay đổi

a b
trên Ox, Oy sao cho: 1 . Chứng minh đường thẳng AB luôn đi qua một điểm
OA OB
cố định.
Giải
Trên Ox, Oy theo thứ tự lấy các điểm X, Y sao cho OX a; OY b.
Dựng hình bình hành OXKY. Do O, X, Y cố định nên K cố định.
OX OY
Ta có: OK OX OY .OA .OB
OA OB
a b a a
.OA .OB .OA 1 .OB
OA OB OA OA

a a
OK OB OA OB BK .BA
OA OA
Điều này chứng tỏ A, B, K thẳng hàng hay đường thẳng AB luôn đi qua điểm K cố định.

Tổng hợp các phương pháp chứng minh đường đi qua điểm cố định Trang 5
THPT Chuyên Hoàng Lê Kha

Ví dụ 3. Cho tam giác ABC nội tiếp đường tròn (O) (BAC 90 ) . Điểm E chạy
trên (O) (E khác B, C). AE theo thứ tự cắt các tiếp tuyến với (O) tại B, C ở M, N.
F BN CM . Chứng minh rằng EF luôn đi qua một điểm cố định.
Giải

Vì BAC 90 nên tiếp tuyến với (O) tại B, C cắt nhau, gọi S là giao điểm của chúng.
Đặt K AS BC. Bỏ qua trường hợp đơn giản AE // BC.
Khi AE, BC không song song, đặt L AE BC. Lấy P trên BC sao cho NP // MB.
Theo hệ quả 2 của định lý Thales dạng đại số, chú ý rằng NP = NC, ta có:
2 2 2 2
MB NB MB NB MB NC
.
ML NL ML NL ML NL

ME.MA NA.NE ME NE NA MA
Từ đó, ta có: 2 2
: : .
ML NL ML NL NL ML

(MNEL) ( NMAL) (MNEL) (CBKL) (xét phép chiếu xuyên tâm S).
Từ đó, MC, NB, EK đồng quy. Suy ra EK đi qua F. Điều đó có nghĩa là EF đi qua K.
Vậy EF luôn đi qua một điểm cố định (điểm K).

3. Vận dụng phương pháp biến hình


Ví dụ 1. Cho hai đường tròn (O1 ),(O2 ) cắt nhau tại A, B. Đường thẳng quay

quanh B và theo thứ tự lại cắt (O1 ),(O2 ) tại C, D. M là trung điểm của CD. AM lại cắt

(O2 ) tại P. Đường thẳng qua M và vuông góc với O1M cắt AC tại Q. Chứng minh rằng

đường thẳng PQ luôn đi qua một điểm cố định.


Tổng hợp các phương pháp chứng minh đường đi qua điểm cố định Trang 6
THPT Chuyên Hoàng Lê Kha

Giải
Gọi O3 là đường tròn ngoại tiếp tam giác ACP; K, N theo thứ tự là giao điểm thứ hai

của (O1 ),(O2 ) với AP, ;(O4 ) là đường tròn ngoại tiếp tam giác AND; S là giao điểm

thứ hai của PQ với đường tròn (O2 ).

Ta có:

MP.MA MB.MD MB.MC MK .MA


MN .MC MA.MP MB.MD MB.MC

Do đó MP MK; MN MB.

Kết hợp với MD MC suy ra phép đối xứng tâm M theo thứ tự biến A, B, C thành P,
N, D.
Điều đó có nghĩa là phép đối xứng tâm M biến O1 thành O4 .

Kết hợp MQ O1O2 , suy ra MQ là trục đẳng phương của các đường tròn O1 , O4 .

Từ đó, chú ý rằng AQ và NP theo thứ tự là trục đẳng phương của các cặp đường tròn
O1 , O3 và O3 , O4 suy ra Q thuộc NP.

Vậy (BS, BA) (PS, PA) (mod ) (vì P thuộc (BSA)).


(PN, PA) (mod ) (vì PS = PQ = PN)
(BK, AK ) (mod ) (vì PN // BK, PA = AK).

Do đó BS tiếp xúc với (O1 ). Suy ra S cố định (đpcm).

Tổng hợp các phương pháp chứng minh đường đi qua điểm cố định Trang 7
THPT Chuyên Hoàng Lê Kha

Ví dụ 2. Cho tam giác ABC. A’, B’, C’ theo thứ tự là trung điểm của BC, CA, AB.
Các điểm P, P’ thay đổi sao cho PA = P’A’, PB = P’B’, PC = P’C’. Chứng minh rằng
đường thẳng PP’ luôn đi qua một điểm cố định.
Giải
Gọi G là trọng tâm tam giác ABC, (E) = (A’B’C’) ( đường tròn Euler (E) của tam giác
1
ABC). Qua phép vị tự V O; , A, B, C theo thứ tự biến thành A’, B’, C’.
2
1
Đặt V O; (P) Q.
2
1 EP 1
Chú ý rằng V O; (O) E, ta có EQ // OP và (1).
2 OQ 2

1
AP B'Q 1 C 'Q 1
A'Q 2 1
Theo giả thiết . Tương tự ; .
A' P ' A' P ' 2 B'P' 2 C'P' 2
1
Vậy đường tròn (E) chính là đường tròn Apollonius xác định bởi hai điểm Q, P’ và số .
2
Do đó E thuộc đường thẳng QP’.

XQ 1 YQ 1
Gọi X, Y là giao điểm của (E) với QP’. Ta có: ; .
XP ' 2 YP ' 2
Coi XY là một trục với gốc là E. Giả sử X, Y, Q, P’ có tọa độ x, y, p, q’.
q x 1 q y 1
Ta có: x y 0; ; . Suy ra
p' x 2 p' y 2

q x 1 q y
x y 0; ; 1.
p' q 3 p' q

Tổng hợp các phương pháp chứng minh đường đi qua điểm cố định Trang 8
THPT Chuyên Hoàng Lê Kha

2q q x q y 1 2
Do đó 1 .
p' q p' q 3 3

q 1
Từ đó dễ dàng suy ra . (2). Gọi S là giao điểm của PP’ và OE.
p' 4

OS OP OP EQ OG q 1 1
Từ (1) và (2) suy ra . . ( 2). .
OE EP ' OQ EP ' EG p ' 4 2

Do đó S cố định.
Ví dụ 3. ( VMO 2013) Cho tam giác không cân ABC. Kí hiệu (I) là đường tròn
tâm I nội tiếp tam giác ABC và D, E, F là các tiếp điểm của (I) với BC, CA, AB. Giả sử
AB
B, C cố định, A thay đổi sao cho tỷ số  k (không đổi). Gọi M, N tương ứng là các
AC
giao điểm IE, IF với (I) (M khác E, N khác F). MN cắt IB, IC tại P, Q. Chứng minh
đường trung trực PQ luôn đi qua một điểm cố định.
Giải

E B'
S
C' F
I
Q
N
P T
M C
B H' A' H

Kẻ AA’ là tia phân giác trong tại A của tam giác ABC. A’ thuộc BC và A’ cố định (gt).
Gọi B’ và C’ lần lượt là giao điểm của BI với EF và CI với EF.

BAC ABC ACB


Ta có : B ' EC AEF 90 CIB '
2 2 2
Do đó EICB’ nội tiếp IB' CB'
Tương tự ta cũng suy ra được : IC' BC' . Từ hai điều này ta suy ra (BC) ngoại
tiếp tứ giác BCB’C’.

Tổng hợp các phương pháp chứng minh đường đi qua điểm cố định Trang 9
THPT Chuyên Hoàng Lê Kha

HS B'C'
Gọi S, H, T là trung điểm B’C’, BC, PQ thì IA ' HS (1)
IA ' EF

Gọi (d) là đường trung trực của PQ và H’ là giao điểm của BC và (d), khi đó HH’TS
là hình thang. Dễ thấy PQ là ảnh của B’C’ qua phép đối xứng tâm I nên T cũng là ảnh
của S qua phép đối xứng tâm I hay I là trung điểm của ST (2).
Từ (1) và (2) suy ra IA’ là đường trung bình của hình thang HH’TS suy ra A’ là
trung điểm của HH’ mà H, A’ cố định nên H’ cố định.
Vậy đường trung trực d luôn đi qua điểm H’ cố định.

4. Vận dụng phương pháp tổng hợp

Ví dụ 1.(VMO 2015) Cho đường tròn (O) và hai điểm B, C cố định trên (O), BC
không là đường kính. Một điểm A thay đổi trên (O) sao cho tam giác ABC nhọn. Gọi E,
F lần lượt là chân đường cao kẻ từ B, C của tam giác ABC. Cho (I) là đường tròn thay
đổi qua E, F và có tâm là I. Giả sử (I) cắt cạnh BC tại hai điểm M, N. Gọi H là trực tâm
tam giác ABC và P, Q là các giao điểm của (I) với đường tròn ngoại tiếp tam giác HBC.
Đường tròn (K) đi qua P, Q và tiếp xúc với (O) tại điểm T (T cùng phía A đối với PQ).
Chứng minh rằng đường phân giác trong của góc MTN luôn đi qua một điểm cố định.

Giải

Trường hợp tam giác ABC cân tại A, bài toán hiển nhiên đúng.
Xét trường hợp tam giác ABC không cân tại A, không mất tính tổng quát, giả sử
AB AC . Gọi G là giao điểm của đường thẳng EF và đường thẳng BC.

Tổng hợp các phương pháp chứng minh đường đi qua điểm cố định Trang 10
THPT Chuyên Hoàng Lê Kha

Xét các đường tròn (BHC), (I) và đường tròn đường kính BC. Ta có
Trục đẳng phương của (BHC) và (I) là PQ.
Trục đẳng phương của (BHC) và đường tròn đường kính BC là BC.
Trục đẳng phương của (I) và đường tròn đường kính BC là EF.

Do đó PQ, EF, BC đồng qui tại điểm G. ta có GT 2 GP.GQ GM .GN nên đường tròn
(TMN) cũng tiếp xúc với (O) tại T.

Từ đó, ta có GTM GNT (cùng chắn cung TM của (K)).

Mặt khác, ta lại có GNT NTC NCT . Hơn nữa, do GT tiếp xúc với (O) nên

GTB GCT . Từ đó, ta được BTM CTN.

Nên phân giác của góc MTN và BTC là trùng nhau.

Hay phân giác của góc MTN đi qua trung điểm của cung BC không chứa A.
Ta có điều cần chứng minh.
Ví dụ 2.(VMO 2006) Cho hình thang cân ABCD có CD là đáy lớn. Xét một điểm M di
động trên đường thẳng CD sao cho M không trùng với C và D . Gọi N là giao điểm
thứ hai khác M của đường tròn BCM và DAM . Chứng minh rằng:

a) Điểm N di động trên một đường tròn cố định.


b) Đường thẳng MN luôn đi qua một điểm cố định.
Giải
P

A B
N

D M C

a) * Nếu M nằm trên cạnh CD thì M và N ở cùng phía đối với đường thẳng AB. Từ
các tứ giác nội tiếp ANMD và BNMC ,

Ta có ANB 2 ANM BNM C D

Tổng hợp các phương pháp chứng minh đường đi qua điểm cố định Trang 11
THPT Chuyên Hoàng Lê Kha

* Nếu M nằm ngoài cạnh CD thì M và N ở khác phía đối với đường thẳng AB .

Từ các tứ giác nội tiếp ANMD và BNMC ta có ANB C D

Vậy N thuộc đường tròn cố định đi qua A , B và P (là giao điểm của AD và BC).
b) Gọi P AD BC thì P cố định và PA.PD PB.PC , suy ra P thuộc trục đẳng
phương của hai đường tròn BCM và DAM P MN .

Ví dụ 3.(VMO 2007) Cho hình thang ABCD có đáy lớn BC và nội tiếp đường tròn
O . Gọi P là điểm thay đổi trên BC và nằm ngoài đoạn BC sao cho PA không là tiếp

tuyến của đường tròn O . Đường tròn đường kính PD cắt O tại E E D . Gọi M

là giao điểm của BC với DE , N là giao điểm khác A của PA với O . Chứng minh
đường thẳng MN qua một điểm cố định.
Giải

A D
c1
N
O
M
B F C P

E
A'
c2

Gọi A ' là điểm đối xứng của A qua tâm O . Ta chứng minh N, M, A ' thẳng hàng, từ đó
suy ra MN đi qua A ' cố định.
Thật vậy, ta có DE là trục đẳng phương của đường tròn O và đường tròn C2 đường

kính PA ' . Giả sử DA ' cắt BC tại F , do DFP 900 PFA ' 900 nên BC là trục
đẳng phương của C1 và C2 . Tương tự ta cũng có N thuộc C2 nên NA ' là trục đẳng

phương của (O) và C2 . Vì các trục đẳng phương đồng quy tại tâm đẳng phương, suy ra
DE, BC và NA ' đồng quy tại điểm M . Vậy M, N, A ' thẳng hàng.

Tổng hợp các phương pháp chứng minh đường đi qua điểm cố định Trang 12
THPT Chuyên Hoàng Lê Kha

Ví dụ 4. (IMO – Shortlist) Cho hai đường tròn (S1) và (S2) cắt nhau tại hai điểm P
và Q. Gọi A, B là hai điểm phân biệt trên (S1) (khác P, Q). Các đường thẳng AP và BP
lần lượt cắt (S2) tại C và D và hai đường thẳng AB và CD cắt nhau tại E. Chứng minh
rằng khi A, B thay đổi thì đường trung trực của đoạn thẳng nối hai tâm đường tròn (AEC)
và (BED) luôn đi qua một điểm cố định.
Giải

B
A O
C
P

D
O1 O2

Trước tiên ta chứng minh (AEC) qua Q.

Ta có: AQC AEC AQP PQC AEC EBP EDP BED 180
Do đó Q thuộc (AEC).
Tiếp theo ta sẽ chứng minh (QO1O2) qua O với O1, O2, O lần lượt là tâm của các đường
tròn (S1), (S2) và (AEC).

OA OQ 1
Thật vậy, ta có: OO1Q AO1Q 180 APQ
O1 A O1Q 2

Tương tự thì OO2Q 180 CPQ . Do đó: OO1Q OO2Q 180

Từ đó suy ra : O thuộc (QO1O2).


Tương tự cho tâm O’ của đường tròn (BED) cũng thuộc (QO1O2). Do đó đường trung
trực của đoạn OO’ qua tâm của (QO1O2) mà Q, O1, O2 cố định nên tâm của (QO1O2) cũng
cố định.

Tổng hợp các phương pháp chứng minh đường đi qua điểm cố định Trang 13
THPT Chuyên Hoàng Lê Kha

BÀI TẬP ĐỀ NGHỊ

Câu 1. Cho tam giác ABC cân tại A. Xét D trên cạnh AB và điểm E trên cạnh BC

BC
sao cho hình chiếu của DE trên BC có độ dài bằng . Chứng minh rằng đường thẳng
2
vuông góc với DE tại E luôn đi qua một điểm cố định.
Câu 2. Cho đường tròn (O, R) và đường thẳng d không có điểm chung với (O). Từ
(O) hạ OH d tại H. Giả sử M là một điểm bất kỳ trên d. Từ M kẻ các tiếp tuyến MA,
MB đến đường tròn (O). Gọi K, I lần lượt là hình chiếu vuông góc của H xuống MA,
MB. Chứng minh rằng đường kính KI luôn đi qua một điểm cố định.
Câu 3. Cho đường tròn (O) và dây cung AB. Lấy điểm E trên dây cung AB (E khác
A và B). Qua E vẽ dây cung CD của đường tròn (O). Trên hai tia DA, DB lấy hai điểm P,
Q đối xứng qua E. Chứng minh rằng đường tròn (I) tiếp xúc với PQ tại E và đi qua C
luôn đi qua một điểm cố định khi E di động trên dây cung AB.
Câu 4. Cho tam giác ABC có hai điểm P, Q lần lượt thuộc cạnh AC và AB. Đường
tròn k qua các trung điểm của BP, CQ và PQ. Chứng minh rằng nếu P, Q thay đổi sao
cho k nhận PQ làm tiếp tuyến thì đường trung trực của PQ luôn đi qua một điểm cố định.
Câu 5. Cho góc vuông xOy. Trên Ox, Oy thứ tự có hai điểm A, B chuyển động sao
cho OA  OB  a (a là độ dài cho trước). Gọi G là trọng tâm tam giác OAB và (d) là
đường thẳng qua G vuông góc với AB. Chứng minh rằng (d) luôn đi qua một điểm cố
định.

Câu 6. Cho đường tròn (O, R) và dây cung AB R 3 . Điểm P thuộc đoạn AB
(khác A và B). Gọi (C, R1) là đường tròn đi qua P tiếp xúc với đường tròn (O, R) tại A,
(D, R2) là đường tròn đi qua P tiếp xúc với (O, R) tại B. Các đường tròn (C, R1) và
(D, R2) cắt nhau tại M khác P. Chứng minh rằng khi P di động trên AB thì đường thẳng
PM luôn đi qua một điểm cố định.
Câu 7. Cho tam giác ABC có đỉnh A cố định và B, C thay đổi trên đường thẳng d cố

định sao cho nếu gọi A’ là hình chiếu của A lên d thì AB. AC âm và không đổi. Gọi M,
N lần lượt là hình chiếu của A’ lên AB, AC. K là giao điểm của các tiếp tuyến của đường
tròn ngoại tiếp tam giác A’MN tại M và N. Chứng minh rằng đường tròn (K, KA) luôn đi
qua một điểm cố định khác A.
Câu 8. Cho đoạn AB cố định, M di động trên AB. Trên cùng một nửa mặt phẳng bờ

Tổng hợp các phương pháp chứng minh đường đi qua điểm cố định Trang 14
THPT Chuyên Hoàng Lê Kha

AB vẽ hai hình vuông MADE và MBHG. Hai đường tròn ngoại tiếp hai hình vuông cắt
nhau tại N. Chứng minh đường thẳng MN luôn đi qua một điểm cố định khi M di chuyển
trên AB.
Câu 9. Cho hai đường thẳng a, b cắt nhau tại O, điểm M không thuộc a, b và không
thuộc các đường phân giác của các góc tạo bởi a, b. Hai điểm A, B theo thứ tự thay đổi

trên a, b sao cho OMA OMB. Chứng minh rằng đường thẳng AB luôn đi qua một điểm
cố định.
Câu 10. Cho ba điểm thẳng hàng A, B, C theo thứ tự đó. Một đường tròn (O) thay đổi
nhưng luôn đi qua B và C. Từ A kẻ hai tiếp tuyến AM, AN đến đường tròn (O). Đường
thẳng MN cắt hai đoạn AO, AC lần lượt tại H và K. Chứng minh đường tròn ngoại tiếp
tam giác OHK luôn đi qua hai điểm cố định.
Câu 11. Cho đường tròn tâm O, dây AB. Điểm M di chuyển trên cung lớn AB. Các
đường cao AE, BF của tam giác ABM cắt nhau ở H. Đường tròn tâm H bán kính HM cắt
MA, MB theo thứ tự ở C, D.
a) Chứng minh rằng đường thẳng kẻ từ M vuông góc với CD luôn đi qua một điểm
cố định.
b) Chứng minh rằng đường thẳng kẻ từ H và vuông góc với CD cũng đi qua một
điểm cố định.
Câu 12. Cho tam giác ABC, M là điểm bất kì thuộc đường tròn (O) ngoại tiếp tam
giác ấy. Gọi D là điểm đối xứng với M qua AB, E là điểm đối xứng với M qua BC.
Chứng minh rằng khi điểm M di chuyển trên đường tròn (O) thì DE luôn đi qua một điểm
cố định.
Câu 13. Cho tam giác nhọn ABC có BC > CA. Điểm M chạy trên đoạn CA. Các
điểm A’, C’ theo thứ tự thuộc các đoạn BC, BA sao cho MC CA '; AM MC ' .
Chứng minh rằng đường tròn (A’BC’) luôn đi qua một điểm cố định.
Câu 14. Cho đường tròn tâm (O). Từ điểm A cố định ở ngoài (O) kẻ tiếp tuyến AB,
AC tới (O) (B, C tiếp điểm). Lấy điểm M trên cung nhỏ BC. Gọi D, E, F thứ tự là hình
chiếu từ M đến BC, AC, AB. Gọi MB cắt DF tại P, MC cắt DE tại Q. Chứng minh đường
thẳng nối giao điểm của hai đường tròn ngoại tiếp tam giác MPF và MQE luôn đi qua
một điểm cố định.

Câu 15. Cho tam giác ABC nội tiếp đường tròn (O). Điểm M chạy trên cung BC
không chứa A của (O). K, L theo thứ tự là tâm đường tròn nội tiếp các tam giác ABM,
ACM. Chứng minh rằng đường tròn (MKL) luôn đi qua một điểm cố định.

Tổng hợp các phương pháp chứng minh đường đi qua điểm cố định Trang 15
THPT Chuyên Hoàng Lê Kha

HƯỚNG DẪN

Câu 1. Cho tam giác ABC cân tại A. Xét D trên cạnh AB và điểm E trên cạnh BC sao

BC
cho hình chiếu của DE trên BC có độ dài bằng . Chứng minh rằng đường thẳng
2
vuông góc với DE tại E luôn đi qua một điểm cố định.
Hướng dẫn
Gọi O là trung điểm BC, đặt hệ trục xOy có Ox trùng BC và Oy trùng OA.
Giả sử : A(0; a); B( b; 0); C(b; 0) . Gọi H là hình chiếu của D trên BC thì H thuộc đoạn

BC
BO. Do EH nên E thuộc đoạn OC.
2
ax0
Giả sử E( x0; 0), 0 x0 b thì D x0 b; . Gọi ( ) là đường thẳng qua E vuông
b

góc DE thì ( ) : (b2 ay)x0 b2 x 0 . Điều này chứng tỏ ( ) luôn đi qua

b
K 0; cố định.
a

Câu 2. Cho đường tròn (O, R) và đường thẳng d không có điểm chung với (O). Từ (O)
hạ OH d tại H. Giả sử M là một điểm bất kỳ trên d. Từ M kẻ các tiếp tuyến MA, MB
đến đường tròn (O). Gọi K, I lần lượt là hình chiếu vuông góc của H xuống MA, MB.
Chứng minh rằng đường kính KI luôn đi qua một điểm cố định.
Hướng dẫn
Gọi J, T lần lượt là giao điểm của AB
với OH và OM. Ta có OM AB do A
MA, MB là các tiếp tuyến của đường
O T K
tròn (O). suy ra MTJ 900

Lại có MHJ 900 do OH HM J


I
MTJ MHJ 0
180 suy ra MTJH B M
L
nội tiếp OJ.OH OT.OM
N
mà OT.OM OA2 R2 tam giác H
OMA vuông tại A có AT là đường cao.

Tổng hợp các phương pháp chứng minh đường đi qua điểm cố định Trang 16
THPT Chuyên Hoàng Lê Kha

R2
suy ra OJ suy ra J cố định.
OH
Gọi N là hình chiếu vuông góc của H xuống AB, L là giao điểm của KI và OH.
Ta có năm điểm M, H, A, B cùng nằm trên đường tròn đường kính OM.
Do K, I, N là hình chiếu vuông góc của H xuống MA, MB, AB nên K, I, N thẳng hàng
(tính chất đường thẳng Simson của tam giác MAB tương ứng với H).

Vậy tứ giác HIBN nội tiếp do đó INH IBH

Mặt khác IBH MOH do tứ giác MOBH nội tiếp

Lại có HN OM cùng vuông góc AB nên MOH JHN INH JHN . Hơn nữa tam
giác JHN vuông góc tại N nên từ đó ta có L là trung điểm của JH. Do J, H cố định nên L
cố định. Vậy đường thẳng KI luôn đi qua điểm L cố định.
Câu 3. Cho đường tròn (O) và dây cung AB. Lấy điểm E trên dây cung AB (E khác A
và B). Qua E vẽ dây cung CD của đường tròn (O). Trên hai tia DA, DB lấy hai điểm P, Q
đối xứng qua E. Chứng minh rằng đường tròn (I) tiếp xúc với PQ tại E và đi qua C luôn
đi qua một điểm cố định khi E di động trên dây cung AB.
Hướng dẫn
Gọi M là giao điểm của AB và đườngtròn (I), EP là tiếp tuyến của (I), nên
C
CMA PEC QED (1)
P
Mặt khác BAC BDC (góc nội tiếp) (2)
I
Từ (1) và (2), suy ra tam giác CMA đồng dạng với
A B
M
tam giác QED (g. g) E
O
AM DE
(3) Q
CM QE
Chứng minh tương tự D

DEP BMC; ADC ABC , nên

tam giác BMC đồng dạng tam giác DEP (g. g)


BM DE DE
(4)
CM PE QE

AM BM
Từ (3) và (4) suy ra AM BM .
CM CM
Do đó đường tròn (I) luôn đi qua trung điểm M của AB là điểm cố định.
Tổng hợp các phương pháp chứng minh đường đi qua điểm cố định Trang 17
THPT Chuyên Hoàng Lê Kha

Câu 4. Cho tam giác ABC có hai điểm P, Q lần lượt thuộc cạnh AC và AB. Đường
tròn k qua các trung điểm của BP, CQ và PQ. Chứng minh rằng nếu P, Q thay đổi sao
cho k nhận PQ làm tiếp tuyến thì đường trung trực của PQ luôn đi qua một điểm cố định.
Hướng dẫn

Gọi K, L, M, B’, C’ lần lượt là trung điểm của BP, CQ, PQ, AC và AB.

Ta có: APQ PML MKL; AQP QMK MLK

BQ
AP MK 2 BQ
Nên APQ ∽ MKL do đó
AQ ML CP CP
2
AP.PC AQ.BQ
Gọi O là tâm đường tròn ngoại tiếp tam giác ABC thì:

OP2 OQ2 (OB '2 PB '2 ) (OC '2 QC '2 )


(OA2 AB '2 ) PB '2 (OA2 AC '2 ) QC '2
(AC '2 QC '2 ) (AB '2 PB '2 )
AQ. BQ AP.CP 0
Vậy OP OQ điều này chứng tỏ đường trung trực
của PQ luôn qua O mà tam giác ABC cố định nên O cố C

định.
Câu 5. Cho góc vuông xOy. Trên Ox, Oy thứ tự có hai
điểm A, B chuyển động sao cho OA+ OB = a ( a là độ n

dài cho trước). Gọi G là trọng tâm tam giác OAB và (d) A
I

là đường thẳng qua G vuông góc với AB. Chứng minh f

G
rằng (d) luôn đi qua một điểm cố định. O D
B

Tổng hợp các phương pháp chứng minh đường đi qua điểm cố định Trang 18
THPT Chuyên Hoàng Lê Kha

Hướng dẫn
Trên Ox, Oy thứ tự lấy 2 điểm C, D sao cho OC = OD = a.
Phân giác của góc xOy cắt CD tại N, cắt (d) tại I
dễ thấy tam giác NAO = tam giác NBD do đó NF vuông góc với AB
OG OI 2 2 1
Xét tam giác ONF có GI // NF OI ON a = hằng số
OF ON 3 3 3
Vậy I cố định hay (d) đi qua điểm cố định I.

Câu 6. Cho đường tròn (O, R) và dây cung AB R 3 . Điểm P thuộc đoạn AB (khác
A và B). Gọi (C, R1) là đường tròn đi qua P tiếp xúc với đường tròn (O, R) tại A, (D, R2)
là đường tròn đi qua P tiếp xúc với (O, R) tại B. Các đường tròn (C, R1) và (D, R2) cắt
nhau tại M khác P. Chứng minh rằng khi P di động trên AB thì đường thẳng PM luôn đi
qua một điểm cố định.
Hướng dẫn
Vẽ đường tròn ngoại tiếp tam giác OAB cắt PM tại I .

vì AB = R 3 => sđ AB của (O) bằng 1200

tam giác BDP cân do đó OBA  DPB


O
tam giác OAB cân do đó OBA OAB   M
C

BDP BOA sđ BP sđ BA 1200 . B


D
P A

Tương tự sđ PA 1200 .

ta có BMP sđ BP 600

ta có AMP sđ AP 600
I
Vậy BMA BMP AMP 1200 BOA
Ta có tứ giác BMOA nội tiếp hay M thuộc đường tròn

ngoại tiếp tam giác BOA và PM là phân giác của BMA


Vậy I là trung điểm cung AB (phần không chứa O) của (OAB), mà (OAB) cố định nên I
cố định.
Câu 7. Cho tam giác ABC có đỉnh A cố định và B, C thay đổi trên đường thẳng d cố

định sao cho nếu gọi A’ là hình chiếu của A lên d thì AB. AC âm và không đổi. Gọi M,
N lần lượt là hình chiếu của A’ lên AB, AC. K là giao điểm của các tiếp tuyến của đường

Tổng hợp các phương pháp chứng minh đường đi qua điểm cố định Trang 19
THPT Chuyên Hoàng Lê Kha

tròn ngoại tiếp tam giác A’MN tại M và N. Chứng minh rằng đường tròn (K, KA) luôn đi
qua một điểm cố định khác A.
Hướng dẫn
Gọi O là tâm đường tròn ngoại tiếp tam giác
A

A’MN và I là giao điểm của OK và MN. Ta


N
thấy O chính là trung điểm của AA’. Gọi D O

và P là giao điểm của AA’ với (ABC) và MN. P


I

M
2
Dễ thấy AM . AB  AA  AN . AC B A' C

Suy ra tứ giác BMNC nội tiếp.


D
 AMN  ACB
Mà ADB  ACB H K

Nên AMN  ADB


Suy ra MPDB nội tiếp.
2
Do đó ta có AP. AD  AM . AB  AA
Mà A, A’ và D cố định suy ra P cố định.
Gọi H là hình chiếu của K trên AA’.
1
Ta có OP.OH  OI .OK  ON 2  AA2
4
Mà O, P, A’ cố định suy ra H cố định.
Vậy K thuộc đường thẳng ( ) (cố định) qua H và vuông góc với AA’. Do đó (K,KA) qua
điểm cố định A’ là điểm đối xứng của A qua ( ) .
Câu 8. Cho đoạn AB cố định, M di động trên AB. Trên cùng một nửa mặt phẳng bờ AB
vẽ hai hình vuông MADE và MBHG. Hai đường tròn
ngoại tiếp hai hình vuông cắt nhau tại N. Chứng
minh đường thẳng MN luôn đi qua một điểm cố định G H
khi M di chuyển trên AB.
N
Hướng dẫn: E D

Giả sử MN cắt đường tròn đường kính AB tại I

Ta có ANM ADM 450 (góc nội tiếp cùng chắn


A M B
cung AM của đường tròn ngoại tiếp hình vuông
AMDE)

Tổng hợp các phương pháp chứng minh đường đi qua điểm cố định I Trang 20
THPT Chuyên Hoàng Lê Kha

Ta có BNM BGM 450 ( góc nội tiếp cùng chắn cung BM của đường tròn ngoại tiếp
hình vuông MBGH)

ANB ANM BNM 900 N thuộc đường tròn đường đường kính .
Vậy I là trung điểm cung AB (phần không chứa N) của (AB), mà (AB) cố định nên I cố
định.
Câu 9. Cho hai đường thẳng a, b cắt nhau tại O, điểm M không thuộc a, b và không
thuộc các đường phân giác của các góc tạo bởi a, b. Hai điểm A, B theo thứ tự thay đổi

trên a, b sao cho OMA OMB. Chứng minh rằng đường thẳng AB luôn đi qua một điểm
cố định.
Hướng dẫn
Qua M, vẽ đường thẳng vuông góc với OM.
Đặt C OM AB, D AB.

Vì góc AMO BMO và OMD 90 nên ta có:


( ABCD) 1.
Do đó: O( ABCD) 1.
Suy ra: (a, b, OM , OD) 1.
Từ đó, chú ý rằng a, b, OM cố định, ta có: OD cố định.
Mặt khác, cố định, do đó D cố định. Vậy, AB luôn đi qua một điểm cố định (điểm D).
Câu 10. Cho ba điểm thẳng hàng A, B, C theo thứ tự đó. Một đường tròn (O) thay đổi
nhưng luôn đi qua B và C. Từ A kẻ hai tiếp tuyến AM, AN đến đường tròn (O). Đường
thẳng MN cắt hai đoạn AO, AC lần lượt tại H và K. Chứng minh đường tròn ngoại tiếp
tam giác OHK luôn đi qua hai điểm cố định.
Hướng dẫn
Qua O Kẻ đường thẳng vuông góc với BC tại I ta có I là trung điểm của BC nên I cố
định.
Lại có tứ giác OHKI nội tiếp n

( OHK OIK 90 ) IOH HKA


o
hay AOI và AHK đồng dạng h

AK AO k
AK.AI AO.AH .
AH AI C I B A

Có ONA vuông, đường cao NH m

Tổng hợp các phương pháp chứng minh đường đi qua điểm cố định Trang 21
THPT Chuyên Hoàng Lê Kha

AO.AH AN2 .
Ta có AN2 AB.AC .
AB.AC
Từ đó suy ra AK.AI AB.AC AK = hằng số => K cố định. Vậy đường
AI
tròn ngoại tiếp tam giác OHK đi qua hai điểm cố định I, K.
Câu 11. Cho đường tròn tâm O, dây AB. Điểm M di chuyển trên cung lớn AB. Các
đường cao AE, BF của tam giác ABM cắt nhau ở H. Đường tròn tâm H bán kính HM cắt
MA, MB theo thứ tự ở C, D.
a) Chứng minh rằng đường thẳng kẻ từ M vuông góc với CD luôn đi qua một điểm
cố định.
b) Chứng minh rằng đường thẳng kẻ từ H và vuông góc với CD cũng đi qua một
điểm cố định.
Hướng dẫn
a) Kẻ tiếp tuyến Mx với đường tròn (O) x
M
Ta có M1 MAB
1
Có tứ giác ABEF nội tiếp đường tròn E

F D
đường kính AB, nên MEF MAB
H
Do đó MEF M1 , suy ra Mx//EF.
C O
Do đó OM EF
A B
Ta có H là tâm đường tròn ngoại tiếp tam
giác MCD, HE MD, nên E là trung điểm MD K

Tương tự F là trung điểm MC


Suy ra EF là đường trung bình tam giác MCD
Do đó EF//CD, mà OM EF
Suy ra OM CD. Do đó điểm cố định là O.
b) Gọi K là điểm đối xứng với O qua AB, ta có OK AB, mà MH AB. Suy ra MH//OK.
Lại có trong tam giác khoảng cách từ trực tâm tam giác đến đỉnh bằng 2 lần khoảng
cách từ tâm đường tròn ngoại tiếp đến cạnh tương ứng. Do đó MH = OK
Vậy tứ giác MHKO là hình bình hành. Suy ra HK//OM, mà OM CD, nên HK CD.
Vậy đường thẳng kẻ từ H vuông góc CD đi qua điểm K.
Do O, AB cho trước, nên K là điểm cố định.
Câu 12. Cho tam giác ABC, M là điểm bất kì thuộc đường tròn (O) ngoại tiếp tam
giác ấy. Gọi D là điểm đối xứng với M qua AB, E là điểm đối xứng với M qua BC.

Tổng hợp các phương pháp chứng minh đường đi qua điểm cố định Trang 22
THPT Chuyên Hoàng Lê Kha

Chứng minh rằng khi điểm M di chuyển trên đường tròn (O) thì DE luôn đi qua một điểm
cố định.
Hướng dẫn
Gọi H, I, K theo thứ tự là chân các đườngvuông góc kẻ từ M đến AB, AC, BC
Ta có H, I, K thẳng hàng (đường thẳng Simson).
Gọi N là trực tâm của tam giác ABC.

AN cắt (O) tại F. Ta có BCN BCF ,


suy ra BC là trung trực NF, D
H

mà BC là trung trực của ME. Suy ra E F1 N1


A M

Có F1 C1 (góc nội tiếp).


I

Có K1 C1 (tứ giác MCKI nội tiếp) N


1
O
1 1

B C
Suy ra K1 E , do đó NE//HK 1
K

Chứng minh tương tự có ND//HK F

Vậy D, N, E thẳng hàng. Hay DE đi qua trực tâm N


E
của tam giác ABC, nên DE đi qua điểm cố định.
Câu 13. Cho tam giác nhọn ABC có BC > CA. Điểm M chạy trên đoạn CA. Các điểm
A’, C’ theo thứ tự thuộc các đoạn BC, BA sao cho MC CA '; AM MC ' . Chứng
minh rằng đường tròn (A’BC’) luôn đi qua một điểm cố định.
Hướng dẫn
Ta có :
BC ' AB AC '
BA ' BC A ' C
2 cos A 2 cos A
AB AC '
BC CM
2 cos A 2 cos A
AB
AM BC CM
2 cos A
AB
AC BC ( không đổi).
2 cos A
Do đó áp dụng ví dụ 1 phần phương pháp vectơ ta có điều phải chứng minh.
Câu 14. Cho đường tròn tâm (O). Từ điểm A cố định ở ngoài (O) kẻ tiếp tuyến AB, AC
tới (O) (B, C tiếp điểm). Lấy điểm M trên cung nhỏ BC. Gọi D, E, F thứ tự là hình chiếu
từ M đến BC, AC, AB. Gọi MB cắt DF tại P, MC cắt DE tại Q. Chứng minh đường thẳng

Tổng hợp các phương pháp chứng minh đường đi qua điểm cố định Trang 23
THPT Chuyên Hoàng Lê Kha

nối giao điểm của hai đường tròn ngoại tiếp tam giác MPF và MQE luôn đi qua một điểm
cố định.
Hướng dẫn
Gọi đường tròn ngoại tiếp tam giác MPF và MQE cắt nhau tại M, N. Đường thẳng MN
cắt PQ, BC thứ tự tại K và I. Ta có các tứ giác MDCE, MDBF nội tiếp.
Từ các tứ giác nội tiếp và góc tạo bởi giữa tiếp tuyến và dây cung.

Suy ra MCE MDE MBC ; MBF MDF MCB


A
Suy ra PMQ PDQ PMQ PDM QDM

= PMQ MCB MBC 1800


Do đó MPDQ là tứ giác nội tiếp

Suy ra MQP MDP MCB E


N
Do đó PQ//BC F M
Từ MQP MCB MEQ .
K Q
Suy ra KQ là tiếp tuyến của đường tròn (MQE) P
B C
Chứng minh tương tự KP là tiếp tuyến của đường
D I
tròn (MPF)
Ta có KM. KN = KQ2, KM. KN = KP2. Suy ra KP = KQ. O
Xét tam giác MBC, PQ//BC, KP = KQ.
Suy ra I là trung điểm BC. Vậy MN đi qua điểm cố định I là trung điểm BC.

Câu 15. Cho tam giác ABC nội tiếp đường tròn (O). Điểm M chạy trên cung BC
không chứa A của (O). K, L theo thứ tự là tâm đường tròn nội tiếp các tam giác ABM,
ACM. Chứng minh rằng đường tròn (MKL) luôn đi qua một điểm cố định.
Hướng dẫn

Tổng hợp các phương pháp chứng minh đường đi qua điểm cố định Trang 24
THPT Chuyên Hoàng Lê Kha

Gọi (O’) là đường tròn tiếp xúc với các đoạn AB, AC (tại E, F) và tiếp xúc trong
với (O) (tại T). Gọi P, Q lần lượt là giao điểm thứ hai của TE, TF với (O); S là giao điểm
thứ hai của TA với (O’).
Dễ thấy K MP; L MQ; PA PK; QL QA; ES PA; FS QA và tứ giác
TESF điều hòa.

Do đó: KPT MPT MQT LQT

PK PA ES FS QA QL
PT PT ET FT QT QT

Suy ra: TPK ∽ TQL . Do đó MKT MLT .


Vậy (MKL) đi qua T cố định.

Tổng hợp các phương pháp chứng minh đường đi qua điểm cố định Trang 25
THPT Chuyên Hoàng Lê Kha

TÀI LIỆU THAM KHẢO

1. Nguyễn Mộng Hy (2003) – Các bài toán về phương pháp vectơ và phương
pháp tọa độ - Nhà xuất bản giáo dục.

2. Đoàn Quỳnh (Chủ biên) – Văn Như Cương – Trần Nam Dũng – Nguyễn Minh
Hà – Đỗ Thanh Sơn – Lê Bá Khánh Trình (2011) – Tài liệu chuyên toán hình học 10 –
Nhà xuất bản giáo dục.

3. Nguyễn Mạnh Hà (Chủ biên) – Nguyễn Xuân Bình – Bài tập nâng cao và một
số chuyên đề hình học 10 – Nhà xuất bản giáo dục.

4. Đỗ Thanh Sơn – Phép biến hình trong mặt phẳng – Nhà xuất bản giáo dục.

5. Các đề thi HSG trong nước và quốc tế.

6. Các bài viết tham khảo trên mạng .

Tổng hợp các phương pháp chứng minh đường đi qua điểm cố định Trang 26

You might also like